+ All Categories
Home > Documents > 8 & 9_MPC_SHP_E1 & E2& E4 & E5

8 & 9_MPC_SHP_E1 & E2& E4 & E5

Date post: 10-Feb-2017
Category:
Upload: ngomien
View: 300 times
Download: 1 times
Share this document with a friend
159
CLASS-VIII_SUBGROUP_E4 & E5 SIMPLE HOLIDAY PACKAGE NARAYANA GROUP OF SCHOOLS Page 1 of 40 (OLYMPIAD AND E-TECHNO) SIMPLE HOLIDAY PACKAGE PONGAL Mathematics Physics Chemistry CLASS – VIII E4 & E5 VIII_MPC_E4 to E5_SHP INDEX SUBJECT PAGE NO. 1. MATHEMATHICS 2 - 16 2. PHYSICS 17 - 30 3. CHEMISTRY 31 - 40
Transcript
Page 1: 8 & 9_MPC_SHP_E1 & E2& E4 & E5

CLASS-VIII_SUBGROUP_E4 & E5 SIMPLE HOLIDAY PACKAGE

NARAYANA GROUP OF SCHOOLS Page 1 of 40

(OLYMPIAD AND E-TECHNO)

SIMPLE HOLIDAY

PACKAGE PONGAL

Mathematics

Physics

Chemistry

CLASS – VIII E4 & E5

VIII_MPC_E4 to E5_SHP

INDEX

SUBJECT PAGE NO.

1. MATHEMATHICS 2 - 16

2. PHYSICS 17 - 30

3. CHEMISTRY 31 - 40

Page 2: 8 & 9_MPC_SHP_E1 & E2& E4 & E5

CLASS-VIII_SUBGROUP_E4 & E5 SIMPLE HOLIDAY PACKAGE

NARAYANA GROUP OF SCHOOLS Page 2 of 40

MATHEMATHICS ( Q.NO.1 TO 80)

1. In the figure, l||m||n and AB||CD. Then x° + w° is equal to

1) 360° – (y° + z°) 2) 2y° + z° 3) y° – z° 4) y° + z°

2. In the figure, l||m and p||q, then x° is

1) 90° 2) 100° 3) 110° 4) 120°

3. In the figure given below, ABCD EF. Then the value of x is

1) 280 2) 720 3) 560 4) 800

4. In the figure given below, ABCD and F = 300. Then ECD is

1) 1300 2) 1200 3) 1500 4) 600

Page 3: 8 & 9_MPC_SHP_E1 & E2& E4 & E5

CLASS-VIII_SUBGROUP_E4 & E5 SIMPLE HOLIDAY PACKAGE

NARAYANA GROUP OF SCHOOLS Page 3 of 40

5. In the figure given below, two straight lines PQ and RS intersect each

other at O. If POT = 750, then the value of c is

1) 84 2) 21 3) 48 4) 105

6. In the given figure, lines XY and MN intersects at O. If POY = 900 and a

: b = 2 : 3. Then c =

1) 1260 2) 540

3) 360

4) 1400

7. ABC is an isosceles triangle with AB = AC and C = 40º. BD is the

bisector of ABC intersects AC at D and AE is perpendicular to BD, then

EAD is

1) 20º 2) 25º 3) 30º 4) 35º

8. If the number of sides of a polygon is equal to the number of diagonals of

the same polygon, then the sum of angles of the polygon is

1) 540° 2) 720° 3) 900° 4) 1080°

9. A mathematics teacher cut a polygon into a triangle, a quadrilateral and

a pentagon along the two diagonals drawn in that polygon from the same

vertex. How many sides does the original polygon have ?

1) 14 2) 12 3) 10 4) 8

10. The side of the largest possible regular octagon that can be cut out of a

square of side 1cm is

1) 2 1 cm 2)

1cm

2 2 3)

1

cm2 1

4) None

11. In the triangle ABC, BC = CD and ABC BAD = 30° . The measure of

ABD is

1) 30° 2) 45° 3) 15° 4) can’t be determined

Page 4: 8 & 9_MPC_SHP_E1 & E2& E4 & E5

CLASS-VIII_SUBGROUP_E4 & E5 SIMPLE HOLIDAY PACKAGE

NARAYANA GROUP OF SCHOOLS Page 4 of 40

12. In ADE, ADE = 140° and the points B, C lie on sides AD, AE

respectively and points A, B, C, D, E are distinct. If lengths AB, BC, CD

and DE are all equal, then the measure of EAD is

1) 5° 2) 6° 3) 7.5° 4) 10°

13. If one of the interior angles of a regular polygon is equal to 5/6 times of

one of the interior angles of a regular pentagon, then the number of sides

of the polygon is

1) 3 2) 4 3) 6 4) 8

14. In a triangle, one of the angles is equal to half of the sum of the other

two angles. If the difference of the other two angles is 40°, then the smallest angle in the triangle is

1) 40° 2) 45° 3) 50° 4) 55°

15. In the given figure, the value of x is

1) 55o 2) 75o 3) 80o 4) 25o

16. In the given figure, AB || CD, AE || FC, FD || BE, then

1) x + y = 00 2) x – 2y = 00 3) x – y = 00 4) x – y = 180°

17. Triangles ABC and ABD are isosceles with AB = AC = BD, and BD inter

sects AC at E. If BD AC , then C D is

1) 115° 2) 120° 3) 130° 4) 135°

Page 5: 8 & 9_MPC_SHP_E1 & E2& E4 & E5

CLASS-VIII_SUBGROUP_E4 & E5 SIMPLE HOLIDAY PACKAGE

NARAYANA GROUP OF SCHOOLS Page 5 of 40

18. In the below figure, ABDC, BAC = 400 andBCD = 200 , then ACB =

1) 200 2) 300 3) 400 4) 600

19. In the below figure, ABCD EF, BCDE, BAC = 690 and DBC = 500,

then EGF =

1) 600 2) 500 3) 400 4) Can’t say

20. In the below figure AB QS CD, PAB = 600 and DCR = 600, then

QXY + QYX =

1) 900 2) 600 3) 800 4) 1000

21. Let X be any point on the side BC of a ABC if XM, XN are drawn parallel

to BA and CA meeting CA, BA is M, N respectively MN meets BC produced

in T. Then

A

B C

MN

Tx

1) 2TX = AN.AB 2) 2TX = TB.TC

3) 2TX = AM.MC 4) 2TX = NM.BC

Page 6: 8 & 9_MPC_SHP_E1 & E2& E4 & E5

CLASS-VIII_SUBGROUP_E4 & E5 SIMPLE HOLIDAY PACKAGE

NARAYANA GROUP OF SCHOOLS Page 6 of 40

22. In figure, ABCD is a parallelogram and X and Y are points on the diagonal

BD such that DX=BY. Then

1) logAXCY is paralle ram 2) AYB CXD

3) AXD CYB 4) All of these

23. In figure, ABCD is a trapezium in which AB CD and AD BC .Then

1) A B 2) ABC BAD

3) diagonal AC diagonal BD 4) All of these

24. In ,ABC AD is the median through A and E is the mid-point of AD. BE

produced meets AC in F. AF

1) 1

3AC 2)

1

2AC 3)

1

4AC 4) none

25. Let ABCD be a rectangle with AB = a and BC = b. Suppose r1 is the

radius of the circle passing through A and B and touching CD, and similarly r2 is the radius of the circle passing through B and C and

touching AD. Then r1 + r2 is

1) 5a+b

8 2)

5(a b)

8 3)

5(a b)

8 4) none

26. A circular park of radius 20 m is situated in a colony. Three boys. Ankur, Syed and David are sitting at equal distance on its boundary each having a toy telephone in his hands to talk to each other. Then the length of the string of each phone is

1) 20 3 m 2) 10 3 m 3) 10m 4) 20m

Page 7: 8 & 9_MPC_SHP_E1 & E2& E4 & E5

CLASS-VIII_SUBGROUP_E4 & E5 SIMPLE HOLIDAY PACKAGE

NARAYANA GROUP OF SCHOOLS Page 7 of 40

27. In the given figure, two circles with centres A and B and of radii

5cm and 3 cm touch each other internally. If the perpendicular

bisector of segment AB meets the bigger circle in P and Q. Then

the length of PQ is

1) 2cm 2) 4 6 cm 3) 2 2 cm 4) 4 2 cm

28.

In the given figure AB is a diameter of the circle with centre ‘O’. CD is a

chord equal to the radius of the circle. AC and BDWhen extended

intersect at a point E. Then ____AEB

1) 015 2)

030 3) 045 4)

060

29. For one root of ax2 + bx + c = 0 to be double the other, the coefficients a, b, c must be related as follows :

1) 4b2 = 9c 2) 2b2 = 9ac 3) 2b2 = 9a 4) b2 – 8ac = 0

30. The difference of the roots of x2 – 7x – 9 = 0 is

1) 7 2) 7

2 3) 9 4) 85

31. If α, β are the zeroes of the expression ax2 + bx + c, then the value

of

2α ββ α

is

1) 2

2 2

b 4ac

a c 2)

2 2

2 2

b 4a b

a c 3)

2 2b 4a b

ac 4)

2 2

2 2

b 4ac b

a c

Page 8: 8 & 9_MPC_SHP_E1 & E2& E4 & E5

CLASS-VIII_SUBGROUP_E4 & E5 SIMPLE HOLIDAY PACKAGE

NARAYANA GROUP OF SCHOOLS Page 8 of 40

32. If a and b are rational and , are the zeroes of x2 + 2ax + b, then

the expression with rational coefficients one of whose zero is 2 2 is

1) 2 4 2x ax b 2) 2 4 2x ax b 3) 2 4 2x ax b 4) 2 4 2x ax b

33. By using formula, the value of x in x(x2 – 1) (x + 2) + 1 = 0 is

1) 1 5

2 2)

2 5

2 3)

1 5

2 4) 1 5

34. The roots of Ax2 + Bx + C = 0 are r and s. For the roots of x2 + px

+ q = 0 to be r2 and s2, p must equal

1) 2

2

B 4AC

A 2)

2

2

B 2AC

A 3)

2

2

2AC B

A 4) B2 – 2C

35. Let the roots of ax2 + bx + c = 0 be r and s. The equation with roots ar + b and as + b is

1) x2 – bx – ac = 0 2) x2 – bx + ac = 0

3) x2 + 3bx + ca + 2b2 = 0 4) x2 + 3bx – ca + 2b2 = 0

36. If one root of the quadratic equation ax2 + bx + c = 0 is 'n' times

the other, then b2n is equal to

1) ac(n + 1) 2) ac(n + 1)2 3) c(n + 1) 4) c(n + 1)2

37. If α and β are the zeroes of x2 + px + q, then the expression

whose zeroes are 2α β and 2α +β is

1) x2 + 2x(p2 – 2q) + p2(p2 – 4q) 2) x2 – 2x(p – 2q) + p2

3) x2 – 2x(p2 – 2q) + p2(p2 – 4q) 4) x2 + 2x(p – 2q) – p2

38. Consider x2 + px + q = 0, where p and q are positive numbers. If the roots of this equation differ by 1, then p equals

1) 4q 1 2) q – 1 3) 4q 1 4) q + 1

39. The sum of the squares of the roots of the equation x2 + 2hx = 3 is 10. The absolute value of h is equal to

1) – 1 2) 1

2 3)

3

2 4) None

40. If

2 2x 2x+1 x 2x 1 22+ 3 + 2 3 =

2 3, then x is

1) 0 2) 1 3) 2 4) Both (1) and (3)

41. Two equal circles of radius r intersect such that each passes through the centre of the other. The length of the common chord of the circles is

1) r 2) 2rAB 3) 3r 4) 3

2r

42. If AB is a chord of a circle, P and Q are the two points on the circle different form A and B, then

1) APB AQB

2) 0180APB AQB or APB AQB

3) 090APB AQB

4) 0180APB AQB

Page 9: 8 & 9_MPC_SHP_E1 & E2& E4 & E5

CLASS-VIII_SUBGROUP_E4 & E5 SIMPLE HOLIDAY PACKAGE

NARAYANA GROUP OF SCHOOLS Page 9 of 40

43. The larger root minus the smaller root of the equation

27 +4 3 x + 2+ 3 x 2 = 0 is

1) 2 3 3 2) 2 3 3) 6 3 3 4) 6 3 3

44. If a, b, cQ and a + b + c = 0, then the roots of

2 b+c a x + c+a b x + a + b c = 0 are

1) Imaginary 2) irrational 3) Rational 4) Nothing can be said about the

roots 45. The nature of the roots of(2y + 5)(3y – 4) = (y – 3)(y + 2) is 1) real and equal 2) real and distinct 3) not real 4) none

46. If a, b, c and a + b + c = 0, then the quadratic equation 4ax2 + 3bx + 2c = 0 has

1) Real roots 2) imaginary roots 3) Can't be determined 4) None of these 47. In the given figure the value of x is

1) 040 2) 050 3) 060 4) 070 48. In the given figure, AB is a diameter of the circle C (O,r) and the

radius OD is perpendicular to AB. If C is any point on arc DB, then BAD =

1) 045 2) 050 3) 060 4) 070

D

C x

O

BA40

0

D

C

BAO

Page 10: 8 & 9_MPC_SHP_E1 & E2& E4 & E5

CLASS-VIII_SUBGROUP_E4 & E5 SIMPLE HOLIDAY PACKAGE

NARAYANA GROUP OF SCHOOLS Page 10 of 40

49. A circle touches a quadrilateral ABCD internally, then the true statement in the following is

1) AB + BC = CD + AD 2) AB + CD = BC + AD 3) BD = AC 4) None of these

50. In the below figure, O is the centre of the circle and AB and CD are two equal chords intersecting at P and M, N are the mid points of AB, CD respectively, then

1) PB = PD and PA = PC 2) PB = PD and PA = CD 3) PA = PB and PC = PD 4) None of these 51. If ABCD is a square with M is a mid-point of DC. And AE = BF. Then

which of the following is not false

1) ME = MF 2) ME = MD 3) MF = MC 4) MF = AF 52. ABCD is a parallelogram. E is the midpoint of AB, then

1) CF = 2AB 2) CF = 2 CD 3) CF = 2AD 4) CF = 2 BE

I

D

A

C

P

N

O

M

B

MD C

F

BA

E

D

CFB

A

E

Page 11: 8 & 9_MPC_SHP_E1 & E2& E4 & E5

CLASS-VIII_SUBGROUP_E4 & E5 SIMPLE HOLIDAY PACKAGE

NARAYANA GROUP OF SCHOOLS Page 11 of 40

53. In the given figure BAD = EAC, then

1) x = 11, y = 21 2) x = 11, y = 23

3) x = 11, y = 20 4) x = 11, y = 22

54. ABCD is a square of side 2 units. Midpoints of sides are joined to form another square. Again midpoints of that square are joined to form another smaller square. Midpoints are again joined to form

another square. This process is continued to get the 2016th square

inside. The area of that 2016th square is A

1

2, then A =

1) 2016 2) 2014 3) 2018 4) None

55. Triangle formed by joining the midpoints of equal sides of an isosclas triangle is

1) Isosceles 2) Equilateral 3) Scalene 4) Right

56. Two regular polygons are such that the ratio between their number of sides is 1 : 2 and the ratio of measures of their interior angles is 3 : 4. The number of sides of each polygon is:

1) 5,10 2) 6,12 3) 4,8 4) 2,3 57. In the following figure ABCD is a trapezium, EF || AD and EF ||BC, then

x, y, z are respectively equal to

1) 50°, 60°, 50° 2) 50°, 50°, 80°

3) 60°, 50°, 60° 4) none

58. In the given figure, PSR = 60º and PRQ = 120º and given that RPS

= 2RPQ. Then PQR is

1) 60º 2) 50º

3) 40º 4) 30º

A

432y +

3

BD10

E x - 1C

Page 12: 8 & 9_MPC_SHP_E1 & E2& E4 & E5

CLASS-VIII_SUBGROUP_E4 & E5 SIMPLE HOLIDAY PACKAGE

NARAYANA GROUP OF SCHOOLS Page 12 of 40

59. In the below figure, the line BE bisects ABD and BED = 4ABE. If

ABCD is rectangle then BDC is

1) 18º 2) 24º

3) 60º 4) 36º

60. In figure, ABCD is a rhombus in which the diagonal

DB is produced To E. If ABE = 160º , then x, y and z are respectively

1) 700, 400, 400 2) 400, 700, 200

3) 700, 200, 900 4) 600, 700, 200

61. ABCD is a rectangle whose area is 108m2 and length is 12m. Then the

area of the square AEFC is

62. Match List - I with List - II and select the correct answer using the codes

given below the lists

Codes:

1) A B C D

4 3 1 2

2) A B C D

2 4 3 1

3) A B C D

3 4 2 1

4) A B C D

3 4 1 2

Page 13: 8 & 9_MPC_SHP_E1 & E2& E4 & E5

CLASS-VIII_SUBGROUP_E4 & E5 SIMPLE HOLIDAY PACKAGE

NARAYANA GROUP OF SCHOOLS Page 13 of 40

63. In ABC, AB = AC and A, D, B are on the same line, then which of the

following statement is not false

1) DC > DB 2) DC < DB

3) DC = DB 4) None

64. In a triangle ABC, E is the mid - point of median AD, then ar (

BED ) =

1) 1

3 ar( ABC)

2) 1

4 ar( ABC)

3) 2

5 ar( ABC)

4) None

65. AB and CD intersect each other at O in a plane , M is a

point such that M O and M . Then (AM + BM + CM

+ DM)

1) = (AO + BO + CO + DO) 2) = 2(AO + BO + CO + DO)

3) < (AO + BO + CO + DO) 4) >(AO + BO + CO + DO)

66. H is the orthocentre of ΔABC and X, Y, and Z are respectively the

midpoints of AH, BH and CH. Then the point H is

1) Circumcentre of XYZ 2) Centroid of XYZ

3) Orthocentre of XYZ 4) Incentre of XYZ

67. If two medians of a triangle are equal, then the triangle is

1) Scalene 2) Isosceles

3) Equilateral 4) Right angled

68. ABCD is a square. X and Y are points on sides AD and BC respectively

such that AY = BX. Then BAY =

1) ABX 2) AXB

3) ABX + AXB 4) ABX – AXB

69. ABCD is a parallelogram. P is a point on AD such that AP =

1

3 AD. Q is a

point on BC such that CQ = 1

3BC. Then AQCP is a

1) Rhombus 2) Rectangle

3) parallelogram 4) square

70. In quadrilateral ABCD with diagonals AC and BD intersecting at O, BO =

4, OD = 6, AO = 8, OC = 3 and AB = 6. The length of AD is

1) 166 2) 10

3) 6 3 4) None

Page 14: 8 & 9_MPC_SHP_E1 & E2& E4 & E5

CLASS-VIII_SUBGROUP_E4 & E5 SIMPLE HOLIDAY PACKAGE

NARAYANA GROUP OF SCHOOLS Page 14 of 40

71. In the given figure, PQRS is a parallelogram. If T is the midpoint of RQ

and the line ST when extended meets PQ at U, then the ratio of lengths

of PQ and QU is

1) 1 : 1 2) 1 : 2

3) 1 : 3 4) 1 : 4

72. If PQRS is a square where PX = QY = RZ, XYZ = 90°, then XZY is

1) 60° 2) 45°

3) 30° 4) 75°

73. Two vertical poles 20m and 80m high stand apart on a horizontal plane.

The height of the point of intersection of the lines joining the top of

each pole to the foot of the other is 'a' metres. Then 'a' is

1) 30 metres 2) 20 metres

3) 25 metres 4) 16 metres

74. In right triangle ABC, , ,BC = 5, AC = 12 AM = x MN AC, NP BC , N is

on AB. If y = MN+ NP , one half the perimeter of rectangle MCPN, then

1) 1y 5 12

2

2) 5x 12y

12 5

3) 144 7xy

12

4) None

Page 15: 8 & 9_MPC_SHP_E1 & E2& E4 & E5

CLASS-VIII_SUBGROUP_E4 & E5 SIMPLE HOLIDAY PACKAGE

NARAYANA GROUP OF SCHOOLS Page 15 of 40

75. If angles A, B, C, D of a quadrilateral ABCD taken in order are in the ratio 3 : 7 : 6 : 4, then ABCD is a

1) Rhombus 2) Parallelogram

3) Trapezium 4) kite

76. ABCD is a rectangle. The diagonals AC and BD are intersect at O. angle DOC = 1620 – 3x and angle CAB = 2x. Then x =

1) 540 2) 360

3) 2400 4) 180

77. PQRS is a parallelogram. Then y equals.

1) 270 2) 610

3) 410 4) 280

78. ABCD is a rhombus. 02 15 ,DAB x 03 30 ,DCB x BDC equals

1) 450 2) 350

3) 37.50 4) 42.50

79. PQRS is a kite. 070 ,P 090.5 ,S R equals

1) 099 2) 091

3) 0111 4) 1090

80. ABCD is a rectangle with 048 .BAC Then DBC equals

1) 380 2) 480

3) 1320 4) 420

Page 16: 8 & 9_MPC_SHP_E1 & E2& E4 & E5

CLASS-VIII_SUBGROUP_E4 & E5 SIMPLE HOLIDAY PACKAGE

NARAYANA GROUP OF SCHOOLS Page 16 of 40

VIII_MATHEMATICS_SHP_E4 & E5_KEY

Q.No. 1 2 3 4 5 6 7 8 9 10

Key 4 3 1 2 3 1 3 1 4 1

Q.No. 11 12 13 14 15 16 17 18 19 20

Key 3 4 2 1 4 3 4 2 4 2

Q.No. 21 22 23 24 25 26 27 28 29 30

Key 2 4 4 1 1 1 2 4 2 4

Q.No. 31 32 33 34 35 36 37 38 39 40

Key 4 2 3 3 2 2 3 1 4 4

Q.No. 41 42 43 44 45 46 47 48 49 50

Key 3 2 4 3 2 1 2 1 2 1

Q.No. 51 52 53 54 55 56 57 58 59 60

Key 1 3 3 2 1 1 1 4 3 3

Q.No. 61 62 63 64 65 66 67 68 69 70

Key 2 1 1 2 4 3 2 1 3 1

Q.No. 71 72 73 74 75 76 77 78 79 80

Key 1 2 4 3 3 4 1 3 4 4

Page 17: 8 & 9_MPC_SHP_E1 & E2& E4 & E5

CLASS-VIII_SUBGROUP_E4 & E5 SIMPLE HOLIDAY PACKAGE

NARAYANA GROUP OF SCHOOLS Page 17 of 40

PHYSICS ( Q.NO. 1 TO 80)

1. A solid body is found floating in water with 3

4

th

of it s volume submerge

the same solid is found floating in a liquid with 3

4

th

of its volume above

the liquid surface . the specific gravity of the liquid is

1) 1/3 2) 3/2 3) 3 4) 2/3

2. A sphere of solid material of relative density 16 has a concentric spherical cavity and sinks in water. if the radius of the sphere be R, then the radius of cavity __________

1) 3 316r R 2)

3 316 / 5r R 3)

3 315

16r R 4)

3 316

15r R

3. A hemispherical portion of radius R is removed from the bottom of a

cylinder of radius R. the volume of the remaining cylinder is V and mass

m. it is suspended by a string in a liquid of density , where it stays

vertical. The upper surface of cylinder is at a depth h below the liquid

surface. The force on the bottom of the cylinder by the liquid is

1)

2g V R h 2) mg 3) mg V g 4) Mg+2R h

g

4. A wooden block of volume 1000 cm3 on air. It is suspended from spring

balance. It’s weight 12N in air .it is suspended in water such that half of the block is below the surface of water. The reading of the balance is___________________

1) 10N 2) 9N 3) 8N 4) 7 N

5. The force does water exert on the base of a house tank of base area 2.5m2 when it is filled with water up to a height of 2m if ( g=10 m/s2)

1) 500 kg wt 2) 1500 kg wt 3) 4500 kg wt 4) 5000 kg wt

6. A piece of gold-aluminium alloy weighs 100 grams in air and 80 g when

immersed in water. What is the weight of gold in the alloy ? (Sp . gravity of gold = 19.3 and of aluminum =2.5).

1) 57.44 g 2) 2.5 g 3) 20 g 4) 38.6 g

7. A body of density ‘d1’ is counter poised by Mg of weights of density ‘d2’ in

air of density ‘d’ then the true mass of the body is

1) M 2)

1

2

1d

Md

3)

1

1d

Md

4)

2

1

1

1

d

dM

d

d

Page 18: 8 & 9_MPC_SHP_E1 & E2& E4 & E5

CLASS-VIII_SUBGROUP_E4 & E5 SIMPLE HOLIDAY PACKAGE

NARAYANA GROUP OF SCHOOLS Page 18 of 40

8. Two bodies with volumes 2V and 4V are equalized on a balanc . the larger

body is then immersed in oil of density 1.6 g/cm3 . what must be the density of liquid in which the smaller body is simultaneously immersed

so as not to disturb the equilibrium of the balance ?

1) 0.9 g/m3 2) 3.2 g/cm3 3) 0.27 g/cm3 4) 1.764 g/cm3

9. A silver ornament is suspected to be hollow. Its weight is 288.75 and it

can displace 30 cc of water. If the specific gravity of silver is 1.5 ,find

the volume of the cavity.

1) 27.5 cc 2) 10.5 cc 3) 19.5 cc 4) 2.5 cc

10. A block of silver of mass 4 kg hanging from a string is immersed in a

liquid of relative density 0.72. If relative density of silver is 10, then

tension in the string will be

1) 37.12 N 2) 42 N 3) 73 N 4) 21 N

11. How much lead ( S.G. = 11) should be added to a place of cork ( S.G.

=0.2) weighing 10 grams so that it may just float on water?

1) 22 g 2) 44 g 3) 66 g 4) 88 g

12. In air a metallic Sphere with an internal cavity weigh’s 40 g and in water

it weigh’s 20 g . The volume of cavity if the density of material with

cavity be 8 g/cm is

1) Zero 2) 15 cm3 3) 5 cm3 4) 20 cm3

13. Density of ice is 900 kg/m3. A piece of ice is floating in water of density

1000 kg/m3. Find the fraction of volume of ice outside the water

1) 0.1 2) 0.9 3) 0.2 4) 0.8

14. Two bodies of equal volume A and B float on water in Equilibrium 2/3 rd of the volume of A and half the volume of B in inside water. The ratio of

densities of A and B is

1) 1:3 2) 3:2 3) 3:4 4) 4:3

15. A cube of wood supporting a 200 g mass just floats in the water. When

the mass is removed the cube raises by 2 cm, then length of side of the

cube is

1) 8 cm 2) 12 cm 3) 6 cm 4) 10 cm

16. The minimum horizontal acceleration of the container so that the

pressure at point A of the container becomes atmospheric is

1) (3/2) g 2) (4/3) g 3) (4/2) g 4) (3/4) g

Page 19: 8 & 9_MPC_SHP_E1 & E2& E4 & E5

CLASS-VIII_SUBGROUP_E4 & E5 SIMPLE HOLIDAY PACKAGE

NARAYANA GROUP OF SCHOOLS Page 19 of 40

17. A sphere of solid material of specific gravity 8 has a concentric

spherical cavity and just sinks in water . The ratio of radius of

cavity to that to outer radius of the sphere must be

1)

1/37

2 2)

1/35

2 3)

1/39

2 4)

1/33

2

18. A breaker of mass 1 kg contains 2kg of water and rests on a scale. A 2 kg

block of aluminum (specific gravity 2.70) suspended from a spring scale

is submerged in water , as shown in figure. Find the reading of both

scales.

1) 36-66N & 12.34N 2) 16-66N& 10-34N

3) 18-33N & 6.17N 4) 9N& 6N

19. The area of cross section of the pump plunger and press plunger of a hydraulic press are 0.02 m2 and 8 m2 respectively. If the hydraulic press

overcomes a load 800 kgf. Calculate the force acting on pump plunger

1) 2 kgf 2) 5 kgf 3) 0.25 kgf 4) 5 gf

20. A hollow cylinder of copper of length 25 cm and area of cross section 15 cm2 floats in water with 3/5 of its length inside water then find weight

of cylinder?

1) 200 gf 2) 300 gf 3) 225 gf 4) 325 gf

21. An iceberg is floating partially immersed in sea water. The density of sea water is 1.03 g cm-3 and that of ice is 0.92 g cm-3. The approximate

percentage of total volume of iceberg above the level of sea water is

1) 8 2) 11 3) 34 4) 89

22. A tank accelerates upwards with acceleration a= 2m/s2 contains water .A

block of mass 2kg and density 0.9 gm/cm2 is held stationary inside the

tank with the help of the string as shown. The tension in the string is

1) 5.5N 2) 2.7N 3) 4N 4) 3.33N

23. You plung a basket ball be near the surface of a swimming pool until half the volume of the basket ball is submerged if the basket ball has a radius

12 cm . what is the buoyancy force on the ball due to the water ?

1) 25N 2) 35N 3) 15N 4) 45N

Page 20: 8 & 9_MPC_SHP_E1 & E2& E4 & E5

CLASS-VIII_SUBGROUP_E4 & E5 SIMPLE HOLIDAY PACKAGE

NARAYANA GROUP OF SCHOOLS Page 20 of 40

24. An ferry boat has internal volume 1m3 and weight 50kg neglecting the

thickness of wood , find the fraction of the volume of boat immersed in

water.

1) 0.4m3 2) 0.7m3 3) 0.5m3 4) 0.9m3

25. A convex mirror has its radius of curvature 20cm. find the position of the image of an object placed at a distance of 12 cm from the mirror

1) 50

11cm

2) 70

11cm

3) 60

11cm

4) 80

11cm

26. A light ray is incident on a glass sphere of refractive index = 3 at an

angle of incidence 600

as show in fig find the angle r, r 1 ,e and the total deviation after two refractions

1) 0 0 0 030 ,30 ,60 ,60 2) 0 0 0 060 ,60 ,30 ,30

3) 0 0 0 030 ,60 ,30 ,60     4) 0 0 0 060 ,90 ,30 ,60 

27. A fish under water at a depth of 20 cm can see the outer atmosphere

through an aperture of radius of (critical angle of water c= 045 )

1) 5 cm 2) 20 cm 3) 10 cm 4) 15 cm

28. A beam of light passes from medium ‘1’ to medium ‘2’ to medium ‘3’ as show in fig . what may be concluded about the three indices of refraction

1, 2n n and 3n

1) 3 1 2n n n 2)

1 3 2n n n

3) 2 3 1n n n 4)

2 1 3n n n

Page 21: 8 & 9_MPC_SHP_E1 & E2& E4 & E5

CLASS-VIII_SUBGROUP_E4 & E5 SIMPLE HOLIDAY PACKAGE

NARAYANA GROUP OF SCHOOLS Page 21 of 40

29. A cube of ice floats partly in water and partly in k. oil. Find the ratio of

the volume of ice immersed in water to that in k.oil .specific gravity of

k.oil is 0.8 and that of ice is 0.9

1. 1:2 2. 2:3

3. 1:1 4. 4:3

30. A metal piece of mass 240 g lies in equilibrium inside a glass of water.

The piece touches the bottom of the glass at a small number of points. If

the density of the metal is 8000 kg / 3m , find the normal force exerted by

the bottom of the glass on the metal piece.

1 1.4 N 2 1.6 N

3 2.1 N 4 1.2 N

31. An object is placed on the principal axis of a concave mirror of focal

length 10 cm at a distance of 8cm from the pole. Find the position and

nature o the image

1. 40cm, real 2. 40cm, virtual

3. 20cm, virtual 4. 20cm, real

32. A ball of mass ‘m’ and density ‘ ’ is immersed in a liquid of density ‘3 ’

at a depth ‘h’ and released. To what height will the ball jump up above

the surface of liquid? (Neglect the resistance of water and air)

1. 2h 2. 3h

3. 4h 4. H

Page 22: 8 & 9_MPC_SHP_E1 & E2& E4 & E5

CLASS-VIII_SUBGROUP_E4 & E5 SIMPLE HOLIDAY PACKAGE

NARAYANA GROUP OF SCHOOLS Page 22 of 40

33. A ray of light is incident normally on one of the faces of a prism of

apex angle 300 and refractive index 2 . The angle of deviation of

the ray is

1) 300 2) 150 3) 600 4) 900

34. If the angle of incidence and angle of refraction at the first

refracting surface are 450 and 300 respectively, then refractive

index of the material of the prism is

1) 2 2) 3 3) 2 4) 1

2

35. The angle of a prism is 80 and its refractive index is 1.5 . The angle

of deviation produced by the prism is

1) 30 2) 90 3) 40 4) 150

36. A ray of light is incident on one of the faces of the prism with an

angle 450 with the surface. The angle of the prism is 600. The

emergent ray is deviated through an angle 420. The angle of

emergence is

1) 480 2) 380 3) 00 4) 570

37. For total internal reflection to take place, the angle of incidence i

and refractive index of the medium must satisfy, the inequality

1) 1

Sini 2)

1

Sini 3) Sini 4) Sini

38. Figure represents three cases of a ray passing through a prism of

refractive edge A. The case corresponding to minimum deviation is

1 2 3

1) 1 2) 2 3) 3 4) None of these

Page 23: 8 & 9_MPC_SHP_E1 & E2& E4 & E5

CLASS-VIII_SUBGROUP_E4 & E5 SIMPLE HOLIDAY PACKAGE

NARAYANA GROUP OF SCHOOLS Page 23 of 40

39. Light is incident normally on face AB of a prism as shown. A liquid

of refractive index is place on face AC of the prism. The prism

is made of glass of refractive index 3

2

The limit of for which total internal reflection takes place on

face AC is

1) 3

4 2)

3 3

4 3) 3 4)

3

2

40. Find the angle of deviation suffered by the light ray shown in

figure. The refractive index = 1.5 for the prism material

1) 40 2) 30 3) 3.50 4) 00

41. The angle of minimum deviation from a prism is 370. If the angle

of prism is 530, find the refractive index of the material of the

prism. (sin 26.50 =0.446)

1) 1.9 2) 1.6 3) 2.3 4) 4.2

42. When the angle of incidence i1 = 00, the ray is incident on the

prism along the normal (i.e. normal incidence). If r2 = (critical

angle), then angle of emergence is

2i

1) 00 2) 450 3) 900 4) 1800

Page 24: 8 & 9_MPC_SHP_E1 & E2& E4 & E5

CLASS-VIII_SUBGROUP_E4 & E5 SIMPLE HOLIDAY PACKAGE

NARAYANA GROUP OF SCHOOLS Page 24 of 40

43. An equilateral prism of refractive index 3

5 is immersed in water of

refractive index 4

3. If a ray of light passing through the prism

undergoes minimum deviation the angle of incidence on the first

refracting face of the prism is

1) 1 5

8Sin

2) 300 3) 450 4) 1 0.225Sin

44. A glass prism of refractive index 1.5 is placed in water of refractive

index 1.33. The minimum value of the angle of the prism so that it

will not be possible to have any emergent ray is 1 0( 0.89 62.5) Sin

1) 1500 2) 1650 3) 1800 4) 1250

45. A prism is made up of material of refractive index 3 . The angle of

the prism is A. If the angle of minimum deviation is equal to angle

of prism, then the value of A is

1) 600 2) 300 3) 450 4) 750

46. The maximum value of index refraction of a material of a prism

which allows the passage of light through it when the refracting

angle of prism is A is

1) 21 cot2

A 2) 21 cos

2

A 3) 21 tan

2

A 4) 0

47. The thin prism P1 of angle 40 and refractive index 1.54 is combined

with another thin prism P2 of refractive index 1.72 to produce

dispersion without deviation. The angle of P2 is

1) 40 2) 30 3) 2.6 4) 5.33

48. The angle of minimum deviation when light is incident at an angle

of 450 on one of the refracting faces of an equilateral prism of

refractive index 1.414 is

1) 300 2) 400 3) 450 4) 500

49. For a prism, A = 600, 7

3 . Then the minimum possible angle of

incidence, so that the light ray is refracted from the second

surface

1) 300 2) 600 3) 900 4) 400

Page 25: 8 & 9_MPC_SHP_E1 & E2& E4 & E5

CLASS-VIII_SUBGROUP_E4 & E5 SIMPLE HOLIDAY PACKAGE

NARAYANA GROUP OF SCHOOLS Page 25 of 40

50. A thin prism of 200 angle gives deviation of 5.20. The value of

refractive index of the material of the prism is

1) 7.6 2) 2.9 3) 4.5 4) 1.26

51. A ray of light passes through an equilateral glass prism, such that

angle of incidence is equal to angle of emergence. If the angle of

emergence is 3

4 times angle of prism . The refractive index of the

glass prism

1) 1.414 2) 1.5 3) 1.616 4) 1.7

52. The angle of minimum deviation produced by a prism is 300. If the

angle of prism is also 300, the refractive index of the material of

the prism is 0( 15 0.2588)Sin

1) 8 2) 7 3) Sin(150) 4) 1.93

53. Two mirrors labelled 1L for left mirror and 2L for right mirror in

the figure are parallel to each other and m apart. A person

standing 1.0 m from the right mirror 2L looks into this mirror

and sees a series of images. The second nearest image in the right

mirror is situated at a distance

1) 2.0 m from the person 2) 4.0 m from the person

3) 6.0 m from the person 4) 8.0 m from the person

54. Two bodies A and B are moving towards a plane mirror with

speeds AV and BV respectively as shown in the figure. The speed

of image of A with respect to the body B is

1) A BV V 2)

A BV V 3)

2 2

A BV V 4)

2 2

A BV V

Page 26: 8 & 9_MPC_SHP_E1 & E2& E4 & E5

CLASS-VIII_SUBGROUP_E4 & E5 SIMPLE HOLIDAY PACKAGE

NARAYANA GROUP OF SCHOOLS Page 26 of 40

55. Two plane mirrors are at an angle of 030 . Find the deviation

produced in the ray after 3 successive reflections as shown in

figure.

1) 800 2) 1600 3) 2000 4) 1000

56. The weight of a body in water is one third of its weight in air

density of body is? 2 0

1H

gm

cc

1) 0.6gm

cc 2)

0.8gm

cc 3)

1.5gm

cc 4) none of the above

57. A point object is moving with a speed 2cms-1 in front of a combination of two plane mirrors as shown in the figure. The magnitude of velocity of image in mirror M1 with respect to image

formed by mirror M2.

1) 2 cms-1 2) 2 3 cms-1 3) 4 cms-1 4) 8 cms-1

58. A cubic body floats on mercury with 0.25 of its volume below the surface. What fraction of the volume of the body will be immersed in the mercury if a layer of water poured on the top of mercury

that covers the body completely 2

16 / , 1Hg H o

gmgm cc

cc

1) 0.4 2) 0.3 3) 0.2 4) None 59. A light ray is incident on a plane mirror M. The mirror is rotated in

the direction shown with angular velocity 18 rads-1. Find the

speed of spot on the wall, when the angle of incidence is 370. (sin

370=3/5)

1) 1000 m/s 2) 1500 m/s 3) 500 m/s 4) 2000 m/s

Page 27: 8 & 9_MPC_SHP_E1 & E2& E4 & E5

CLASS-VIII_SUBGROUP_E4 & E5 SIMPLE HOLIDAY PACKAGE

NARAYANA GROUP OF SCHOOLS Page 27 of 40

60. Which of the following graphs represents the variation of deviation ( ) with angle of incidence i incase of reflection from a plane

mirror.

1) 2)

3) 4) 61. An object is placed on the principal axis of concave mirror of focal length

10 cm, at a distance of 8.0 cm from the pole. The position of the image is

1) 20 cm 2) 40cm

3) 60 cm 4) 80 cm

62. The focal length of a concave mirror is 12 cm. Where should an object of

length 4 cm be placed so that 1cm long image is formed.

1) 48 cm 2) 30 cm

3) 60 cm 4) 15 cm

63. A point object is placed at a distance of 20 cm from a convex mirror of

focal length 20 cm. The image will be formed at

1) Infinity 2) Focus

3) 15 cm behind mirror 4) 10 cm behind mirror

64. When an object lies at focus of a concave mirror, then the position of the

image formed and its magnification are

1) Pole and unity 2) Infinity and unity

3) Infinity and infinity 4) Centre of curvature & unity

65. Figure shows two rays A and B being reflected by a mirror and going as A'

and B'. The mirror

1) is plane 2) is convex

3) is concave 4) may be any spherical mirror

Page 28: 8 & 9_MPC_SHP_E1 & E2& E4 & E5

CLASS-VIII_SUBGROUP_E4 & E5 SIMPLE HOLIDAY PACKAGE

NARAYANA GROUP OF SCHOOLS Page 28 of 40

66. An object is 30 cm from a spherical mirror, along the central axis. The

absolute value of lateral magnification is 1

2. The image produced is

inverted. What is the focal length of the mirror ?

1) 20 cm 2) -10 cm

3) 15 cm 4) -5 cm

67. In image formation from spherical mirror, only paraxial rays are considered

because they

1) Are easy to handle geometrically 2) Contain most of the intensity of

the incident light

3) Form nearly a point image of a

point source

4) Show minimum dispersion effect.

68. How far should an object be from a concave spherical mirror of radius

36cm to form a real image one – ninth of its size ?

1) 180 cm 2) 90 cm

3) 180 m 4) 90 m

69. A concave mirror having a radius of curvature 40 cm is placed infront of a

illuminated point source at a distance of 30 cm from it. The image

distance is

1) 40 cm 2) 50 cm

3) 60 cm 4) 70 cm

70. An object is placed at 10cm infront of a concave mirror of radius of

curvature 15 cm the position of image (v) and its magnification (m) are

1) v= 30cm, m=3 (real , inverted) 2) v=20cm, m=3 ( virtual, erect)

3) v= 10 cm, same size (real, inverted) 4) v=10 cm, same size ( virtual, erect)

71. A spherical mirror forms an image of magnification 3. The object distance,

if focal length of mirror is 24 cm

1) 32 cm & 24 cm 2) 32 cm & 16 cm

3) 32 cm only 4) 16 cm only

72. An object is 20 cm from a convex mirror of focal length 10 cm. The image

formed by the mirror is

1) Real and 20 cm from the mirror 2) Virtual and 20 cm from the mirror

3) Virtual and

20

3 cm from the mirror

4) Real and at

20

3 cm from the

mirror

73. A U - shaped wire is placed before a concave mirror having radius of

curvature 20 cm as shown in figure. Find the total length of the image.

1) 20 cm 2) 10 cm

3) 30 cm 4) 40 cm

Page 29: 8 & 9_MPC_SHP_E1 & E2& E4 & E5

CLASS-VIII_SUBGROUP_E4 & E5 SIMPLE HOLIDAY PACKAGE

NARAYANA GROUP OF SCHOOLS Page 29 of 40

74. A person looks into spherical mirror . The size of image of his face is twice

the actual size of his face. If the face is at a distance of 20 cm. Then find

radius of curvature of the mirror ?

1) 40cm 2) 60cm

3) 80 cm 4) 100 cm

75. An image of a candle on a screen is found to be doubled its size. When the

candle is shifted by a distance of 5 cm, then the image becomes triple its

size the radius of curvature of the mirror is

1) 30 cm 2) 40 cm

3) 50 cm 4) 60 cm

76. An object of length 2.5 cm is placed at a distance of 1.5f from a concave

mirror where f is the magnitude of focal length of the mirror. The length

of the object is perpendicular to the principal axis. The length of the image

is

1) -2.5 cm 2) – 5 cm

3) - 10 cm 4) – 15 cm

77. Figure shows a spherical concave mirror with its pole at ( 0, 0) and

principal axis along x- axis there is a point object at ( -40 cm , 1cm ) the

position of image in co-ordination from is

1) 7 7

,40 1cm cm

2) 40 1

,7 7cm cm

3) 80 10,

7 7cm cm

4) 80 10

,7 7cm cm

78. A point object is placed 60 cm from the pole of a concave mirror of focal

length 10 cm on the principal axis. The image is located at

1) -6 cm 2) -12 cm

3) -24 cm 4) -48 cm

Page 30: 8 & 9_MPC_SHP_E1 & E2& E4 & E5

CLASS-VIII_SUBGROUP_E4 & E5 SIMPLE HOLIDAY PACKAGE

NARAYANA GROUP OF SCHOOLS Page 30 of 40

79. A concave mirror is of focal length 20 cm. A real object is placed at a

distance of 20 cm infront of the mirror from the pole. The mirror produces

an image at

1) Infinity 2) 20 cm

3) 40 cm 4) 10 cm

80. The distance between a real object and image in a convex mirror of focal

length 12 cm is 32 cm the size of the image if the object size is 1 cm

1) 1/2 cm 2) 1/3 cm

3) 2/3 cm 4) 1 cm

VIII_PHYSICS_SHP_TERM_I_KEY

Q.No. 1 2 3 4 5 6 7 8 9 10

Key 3 3 1 4 4 1 4 2 4 1

Q.No. 11 12 13 14 15 16 17 18 19 20

Key 2 2 1 4 4 2 1 1 1 3

Q.No. 21 22 23 24 25 26 27 28 29 30

Key 2 2 2 3 3 1 2 4 3 3

Q.No. 31 32 33 34 35 36 37 38 39 40

Key 2 1 2 3 3 4 2 2 3 3

Q.No. 41 42 43 44 45 46 47 48 49 50

Key 2 3 4 4 1 1 2 1 1 4

Q.No. 51 52 53 54 55 56 57 58 59 60

Key 1 4 3 1 1 3 1 3 2 1

Q.No. 61 62 63 64 65 66 67 68 69 70

Key 2 3 4 3 1 2 3 1 3 1

Q.No. 71 72 73 74 75 76 77 78 79 80

Key 2 3 2 3 4 2 2 2 1 2

Page 31: 8 & 9_MPC_SHP_E1 & E2& E4 & E5

CLASS-VIII_SUBGROUP_E4 & E5 SIMPLE HOLIDAY PACKAGE

NARAYANA GROUP OF SCHOOLS Page 31 of 40

CHEMISTRY (Q.No.1 TO 80)

1. What is the formula charge of carbon atom in carbonate ( 2

3CO ).

1) 0 2) 1 3) 2 4) 3

2. What is the formula charge of sulphur atom in per-oxosulphate (2

5SO ).

1) 0 2) 1 3) 2 4) 3 3. Lewis structure of the elements M and Z are shown below.

M Z

Compound formed is

1) 5 3M Z 2) 2 3M Z 3) MZ 4) 3MZ

4. Out of the following, maximum covalent nature is in 1) NaF 2) MgO 3) AIN 4) SiC

5. Covalent nature of NaF, 2Na Oand 3Na N in the increasing order is

1) 3 2Na N Na O NaF 2) 2 3NaF Na O Na N

3) 2 3Na O NaF Na N 4) 2 3Na O Na N NaF

6. Solubility of CuS (I), ZnS (II) and 2Na S (III) in water is in the order

1) I<II<III 2) III<I<II 3) II<I<III 4) III<II<I 7. Which pair is not in the correct order of lattice energy? 1) KCI MgO 2) AIN MgO

3) 3 3BeCO MgCO 4) 3 3BeCO MgCO

8. Out of the following ions, which pair will make the compound

most covalent? 2 3 4 2 3 4Na ,Mg ,Al ,Si ,F ,O ,N ,C

1) Na ,F 2) 2 2Mg ,O 3) 3 3Al ,N 4) 4 4Si ,C

9. Which of the following can be calculated based on Born-Haber

cycle of formation of a lattice A B (S) from A(S) and B(g)?

1) Lattice energy of A B (S) 2) Electron affinity of B (g)

3) Ionization energy of A (s) 4) All of these 10. Maximum convalency is equal to the number 1) Paired p-electrons 2) Unpaired s-electrons 3) Unpaired s and p-electrons 4) s-and p-electrons in the valence shells 11. Which of the following has incomplete octet?

1) 3NH 2) 3BCl 3) 4CCl 4) 3PCl

12. Covalency of carbon in CO is three because 1) An unexcited carbon atom has two unpaired electrons 2) The C-atom can be an acceptor of an electron pair 3) The C-atom has four valence electrons 4) The maximum covalency of carbon is three 13. Expansion of octet cannot take place in 1) N 2) S 3) Si 4) P 14. Octer rule is not followed in

1) 4 2 4 2 5CCl ,N O ,N O 2) 3 2 2BF ,BeCl ,NO

3) 2NaCl,MgCl ,MgO 4) 3 3 2PCl ,NH ,H O

Page 32: 8 & 9_MPC_SHP_E1 & E2& E4 & E5

CLASS-VIII_SUBGROUP_E4 & E5 SIMPLE HOLIDAY PACKAGE

NARAYANA GROUP OF SCHOOLS Page 32 of 40

15. Which property could describe a covalent compound? 1) It is conducting in molten (fused) state 2) It is a gas at room temperature 3) It is composed of metal and non-metal 4) It is a solid with high melting point

16. Electron dot structure of 2SO is given below

O S O

Formal charges on each atom are

1) O left S O right

1 0 1 2)

O left S O right

1 0 0

3) O left S O right

1 0 1 4)

O left S O right

0 0 0

17. Out of the following two structures of 2N O , which is more

accurate?

N N O N N O

i ii 1) I 2) II 3) Both equally 4) None of the above 18. Which one of the following has the largest lattice energy? 1) LiF 2) NaF 3) CaF2 4) AlF3 19. Arrange the following compounds in order of increasing lattice

energy: NaF, CaO, CsI 1) CsI < NaF < CaO 2) CaO < CsI < NaF 3) CsI < CaO <NaF 4) CaO < NaF < CsI 20. What is the lattice energy of AgCl using the following data:

Ag(s)Ag(g);ΔH=+284kJ/mol Ag(g)Ag+(g)+e-;ΔH=+731kJ/mol Cl2(g)2Cl(g);ΔH=+244kJ/mol Cl(g)+e-Cl-(g);ΔH=-349kJ/mol

Ag(s) + 1/2 Cl2(g) AgCl(s); ΔH = -127 kJ/mol 1) 91.5 kJ/mol 2) 915 kJ/mol 3) -91.5 kJ/mol 4) -915 kJ/mol

21. Potassium permanganate, KMnO4, at one time was used for an

anti-fungal agent. You could always tell someone who had just

been treated because their feet were purple. The pharmacy gives

you 250 grams of the stuff in a bottle. How many moles of it do

you have?

(At.wt of K=39.10, At.wt of Mn = 54.94 , At.wt of O = 16.00 )

1)158.04mol 2)15.8mol 3)0.158mol 4)1.58mol

Page 33: 8 & 9_MPC_SHP_E1 & E2& E4 & E5

CLASS-VIII_SUBGROUP_E4 & E5 SIMPLE HOLIDAY PACKAGE

NARAYANA GROUP OF SCHOOLS Page 33 of 40

22. Two flasks A and B of equal capacity contain 10g of oxygen and

10g of ozone (O3) respectively. Which will have more number of

molecules?

1)Flask A will be more than those in flask B

2)Flask B will be more than those in flask A

3)Both Flask A and flask B are same

4)None

23. If grams atomic weight of Ag is 108 grams then what is weight of

one Ag atom?

1)6.023x10-23 grams 2)17.93x10-23grams

3)17.93x1023 grams 4)6.023x1023 grams

24. Which of the following represents the given mode of hybridization

2 2 Sp Sp Sp Sp from left to right?

1) 2CH C CH CH 2) 3HC C CH CH

3) 2 2CH C C CH 4) 2 2H C CH

25. Which of the following is incorrect regarding covalent character

1) 2 4GeCl <GeCl 2)NaCl>NaI 3) 2 4PbCl <PbCl 4)NaCl<CuCl

26. A molecule 2XY contains two sigma bonds, Two Pi bonds and one

lone pair of electrons in the valence shell of X, the shape of molecule is ?

1)Linear 2)Angular 3)Tetrahedral 4)Pyramidal

27. The ratio of hybrid and un hybridized orbitals involved with

bonding of a

H

HH

HH

H (benzene )molecule is

1)3:2 2)1:1 3)3:1 4)1:3

Page 34: 8 & 9_MPC_SHP_E1 & E2& E4 & E5

CLASS-VIII_SUBGROUP_E4 & E5 SIMPLE HOLIDAY PACKAGE

NARAYANA GROUP OF SCHOOLS Page 34 of 40

28. The oxidation number of manganese in sodium manganate is

1) +7 2) +6

3) +4 4) +2

29. The oxidation states of s-atoms in Caro’s acid 2 5( )H SO and Marshell’s

acid 2 2 8( )H S O are

1) +6, +6 2) +6, +4

3) +6, -6 4) +4, +6

30. Which of the following is Gaseous reducing Agent?

1) 3HNO 2)

2H

3) Carbon 4) 2SnCl

31. Chlorine has two different Oxidation numbers in

1) 2 6Cl O 2)

2Cl O

3) 2CaOCl 4)

2 7Cl O

32. Phosphorous has the Oxidation state of +1 in

1) Ortho phosphoric acid 2) phosphorous acid

3) Hypo phosphorous acid 4) Meta phosphoric acid

33. The Oxidation state of the most electron affinity atom in each of the

product of the following reaction is 33 2HOCl HCl HClO

1) 1, 3 2) 5, 1

3) 1, 5 4) 1, 1

34. In the reaction of chlorine with dry slaked lime, the oxidation number

of chlorine changes,

1) from zero to -1 2) from zero to +1

3) from zero to zero 4) Both 1and 2

35. The oxidation states of the most electronegative element in the

products of reaction between 2BaO & dilute 2 4H SO are

1) 0 and -1 2) –1 and -2

3) -2 and -2 4) -2 and 0

36. 2 3 23 6 5 3Cl NaOH NaCl NaClO H O , In this reaction chlorine gets

1) Oxidized 2) Reduced

3) Both 1&2 4) reaction doesn’t takes place

37. The oxidation number of metals in sodium Amalgam are

1) +1, -1 2) +1, +1

3) -1, 0 4) 0, 0

38. The oxidation states of sulphur in the anions 2 2 2

3 2 4 2 6, SO S O and S O follow

the order

1) 2 2 2

2 4 3 2 6

S O SO S O 2) 2 2 2

3 2 4 2 6

SO S O S O

3) 2 2 2

2 4 2 6 3

S O S O SO 4) 2 2 2

2 6 2 4 3

S O S O SO

39. In the following reactions, which one does not involve either oxidation

or reduction.

1) 2 2 3

VO V O 2) 2

2 2 3VO V O

3) 2 2

4 2 7CrO Cr O

4) 2Zn Zn

Page 35: 8 & 9_MPC_SHP_E1 & E2& E4 & E5

CLASS-VIII_SUBGROUP_E4 & E5 SIMPLE HOLIDAY PACKAGE

NARAYANA GROUP OF SCHOOLS Page 35 of 40

40. Possible oxidation states of sulphur in its compounds are

1) +6 2) +4

3) 0 4) All of these

41. Peroxy linkage can be observed in

i) 5CrO ii) 2 2 8H S O

iii) 2BaO iv) 3CrO

1) Only i, ii and iv 2) Only i, iv

3) Only i, iii and iv 4) Only i, ii and iii

42. What is the oxidation number of middle Bromine atom in 3 8BrO

1) +6 2) +4

3) +7 4) -1

43. One mole of Hydrazine loses 10 moles of electrons, If all the nitrogen

content is present in the product. What is the oxidation number of

Nitrogen in the product (there is no change in the oxidation number of

the hydrogen.)

1) -3 2) -5

3) +3 4) +5

44. Which of the following are reducing agents

1) 2SnCl 2)

2H S

3) 3NH 4) All of these

45. Which of the following are oxidizing agents

i) 4PbO ii) 2MnO

iii)

2HNO

iv) 3KClO

1) Only i, ii and iii 2) Only ii and iii

3) Only i, ii and iv 4) Only i, iii and iv

46. A sample of municipal water contains one part of urea [m.wt-60] per

million parts of water by weight. the number of urea molecules in a

drop of water of volume 0.05 ml is

1) 142.5 10 2) 145 10

3) 135 10 4) 155 10

47. Analysis of chlorophyll shows that it contains 2.68%Mg.number of

magnesium atoms present in 2.4 grams of chlorophyll is

1) 212.68 6 10 2) 232.68 6 10

3) 202.68 6 10 4) 222.68 6 10

48. Following types of overlapping can take place in the formation of bonds.

Strength of bonds formed by these types of overlapping is 1)i<iii<ii<iv 2)i<ii<iii<iv 3)iv<iii<ii<i 4)iv<i=iii<ii 49. Select the correct statement about valence bond approach. 1)Each bond is formed by maximum overlap for its maximum stability 2)It represents localized electron model of bonding 3)Most of the electrons retain the same orbital locations as in separated

atoms 4)All of the above are correct statements

Page 36: 8 & 9_MPC_SHP_E1 & E2& E4 & E5

CLASS-VIII_SUBGROUP_E4 & E5 SIMPLE HOLIDAY PACKAGE

NARAYANA GROUP OF SCHOOLS Page 36 of 40

50. 4 3 2CH ,NH andH Oare isoelectronic but not isostructured. Taking A as

the a central atom, X as other bonding atoms and E the lone pairs, then they are represented as

1) 4 3 2

4 3 2

CH NH H O

AX AX AX 2)

4 3 2

4 3 2 2

CH NH H O

AX AX E AX E

3) 4 3 2

4 3 2

CH NH H O

AX AX E AX 4)

4 3 2

4 3 2

CH NH H O

AX AX E AX E

51. Which of the following sequences shows the correct bond angle order for

isoelectronic species 3 2O ,NO and NOF ?

1)2 3NO NOF O 2)

3 2O NO NOF

3)2 3NOF NO O 4)Cannot be predicted

52. Molecular shape of 4 4 4CF ,SF and XeF are

1)The same, with 2, 0 and 1 lone pair of electrons respectively

2) The same with 1,1 and 1 lone pair of electrons

3) Different, with 0, 1 and 2 lone pair of electrons

4) different, with 1, 0 and 2 lone pair of electrons

53. Two of the following species have same shape 2

3 3 3 3Nl , l ,SO ,NO .These are

1)3 3Nl , l 2) 2

3 3Nl ,SO 3) 2

3 3SO ,NO 4)3 3l ,NO

54. The correct order of increasing bond angles in the following species is

1) 2 3 4ClO ClO ClO 2) 2 4 3ClO ClO ClO

3) 4 3 2ClO ClO ClO 4)Equal since each Cl atom is 3Sp -hybridised

55. Bell metal contain 80% copper. The mass of bell metal which contains 201.5 10 atoms of copper is [Cu=63.5]

1) 2mg 2) 20mg

3) 200mg 4) 2000mg

56. How many molecules of water is evolved when 387 gms of Gypsum

( 4 2CaSO .2H O ) is completely decomposed

1) 2327.09 10 molecules 2) 2367.5 10 molecules

3) 2313.5 10 molecules 4) 2312 10 molecules

57. No.of oxygen atoms present in 44gms of 2CO

1) 241.2 10 atoms 2) 236.023 10 atoms

3) 2318 10 atoms 4) 2318 10 atoms

58. Isostructural species are those which have the same shape and

hybridisation. Among the given species identify the isostructural pairs.

1) [NF3 and BF3] 2) [ 4BF and 4NH

]

3) [BCl3 and BrCl3] 4) [NH3 and 3NO ]

59. The types of hybrid orbitals of nitrogen in 2 3 4NO ,NO and NH respectively

are expected to be

1) 3 2sp,sp andsp 2) 2 3sp,sp andsp

3) 2 3sp ,spandsp

4) 2 3sp ,sp andsp

Page 37: 8 & 9_MPC_SHP_E1 & E2& E4 & E5

CLASS-VIII_SUBGROUP_E4 & E5 SIMPLE HOLIDAY PACKAGE

NARAYANA GROUP OF SCHOOLS Page 37 of 40

60. Out of the following

2

3 3 3 3NO ,CO ,ClO andSO .

Isostructural and isoelectronic species are

1) 2

3 3NO ,CO 2)

3 3SO ,NO

3) 2

3 3C O ,COl 4) 2

3 3CO ,SO

61. which of the following statement regarding valence bond theory (VBT) is

not true?

1) a molecule is considered to be a

collection of atoms, and then

interactions between different

atoms is considered

2) For a molecule to be stable, the

electrostatic attractions must

predominate over the repulsion.

3) The potential energy of a diatomic

molecule is less than the sum of

potential energies of free atoms

4) The net force of alteration acting on

the atoms in a molecule is not zero.

62. In which of the following species, the central atom has the type of

hybridization which is not the same as that present in the other three?

1) 5PCl 2)

4SF

3) 3I 4)

6SbCl

63. 3BF is electron deficient (Lewis acid) and 3NH is electron-rich (Lewis acid).

In 1:1 complex of 3 3BF andNH , then coordination geometry and

hybridization of N and B atoms are:

1) N = tetrahedral, sp3

B = tetrahedral, sp3

2) N = pyramidal, sp3

B = pyramidal, sp3

3) N = pyramidal, sp3

B = planar, sp2

4) N = pyramidal, sp3

B = tetrahedral, sp3

64. The ratio of pure orbitals to hybrid orbitals in ethane (C2H6) molecule is

1) 1:2 2) 2:3

3) 4:3 4) 3:4

65. The hybridization of carbon in HCN molecule is

1) Sp 2) Sp3

3) Sp2

4) Sp3d2

Page 38: 8 & 9_MPC_SHP_E1 & E2& E4 & E5

CLASS-VIII_SUBGROUP_E4 & E5 SIMPLE HOLIDAY PACKAGE

NARAYANA GROUP OF SCHOOLS Page 38 of 40

66. The shape of BeCl2 molecule is

1) Trigonal planar 2) Linear

3) Tetrahedral 4) Pyramidal

67. The number of hybrid orbitals present in CH3 – C ≡ CH

1) 8 2) 6

3) 4 4) 2

68. Sp2 hybridization shown by

1) BeCl2 2) BF3

3) NH3 4) XeF3

69. Which of the following statements are true?

I) Hybridization of 4NH is Sp3

II) The concept of hybridization was introduced by Pauling

III) The shape of 2CO molecule is Linear.

IV)The bond angle between Sp3 hybrid orbitals is 1090.28′

1) Only I and II 2) Only I,II and III

3) Only II and III 4) I,II,III and IV

70. Which of the following pairs are correctly matched?

1) CO2 – SP2 2) C2H4 – Sp

3) H3O+ - Sp3 4) NH3- Sp2

71. The mode of hybridisation of central carbon in C3O2 is

1) sp 2) sp3

3) sp2 4) 3sp d

72. The type of hybrid orbitals used by the oxygen atom in Cl2O molecule is

1) sp3 2) sp2

3) sp 4) 3 2sp d

73. 38 grams of Fluorine contains (atomic weight of fluorine is 19)

1) 4 gram atoms of fluorine 2) 1 moles of fluorine

3) 12 x 1023 fluorine molecules 4) all the above

Page 39: 8 & 9_MPC_SHP_E1 & E2& E4 & E5

CLASS-VIII_SUBGROUP_E4 & E5 SIMPLE HOLIDAY PACKAGE

NARAYANA GROUP OF SCHOOLS Page 39 of 40

74. The no. of gram atoms present in 20 gms of Ca CO3 are

1) 5 2) 0.5

3) 1.0 4) 0.02

75. The mass of one oxygen molecule is

1) 2.656 × 10-23 2) 5.312 × 10-23 gm

3) 1.66 × 1024 gm 4) 32 gm

76. The weight of 8gm. atoms of nitrogen is

1) 56 gms. 2) 112 gms.

3) 42 gms. 4) 28 gms.

77. Volume occupied by 44 g of 2CO is

1) 2240 lit at STP 2) 2.24 lit at STP

3) 22.4 lit at STP 4) 224 lit at STP

78. Which of the following is correct?

1) Molecular weight of oxygen is 48. 2) Gram molecular mass of sulphur

(S8) is 256 g.

3) The weight of one molecule of O3 is

48 amu.

4) Both 2 and 3

79. The number of oxygen atoms in 88g CO2 are

1) 2.40 x 1023 2) 24.08 x 1024

3) 24.08 x 1023 4) 24.08 x 1022

80. The number of moles of a gas in 1 m3 of volume at NTP is

1) 0.446 2) 4.46

3) 1.464 4) 44.6

Page 40: 8 & 9_MPC_SHP_E1 & E2& E4 & E5

CLASS-VIII_SUBGROUP_E4 & E5 SIMPLE HOLIDAY PACKAGE

NARAYANA GROUP OF SCHOOLS Page 40 of 40

VIII_CHEMISTRY_SHP_E4 & E5_KEY

Q.No. 1 2 3 4 5 6 7 8 9 10

Key 1 1 3 4 2 1 4 4 4 4

Q.No. 11 12 13 14 15 16 17 18 19 20

Key 2 2 1 2 2 1 1 4 1 4

Q.No. 21 22 23 24 25 26 27 28 29 30

Key 4 1 2 1 2 2 1 2 1 2

Q.No. 31 32 33 34 35 36 37 38 39 40

Key 3 3 3 4 2 3 4 1 3 4

Q.No. 41 42 43 44 45 46 47 48 49 50

Key 4 2 3 4 3 2 3 3 4 2

Q.No. 51 52 53 54 55 56 57 58 59 60

Key 2 3 2 1 2 1 1 2 2 1

Q.No. 61 62 63 64 65 66 67 68 69 70

Key 4 4 1 4 1 2 1 2 4 3

Q.No. 71 72 73 74 75 76 77 78 79 80

Key 1 1 2 3 2 2 3 4 3 4

Page 41: 8 & 9_MPC_SHP_E1 & E2& E4 & E5

CLASS-VIII_SUBGROUP_E1 TO E3 SIMPLE HOLIDAY PACKAGE

NARAYANA GROUP OF SCHOOLS Page 1 of 34

(OLYMPIAD AND E-TECHNO)

SIMPLE HOLIDAY PACKAGE PONGAL

Mathematics

Physics

Chemistry

CLASS – VIII E1– E3

VIII_MPC_E1 to E3_SHP

INDEX

SUBJECT PAGE NO.

1. MATHEMATHICS 2 - 12

2. PHYSICS 13 - 26

3. CHEMISTRY 27 - 34

Page 42: 8 & 9_MPC_SHP_E1 & E2& E4 & E5

CLASS-VIII_SUBGROUP_E1 TO E3 SIMPLE HOLIDAY PACKAGE

NARAYANA GROUP OF SCHOOLS Page 2 of 34

MATHEMATHICS ( Q.NO.1 TO 80)

1. If AB || CF and BC || ED, then which of the following is not

correct ?

1) ABC = CDE 2) FDE = FDG

3) ABC = FDG 4) all of these

2. If (6x + 20o), (9y + 30o) and (3z + 40o) form a complete angle, then

which of the following relationship is correct between x, y and z

are

1) 3x + 2y + 3z = 90° 2) 2x + 3y + z = 90°

3) x + 2y + 3z = 90° 4) x + y + 3z = 90°

3. ABC is an isosceles triangle with AB = AC and C = 40º. BD is the

bisector of ABC intersects AC at D and AE is perpendicular to BD, then

EAD is

1) 20º 2) 25º 3) 30º 4) 35º

4. The side of the largest possible regular octagon that can be cut out of a

square of side 1cm is

1) 2 1cm 2)

1cm

2 2 3)

1

cm2 1

4) None

5. Triangles ABC and ABD are isosceles with AB = AC = BD, and BD

intersects AC at E. If BD AC , then C D is

1) 115° 2) 120° 3) 130° 4) 135°

Page 43: 8 & 9_MPC_SHP_E1 & E2& E4 & E5

CLASS-VIII_SUBGROUP_E1 TO E3 SIMPLE HOLIDAY PACKAGE

NARAYANA GROUP OF SCHOOLS Page 3 of 34

6. In the below figure AB QS CD, PAB = 600 and DCR = 600, then

QXY + QYX =

1) 900 2) 600 3) 800 4) 1000

7. A regular pentagon is a five sided figure which has all of its angles

equal and all of its side lengths equal. In the diagram, TREND is a

regular pentagon, PEA is an equilateral triangle, an OPEN is a

square. Then PER is

1) 1620 2) 460 3) 280 4) 390

8. The value of x0 that will make PQ parallel to RS is

1) 170 2) 500 3) 130 4) 270

9. The interior angles of a pentagon are in the ratio 2 : 3 : 4 : 5 : 6

respectively. Then sum of the first and third angles is

1) 900 2) 1400 3) 1350 4) 1620

10. Exterior angle of a regular polygon measures 10 degrees where

is a natural number. Then

1) There is no such 2) There are infinitely many such

3) There are precisely nine such 4) There are precisely seven such

Page 44: 8 & 9_MPC_SHP_E1 & E2& E4 & E5

CLASS-VIII_SUBGROUP_E1 TO E3 SIMPLE HOLIDAY PACKAGE

NARAYANA GROUP OF SCHOOLS Page 4 of 34

11. Four angles of a nonagon are 50°, 60°, 70°, 40° respectively. And

all the other angles are equal. Then the measure of each one of the

equal angles is

1) 220° 2) 208° 3) 240° 4) 260°

12. Which of the following statement is wrong?

1) The measure of each exterior angle of a regular polygon0360

n

2) The ratio of the sides of a polygon to the diagonals of a polygon is

3: n 3

3) The ratio of the interior angle of a regular polygon to its exterior

angle is n 2 : 2

4) The sum of all the interior angles of any polygon 02n 4 90

13. The measure of each interior angle of a regular polygon of 6 sides.

1) 110º 2) 140 º 3) 120º 4) 108º

14. One of the interior angle of a Regular Polygon is 1200, then the

number sides of regular polygon are

1) 5 2) 4 3) 7 4)6

15. In the above figure, PQS SRQ, QPR SQR U is a point on QR

such that 0QPU = 30 . thenQUP is

1) 400 2) 500 3) 600 4) 700

16. In the given figure, which of the following is false?

1) x = 109° 2) y = 34° 3) x = 71° 4) none

Page 45: 8 & 9_MPC_SHP_E1 & E2& E4 & E5

CLASS-VIII_SUBGROUP_E1 TO E3 SIMPLE HOLIDAY PACKAGE

NARAYANA GROUP OF SCHOOLS Page 5 of 34

17. In a 0, 90 ,ABC A AL is drawn perpendicular to BC. Then BAL is

equal to :

1) ALC 2) ACB 3) BAC 4) B BAL

18. OB and OC are respectively the bisectors of ABC and .ACB Then

BOC is equal to :

1) 0 190

2A 2) 090 A 3) 0 1

902

A 4) 0 1180

2A

19. In figure, sides AB and AC of a ABC are produced to P and Q

respectively. The bisectors of PBC and QCB intersect at O . Then

BOC is equal to:

1) 0 190

2BAC 2) 1

2PBC QCB

3) 0 190

2BAC 4)none of these

20. Statement - I :In the triangle ABC, BC = CD and (ABC –BAC) =

30°.The measure of ABD is 15°

Statement - II :If the equal sides of an isosceles triangle are

produced, the exterior angles are equal.

1) Both statements - I and statement - II are true

2) Both statements - I and statement - II are false

3) Statement - I is true and statement - II is false

4) Statement - I is false and statement - II is true

Page 46: 8 & 9_MPC_SHP_E1 & E2& E4 & E5

CLASS-VIII_SUBGROUP_E1 TO E3 SIMPLE HOLIDAY PACKAGE

NARAYANA GROUP OF SCHOOLS Page 6 of 34

21. In the given figure AB =BC and AC =CD , IfBAD = yand ADB = x ,

theny =

1) x 2) 2x 3) 3x 4) None

22. If each interior angle of a regular polygon is 140°, then the number of

diagonals are

1) 135 2) 27

3) 60 4) 54

23. Exterior angle of a regular polygon measures 6 degrees where is a

natural number. Then

1) There is no such 2) There are infinitely many such

3) There are precisely ten such 4) none

24. Let ABCD be a rectangle with AB = a and BC = b. Suppose r1 is the radius

of the circle passing through A and B and touching CD, and similarly r2

is the radius of the circle passing through B and C and touching AD.

Then r1 + r

2 is

1) 5a+b

8

2) 5(a b)

8

3) 5(a b)

8

4) none

25. In ADE, ADE = 140°and the points B, C lie on sides AD,

AE respectively and points A, B, C, D, E are distinct. If

lengths AB, BC, CD and DE are all equal, then the

measure of EAD is

1) 5° 2) 6°

3) 7.5° 4) 10°

26. In the figure || ||AD EF BC if EB=2AE and DF=2.5cm then the length of

FC is

1) 5cm 2) 2.5cm

3) 0.5cm 4) 10cm.

27. A circular park of radius 20 m is situated in a colony. Three boys.

Ankur, Syed and David are sitting at equal distance on its boundary

each having a toy telephone in his hands to talk to each other. Then

the length of the string of each phone is

1) 20 3 m 2) 10 3 m

3) 10m 4) 20m

Page 47: 8 & 9_MPC_SHP_E1 & E2& E4 & E5

CLASS-VIII_SUBGROUP_E1 TO E3 SIMPLE HOLIDAY PACKAGE

NARAYANA GROUP OF SCHOOLS Page 7 of 34

30. The roots of 242 x x 0 are

1) 2,–6 2) –2,6 3) 2,6 4) 6,–7

31. If 1 x 1 x3 3 10 then the values of x are

1) 1,-1 2) 1,0 3) 1,2 4) -1,-2

32. The nature of the roots of the equation 2x 5x 5 0 are

1) Real and rational 2) Real and irrational

3) Real and equal 4) none

33. If 215 28 x x then x

1) 7/5 2) -4/3 3) both(1)&(2) 4) 2/3

34. In a quadratic equation 2 0ax bx c if ‘a’ and ‘c’ are of opposite

signs and ‘b’ is real, then roots of the equation are

1) real and distinct 2) real and equal

3) Imaginary 4) both roots positive

35. Solve the following quadratic equation :

If 2 2 23 8 4 0, 0. a x abx b a Then x=

1) -2b/a 2) 2b/a 3) 2b/3a 4) b/3a

36. Solve for x : 2 2 29 9 2 5 2 0x a b x a ab b

1) 2

3

a b 2)

2

3

a b

3) 2

3

a b 4) both (1)&(2)

28. In the given figure, two circles with centres A and B and of radii 5cm

and 3 cm touch each other internally. If the perpendicular bisector of

segment AB meets the bigger circle in P and Q. Then the length of PQ is

1) 2cm 2) 4 6 cm

3) 2 2 cm 4) 4 2 cm

29. Two chords AB and CD of lengths 5 cm and 11 cm respectively of a

circle are parallel to each other and are opposite side of its centre. If

the distance between AB and CD is 6 cm, then the radius of the circle is

1) 3 3 cm. 2) 5 cm

3) 5 5

2cm

4) 5 5 cm.

Page 48: 8 & 9_MPC_SHP_E1 & E2& E4 & E5

CLASS-VIII_SUBGROUP_E1 TO E3 SIMPLE HOLIDAY PACKAGE

NARAYANA GROUP OF SCHOOLS Page 8 of 34

37. If x 5 5 5 ........... , then the value of x is

1) 1 21

2

2) -1 3) 0 4)

1 21

2

38. Statement I: If 27x 6x 13 7 0 , then 13 7

x , 77

Statement II: If 2ax bx c 0; a,b,c R, a 0, then

2b b 4ac

x2a

1) Both Statements are true

2) Both Statements are false

3) Statement I is true, Statement II is false.

4) Statement I is false, Statement II is true.

39. Sum of roots of the equation 22x 4x 3 0 is

1) 32

2) –2 3) 32

4) 2

40. The quadratic equation whose roots are 2,1 is

1) 2x 2x 1 0 2) 2x 3x 2 0

3) 2x 3x 2 0 4) 2x 3x 2 0

41. Quadratic equation whose roots are 2, 3 2 is

1) 2x 2 3 2 x 6 2 0 2) 2x 2 3 2 x 6 2 0

3) 2x 2 3 2 x 6 2 0 4) 2x 2 3 2 x 6 2 0

42. If , are roots of 2x 3x 5 0 , then 2 2 =

1) 5 2) 9 3) 25 4) 30

43. If , are the roots 2 0ax bx c of then 2 2

1 1

1) 2

2

2b ac

a 2)

bc

3) 2

2

2b ac

c 4)

2 2b ac

ac

44. If , are roots of 2 1 0x k x c then 1 1

1) C 2) 1 3) 1 – c+k 4) 1 - c

45. If 4 , 2 then 3 3

1) 8 2) 2 3) 12 4) none

46. The value of k, if the equation 2 22 1 0x k x k has equal roots is

1) -1/2 2) -3/4 3) 3/4 4) 2/3

47. If (3 + i ) is a root of the equation 2 0x ax b then a is

1) 3 2) -3 3) 6 4) -6

48. If are the roots of 2x x 1 0,then

1)

1 2)

2 2

1 11 3)

1 4) none

Page 49: 8 & 9_MPC_SHP_E1 & E2& E4 & E5

CLASS-VIII_SUBGROUP_E1 TO E3 SIMPLE HOLIDAY PACKAGE

NARAYANA GROUP OF SCHOOLS Page 9 of 34

49. If , are the roots of the equation 2ax bx c 0 then

b c;a a

. Then 2 2 is

1)

2b

a 2)

2b 2c

a a 3)

2b c

a a 4)

2

2

b ac

a

50. If 1 x 1 x5 5 26 , then the sum of the roots is

1) 0 2) 1 3) 2 4) 4

51. For what values of k, does the following quadratic equation have

equal roots ?

24 1 1 0k x k x

1) 2 2) -5 3) 3 4) 5

52. In the below triangle ABC, B = 700, C = 500 and AD is the bisector of

A. Then which of the following is false?

1) AB > AD 2) AD > CD 3) AB < AD 4) both (1) and (2)

53. In the given figure, AB = CD, BAC = ACD = 10°, CAD = 85°, then

ABC is

1) 50° 2) 65° 3) 75° 4) 85°

54. In ABC ‘O’ is the orthocentre and BOC=2 A . Then BOC is

1) 2) 600 3) 900 4) 1000

A

B C

D

0120

Page 50: 8 & 9_MPC_SHP_E1 & E2& E4 & E5

CLASS-VIII_SUBGROUP_E1 TO E3 SIMPLE HOLIDAY PACKAGE

NARAYANA GROUP OF SCHOOLS Page 10 of 34

55. In the given figure CAB = 1

ACD2 , BAD = 85° and

ADC = 40°, then ACB =

1) 45° 2) 55° 3) 70° 4) 60°

56. In ABC,DEF are midpoint of sides AB, BC and CA respectively.

If Ar 11gm DECF 34 sq. cm then Ar DEB

1) 9 sq cm 2) 17 sq cm 3) 8.5 sq cm 4) 36 sq cm

57. Statement I: If two medians of a triangle are equal, then the triangle is

scalene.

Statement II: If a transversal intersects by two parallel lines then

vertically opposite angles are equal.

1) Both statements I and II are true

2) Both statements I and II are false

3) Statement – I is true but statement – II is false

4) Statement– I is false but statement – II is true

58. D is the midpoint of the side BC of a ABC and E is the mid-point of BD.

If O be the mid point of AE, then area of a BOE is

1) 2)

3) 4)

59. In a ABC , the bisectors of B and C intersect each other at a point

O. Then BOC =.

1) 0 190 A

2 2) 0 1

90 A2

3) 090 A 4) 090 A

60. ABCD is a quadrilateral in which0 0 080 , 110 , 80 A B C and D

1) 900 2) 1700

3) 1350 4) 1300

61. ABCD is a quadrilateral in which A= 2x+200, B = 5x+80,

C = 3x+100, D = x+30, then the value of x is

1) 470 2) 290

3) 430 4) 270

A

B DC

)ABC(ar4

1 )ABC(ar

8

1

)ABC(ar16

1 )ABC(ar

32

1

Page 51: 8 & 9_MPC_SHP_E1 & E2& E4 & E5

CLASS-VIII_SUBGROUP_E1 TO E3 SIMPLE HOLIDAY PACKAGE

NARAYANA GROUP OF SCHOOLS Page 11 of 34

62. ABCD is a trapezium. AB//CD, E & F are midpoints on AD and BC. If AB

= 20cm,CD = 6cm, then EF is

1) 12cm 2) 13cm

3) 8cm 4) 24cm

63. In a quadrilateral, three angles are 0 0 0100 ,65 ,55 then the fourth angle is

1) 1400 2) 1350

3) 1200 4) 1500

64. ABCD is a parallelogram, AB = 6cm, BC = 4cm, then the value of

is

1) 92 2) 104

3) 72 4) none

65. In a parallelogram ABCD, A = x + 13, B = 2x + 17, then D is

1) 1170 2) 730

3) 340 4) 630

66. In a rectangle ABCD. If AB = 7cm, BC = 24cm, then AC is

1) 9cm 2) 5cm

3) 3cm 4) 25cm

67. ABCD is a rectangle. O is the interior point of a rectangle. If OA = 9cm,

OC = 40cm, then 2 2OB OD is

1) 1681 2) 1460

3) 1545 4) none

68. ABCD is a rhombus AC and BD are diagonals. If AC = 40cm, BD = 30cm,

then the area of the rhombus is

1) 350sqcm 2) 600sqcm

3) 250sqcm 4) 300sqcm

69. In a parallelogram ABCD 02 40 A x and 0120C x then x in degrees

1) 600 2) 900

3) 800 4) 1500

70. In a Rhombus ABCD the diagonal AC bisects ,A C and the diagonal BD

bisects

1) ,A B 2) ,C D

3) ,B D 4) ,C A

71. In a parallelogram ABCD if 0115A then which of the following is true?

1) 0 0 065 , 115 , 65B C D 2) 0 0 0115 , 65 , 65B C D

3) 0 0115 , 65C B D 4) Both (1)&(3)

72. Statement-I: In a parallelogram the sum of any two adjacent angles is

equal to 1800

Statement-II: In a parallelogram ABCD, if 0180A B then0180A D , 0180C D , 0180B C

1) Both statements I and II are true 2) Both statements I and II are false

3) Statement – I is true but

statement – II is false

4) Statement – I is false but statement – II

is true

2 2AC BD

Page 52: 8 & 9_MPC_SHP_E1 & E2& E4 & E5

CLASS-VIII_SUBGROUP_E1 TO E3 SIMPLE HOLIDAY PACKAGE

NARAYANA GROUP OF SCHOOLS Page 12 of 34

73. In the given figure ABCD is a rectangle whose diagonals AC and BD

intersect at ‘O’ if 0OAB 32 . Then the value of ACB =

1) 058 2) 090

3) 060 4) 030

74. The lengths of the diagonals of a rhombus are AC=28 cm and BD=18 cm respectively. Then area of the rhombus is

1) 216 cm2 2) 316 cm2 3) 416 cm2 4)252 cm2

75. ABCD is a rhombus with 056 ,ABC then ACD is equal to

1)

090 2) 060 3)

056 4)062

76. The measure of each angle of a parallelogram, if one of its angles is 030 less than twice the smallest angle is

1) 0 0 0 060 ,100 ,90 ,20 2)

0 0 0 080 ,40 ,120 ,90

3) 0 0 0 0100 ,90 ,90 ,80 4)

0 0 0 070 ,110 ,70 ,110

77. If ABCD is a quadrilateral in which diagonals bisect at right angles and ACBD, then it is a

1) Isosceles trapezium 2) Rectangle 3) Rhombus 4) Square

78. In the given figure ABCD is a rectangle whose diagonals AC and BD

intersect at ‘O’ if 0OAB 32 .The value of OBC =

1)

090 2) 060 3)

058 4) 045

79. The side of a square ABCD is 10cm and the length of a diagonal is

1) 10 2 cm 2) 2 5 cm 3) 5 2 cm 4) none

80. In the given figure, PSR = 60º and PRQ = 120º and given that RPS

= 2RPQ. Then PQR is

1) 60º 2) 50º 3) 40º 4) 30º

P T

QRS

Page 53: 8 & 9_MPC_SHP_E1 & E2& E4 & E5

CLASS-VIII_SUBGROUP_E1 TO E3 SIMPLE HOLIDAY PACKAGE

NARAYANA GROUP OF SCHOOLS Page 13 of 34

VIII_MATHEMATICS_SHP_E1 TO E3_KEY

Q.No. 1 2 3 4 5 6 7 8 9 10

Key 4 2 3 1 4 2 1 3 4 4

Q.No. 11 12 13 14 15 16 17 18 19 20

Key 2 2 3 4 3 3 2 3 1 1

Q.No. 21 22 23 24 25 26 27 28 29 30

Key 3 2 3 1 4 1 1 2 2 4

Q.No. 31 32 33 34 35 36 37 38 39 40

Key 1 2 3 1 1 4 4 4 4 3

Q.No. 41 42 43 44 45 46 47 48 49 50

Key 4 4 3 3 4 1 4 3 2 1

Q.No. 51 52 53 54 55 56 57 58 59 60

Key 4 3 4 1 3 2 4 2 1 1

Q.No. 61 62 63 64 65 66 67 68 69 70

Key 2 2 1 2 1 4 1 2 3 3

Q.No. 71 72 73 74 75 76 77 78 79 80

Key 4 1 1 4 4 4 3 3 1 4

Page 54: 8 & 9_MPC_SHP_E1 & E2& E4 & E5

CLASS-VIII_SUBGROUP_E1 TO E3 SIMPLE HOLIDAY PACKAGE

NARAYANA GROUP OF SCHOOLS Page 14 of 34

PHYSICS ( Q.NO. 1 TO 80)

1. A piece of gold-aluminium alloy weighs 100 grams in air and 80 g when immersed in water. What is the weight of gold in the alloy ? (Sp . gravity of gold = 19.3 and of aluminum =2.5).

1) 57.44 g 2) 2.5 g 3) 20 g 4) 38.6 g

2. Density of ice is 900 kg/m3. A piece of ice is floating in water (of

density 1000 kg/m3). Find the fraction of volume of the piece of ice inside the water.

1) 0.1 2) 0.9 3) 0.2 4) 0.8

3. The density of ice is 917 kgm–3. What fraction of the volume of a piece of ice will be above water, when floating in fresh water ?

1) 0.06 2) 0.083 3) 0.038 4) 0.068

4. The volume of 50g of a substance is 20cm3. If the density of water

is 31 /g cm . The substance will

1) float 2) Sink

3) Neither float nor sink 4) None of the above

5. A boy is carrying a fish in one hand and a bucket full of water in the other hand. He then places the fish in the bucket thinking that in accordance with Archimedes’s principle he is now carrying less weight as the weight of the fish will reduce due to upthrust. Is he right?

1) yes 2) No

3) cannot say 4) data insufficient

6. A body will partially float in a liquid, if the density of solid is ____________ than the density of liquid.

1) Greater 2) Less

3) Equal or less 4) Equal or greater

7. The volume of a 500 g sealed packet is 3350cm . If the packet is

immersed in the water of density 31 /g cm . The mass of the water

displaced by this packet is

1) 350 g 2) 500 g

3) 1 g 4) None of the above

8. The density of ice 0.918 g /cc and that of sea water is 1.03 g/cc. An iceberg floats in water with a portion of volume 224 cc above the surface of water . Find the volume of the whole iceberg.

1) 3060 cc 2) 2060 cc 3) 4060 cc 4) 5060 cc

9. Density of ice is and that of water is . What will be the decrease in volume when mass ‘M’ of ice melts

1)

M

2)

M

3)

1 1M

4)

1 1 1

M

Page 55: 8 & 9_MPC_SHP_E1 & E2& E4 & E5

CLASS-VIII_SUBGROUP_E1 TO E3 SIMPLE HOLIDAY PACKAGE

NARAYANA GROUP OF SCHOOLS Page 15 of 34

10. The tension in a string holding a solid block below the surface of a liquid (or density greater than that of solid) as shown in figure is T0 when the system is rest.

What will be the tension in the string if the system has an upward

acceleration a. 1) T0 [1 – a/g] 2) T0 [1 + a/g] 3) T0 [1 – g/a] 4) T0 [1 + g/a]

11. A wooden cylinder of diameter 4r, height H and density / 3 is

kept on a hole of diameter 2r of a tank, filled with liquid of density as shown in the figure.

Now level of the liquid start decreasing slowly. When the level of liquid is at a height h1 above the cylinder the block starts moving

1) 1(A H)/g 2)

1(A g)3

3)

1(A H)g3

4)

1(A H)g2

12. A wooden cylinder of diameter 4r, height H and density / 3 is

kept on a hole of diameter 2r of a tank, filled with liquid of density as shown in the figure.

Now level of the liquid start decreasing slowly. When the level of

liquid is at a height h1 above the cylinder the block starts moving

up. Find the area of cross section (A1) of the cylinder.

1) 24 r 2) 23 r 3) 22 r 4) 2r

13. A wooden cylinder of diameter 4r, height H and density / 3 is

kept on a hole of diameter 2r of a tank, filled with liquid of density as shown in the figure.

Now level of the liquid start decreasing slowly. When the level of

liquid is at a height h1 above the cylinder the block starts moving

up. Find the area of cross section (A1) of the cylinder. Find the

area(A2) of the base of the cylinder that is exposed to air.

1) 24 r 2) 23 r 3) 22 r 4) 2r

Page 56: 8 & 9_MPC_SHP_E1 & E2& E4 & E5

CLASS-VIII_SUBGROUP_E1 TO E3 SIMPLE HOLIDAY PACKAGE

NARAYANA GROUP OF SCHOOLS Page 16 of 34

14. A lift is moving upwards with an acceleration of 1.2 m/s2, then the value of geff is:

1) 9.8 m/s2 2) 8.6 m/s2 3) 0 4) 11 m/s2

15. A block of mass 4kg and density 0.8 g/cm3 is held stationary with the help of a string. The tank is accelerating vertically upwards

with 1 m/s2. (g = 10 m/s2) then find up thrust.

1) 13.75 N 2) 27.50 N 3) 55 N 4) 30 N

16. A solid sphere of mass m and specific gravity s is held stationary relative to a tank filled with water as shown in fig. The tank is accelerating vertically upwards with a. Then tension in the thread connected between the sphere and the bottom of the tank is

if the thread snaps then acceleration of sphere with respect to

tank is (here F-net buoyant force , W-weight of the solid sphere)

1) F+W - ma 2) F - W

m 3) F+W +ma 4) F - mg

17. A hollow spherical body of inner and outer radii 6 cm and 8 cm respectively floats half-submerged in water. Find the density of the material of the sphere.

1) 3845 /kg m 2) 3765 /kg m 3) 3865 /kg m 4) 3665 /kg m

18. The fraction of floating object of volume v0 and density 8 g/cc

above the surface of the liquid of density 12 g/cc will be 1) 3 2) 2/3 3) 1/3 4) 3/2

19. A cylinder of area of cross section 4 cm2 and length 40 cm is

totally immersed in water (g = 10 m/s2) (density of solid = 1500

kg/m3, density of water = 1000 kg/m3) upthrust acting on the cylinder is

1) 36 N 2) 1.6 N 3) 16 N 4) 3.6 N

20. A block of mass 1 kg and density 0.6 g/cm3 is held stationary with the help of a string as shown. The tank is accelerating vertically up

with acceleration 1 ms–2. The tension in the string is

1) 7.3 N 2) 13.3 N 3) 16.3 N 4) 8.3 N

21. If refractive indices of glass and water are respectively 3 4 and 2 3

, then

the refractive index of water with respective to glass is

1) 2 2) 89

3) 98

4) 12

Page 57: 8 & 9_MPC_SHP_E1 & E2& E4 & E5

CLASS-VIII_SUBGROUP_E1 TO E3 SIMPLE HOLIDAY PACKAGE

NARAYANA GROUP OF SCHOOLS Page 17 of 34

22. The critical angle of a medium w.r.t. to vacuum is 300, velocity of light

in this medium in m/s is

1) 83 10 2) 82 10 3) 81.5 10 4) 80.5 10

23. The velocity of light in air is 8 13 10 ms and in glass is 8 12 10 ms .The

refractive index of air w.r.t glass is

1) 2/3 2) 3/2 3) 4/3 4) 9/4

24. A concave mirror has its radius of curvature 30 cm. The position of the

image of an object placed at a distance of 30 cm from the mirror is

1) 30 cm 2) Infinity 3) 15 cm 4) 60 cm

25. A printed page is kept pressed by a glass cube 1.5 of edge 6 cm. By

what amount will the printed letters appears to be shifted when

viewed from the top.

1) 2 cm 2) 3 cm 3) 4 cm 4) 6 cm

26. A hollow spherical body of inner and outer radii 6 cm and 8 cm

respectively floats half-submerged in water. The density of the material

of the sphere.

1) 865 kg/m3 2) 965 kg/m3 3) 1065 kg/m3 4) 765 kg/m3

27. A man uses a concave mirror for shaving . He keeps his face at a

distance of 25 cm from the mirror and gets an image which is 1.4

times enlarged. The focal length of the image is

1) 75.5 cm 2) 68.5 cm 3) 87.5 cm 4) 94 cm

28. An object moves with 5 m/s towards right while the mirror moves 2 m/s

towards the left as shown. Then the velocity of image.

1) 9 m/s towards left 2) 9m/s towards right

3) 5 m/s towards right 4) 5 m/s towards left

29. An equilateral prism of refractive index

5

3is immersed in water

of refractive index 4

3.If a ray of light passing through the prism

undergoes minimum deviation, the angle of incidence on the

first refracting face of the prism is

1) 1 5sin

8

2) 030

3) 045 4) 1 3sin

5

Page 58: 8 & 9_MPC_SHP_E1 & E2& E4 & E5

CLASS-VIII_SUBGROUP_E1 TO E3 SIMPLE HOLIDAY PACKAGE

NARAYANA GROUP OF SCHOOLS Page 18 of 34

30. For a prism PQR the incident and emergent rays are parallel.

The minimum value of refractive index of the prism material is

1) 3 2) 1)5

3) 2 4) 2

31. A glass prism of refractive index 1.5 is placed in water of

refractive index 1.33. The minimum value of the angle of the

prism so that it will not be possible to have any emergent ray is

1) 1500 2) 1250

3) 1650 4) 1800

32. The minimum deviation produced by a hollow prism filled with

a certain liquid is found to be 030 .The light ray is also found

to be refracted at angle of 030 .The refractive index of the

liquid is

1) 2 2) 3

3) 3

2

4) 3

2

33. Three glass prisms A,B and C of same refractive index are

placed in contact with each other as shown in figure , with no

air gap between the prisms. Monochromatic ray of light OP

passes through the prism assembly and emerges as QR. The

conditions of minimum deviation is satisfied in the prisms

1) A and C 2) B and C

3) A and B 4) In all prisms A,B and C

34. Angle of prism is A and its one surface is silvered . Light rays falling at an angle of incidence 2A on first surface return back through the same path after suffering reflection at second silvered surface . Refractive index of the material of prism is

1) 2sin A 2) 2cos A

3) 1cos

2A

4) tan A

Page 59: 8 & 9_MPC_SHP_E1 & E2& E4 & E5

CLASS-VIII_SUBGROUP_E1 TO E3 SIMPLE HOLIDAY PACKAGE

NARAYANA GROUP OF SCHOOLS Page 19 of 34

35. For a prism of refractive index 1.732, the angle of minimum

deviation is equal to the angle of the prism . The angle of

the prism is

1) 080 2) 070

3) 060 4) 050

36. A prism of refractive index and angle A is placed in the

minimum deviation position. If the angle of minimum

deviation is A, then the value of A in terms of is

1) 1sin2

2)

1 1sin

2

3) 12cos2

4) 1cos

2

37. At the angle of minimum deviation condition of an equilateral prism

1) incident and emergent

angles are equal

2) the angle of refraction 1 2r and r are

not equal

3) refractive ray is not parallel to base of the prism

4) deviated angle is equal to incident angle

38. ABC is right angled prism. A ray (1) is incident on the face AB along the normal Another ray (2) is incident on the face AB such that it emerges normally from the face AC. Then

The minimum value of refractive index of the material of the prism for which the ray (1) under goes total internal reflection on the face AC is

1) 1 2) 2

3) 5 4) 2

39. The angle between two given refracting surfaces is called

1) Refracting edge 2) Angle of prism

3) Principal section 4) Base of the prism

40. In the case of a prism, for normal incidence

1) 1 1 2 2 2i r 0;D i A I r 2) 2 2 1 1i r 0;D i r

3) 2 1i 90 ;A r c 4)

1 2i 90 ,A r c

Page 60: 8 & 9_MPC_SHP_E1 & E2& E4 & E5

CLASS-VIII_SUBGROUP_E1 TO E3 SIMPLE HOLIDAY PACKAGE

NARAYANA GROUP OF SCHOOLS Page 20 of 34

41. In the case of a prism, for grazing incidence

1) 1 1 2 2 2i r 0;D i A I r 2) 2 2 1 1i r 0;D i r

3) 2 1i 90 ;A r c 4)

1 2i 90 ,A r c

42. The prism is surrounded by a medium of refractive index 1 . If

the refractive index index of the material of the prism is 2 ,

then

1)

2

1

sin2

sin2

mA

A

2)

1

2

sin2

sin2

mA

A

3)

1

sin2

sin2

mA

A

4)

2

sin2

sin2

mA

A

43. The angle between the incident ray produced and emergent ray is

1) Angle of deviation 2) Angle of incidence 3) Angle of emergence 4) Angle of refraction

44. If the angle of a prism is 800 and angle of minimum deviation is

400, then the angle of refraction in the prism will be

1) 300 2) 40°

3) 50° 4) 20°

45. A thin prism of 30 angle gives a deviation of 2.40.The value of refractive index of the material of the prism is

1) 1.6 2) 1.7 3) 1.8 4) 1.9

46. The side AC of a glass prism of refractive index 1.5 is silvered. A ray of light falls on the face AB such that it retraces its path. What is the angle of incidence, if the angle of the prism is . (sin

35º= 0.574) sin–1(0.86) =59.4º

1) 059.4 2) 064.6

3) 035 4) 072

Page 61: 8 & 9_MPC_SHP_E1 & E2& E4 & E5

CLASS-VIII_SUBGROUP_E1 TO E3 SIMPLE HOLIDAY PACKAGE

NARAYANA GROUP OF SCHOOLS Page 21 of 34

47. According to the refractive index of the prism ‘A’ is

1) 1 2A r r 2) 1 2A i i

3) 1 2A r r 4) 1 2A i i

48. Angle of deviation for small angled prism

1) d 1 A 2) d 1 A

3) d 1 4) d A

49. A prism produces a minimum deviation in a light beam. If three such prisms are combined, the minimum deviation produced will be

1) 4d 2) 2d

3) d 4) 0

50. If a small angled prism, made of glass is immersed in a liquid of refractive index 1 and a ray of light is made incident on it, then

1) its deviation will be zero 2) it will suffer total reflection

3) the emergent ray is bent towards the edge of the

prism

4) the emergent ray is bent towards the base of prism

51. For total internal reflection to take place, the angle of incidence i and the refractive index of the medium must

satisfy; the inequality

1) 1

sin i

2) 1

sin i

3) sin i < 4) sin i >

52. The path of a refracted ray of light in a prism is parallel to the base of the prism only when the

1) light is of a particular wavelength

2) ray is incident normally at one face

3) ray undergoes minimum

deviation

4) prism is made of a particular type

of glass 53. A small object is placed 10 cm infront of a plane mirror. If you stand

behind the object 50 cm from the mirror and looks at its image, the distance focussed for your eye will be

1) 60 cm 2) 20 cm 3) 40 cm 4) 80 cm

54. A ray reflected successively from two plane mirrors inclined at a

certain angle undergoes a deviation of 0300 . The number of observable images

1) 60 2) 12

3) 11 4) 5 55. A ray of light is incident towards a plane mirror at an angle of 20-

degrees with the mirror surface. What will be the angle of reflection?

1) 600 2) 700 3) 800 4) 900

Page 62: 8 & 9_MPC_SHP_E1 & E2& E4 & E5

CLASS-VIII_SUBGROUP_E1 TO E3 SIMPLE HOLIDAY PACKAGE

NARAYANA GROUP OF SCHOOLS Page 22 of 34

56. Two plane mirrors are arranged at right angles to each other as shown in figure. A ray of light is incident on the horizontal mirror at angle ‘ ’ . For what value of the ray emerges parallel to the incoming ray after reflection from the vertical mirror?

1) 060 2) 030

3) 045 4) all of these

57. An object moves with 3 m/s towards right while the mirror moves 2 m/s towards the left as shown. Then the velocity of image.

1) 7 m/s towards left 2) 7m/s towards right

3) 5 m/s towards right 4) 5 m/s towards left 58. Fig. Shows a plane mirror onto which a light ray is incident. If the

incidenting light ray is turned by 010 and the mirror by as shown,

then the angle turned by 020 the reflected ray is

1) 030 clockwise 2) 030 anticlock wise

3) 050 clockwise 4) 050 anticlock wise

59. Two plane mirrors are at 060 to each other. If an object is placed between them, then the number of images will be

1) 5 2) 9

3) 7 4) 8

60. A man runs towards a mirror at a rate of 13 ms . If the mirror is at

rest, his image will have a velocity (with respect to man)

1) 112 ms 2) 16ms

3) 16 ms 4) 112ms

61. A concave mirror having a focal length 20 cm is placed in front

of an illuminated point source at a distance of 30 cm from it.

The location of the image is

1) 50 cm 2) 30 cm

3) 60 cm 4) 25 cm

Page 63: 8 & 9_MPC_SHP_E1 & E2& E4 & E5

CLASS-VIII_SUBGROUP_E1 TO E3 SIMPLE HOLIDAY PACKAGE

NARAYANA GROUP OF SCHOOLS Page 23 of 34

62. An object is placed at a distance 2 f from the pole of a convex

mirror of focal length f . The linear magnification is

1) 1

3

2) 2

3

3) 3

4

4) 1

63. When object is placed between centre of curvature (C) and focus (f) of a concave mirror the nature and size of the image is

1) real, inverted and very

small

2) real, inverted, same size of object

3) real, inverted and

bigger than the object

4) virtual, erect and enlarged

64. A U - shaped wire is placed before a concave mirror having radius of curvature 20 cm as shown in figure. The total length of the image is

1) 20 cm 2) 15 cm

3) 10 cm 4) 5 cm

65. A convex mirror is used to form an image of a real object. The wrong statement is

1) the image lies between

the pole and focus

2) the image is diminished in size

3) the image is erect 4) the image is real

66. A 3cm tall object is placed at a distance of 7. 5 cm from a convex mirror of focal length 6 cm. The size of the image is

1) 1.33 cm 2) 0.75 cm

3) 3.33 cm 4) 0.3 cm

67. The diameter of spherical mirror in which reflection takes place is called

1) radius of curvature 2) centre of curvature

3) linear aperture 4) focal length

68. In which of the following case the image formed by a convex mirror of real object is larger than the object?.

1) When u 2f 2) When u 2f

3) for all values of u 4) for no value of u

69. A spherical mirror whose inner hollow surface is the reflecting surface is called

1) concave mirror 2) convex mirror

3) plane mirror 4) none of these

Page 64: 8 & 9_MPC_SHP_E1 & E2& E4 & E5

CLASS-VIII_SUBGROUP_E1 TO E3 SIMPLE HOLIDAY PACKAGE

NARAYANA GROUP OF SCHOOLS Page 24 of 34

70. When a convex mirror of focal length ‘ f’ produces an image equal

to

1th

n of the size of the object, then distance of the object

from the mirror is

1) f

n

2) n 1 f

3) n 1 f 4) fn

71. Which of the following forms a virtual and erect image for all

positions of a real object?

A) plane mirror B) convex mirror

1) Both A and B 2) Only A

3) Only B 4) Neither A nor B

72. For a concave mirror of radius of curvature is 36 cm, if object

distance is u 18 cm then the image formed is

1) Real image, erect

image

2) Real image, inverted image

3) Virtual image, erect

image

4) Virtual image, inverted image

73. Which of the following mirror can form real and virtual image?

1) Concave mirror 2) Convex mirror

3) Both Concave and

convex mirror

4) Neither Concave nor convex mirror

74. The image formed by a concave mirror is observed to be virtual,

erect and larger than the object. Where should be the position of

the object?

1) Between the principal

focus and the centre of

curvature

2) At the centre of curvature

3) Beyond the centre of

curvature

4) Between the pole of the mirror and its

principal focus

75. A full length image of a distant tall building can definitely be

seen by using

1) a concave mirror 2) a convex mirror

3) a plane mirror 4) both concave as well as plane mirror

76. A 9 cm long object is placed (perpendicular to principal axis) at

40cm infront of a concave mirror of radius of curvature 40 cm.

The size of the image is _______cm.

1) 9 2) 18

3) 3 4) 4.5

Page 65: 8 & 9_MPC_SHP_E1 & E2& E4 & E5

CLASS-VIII_SUBGROUP_E1 TO E3 SIMPLE HOLIDAY PACKAGE

NARAYANA GROUP OF SCHOOLS Page 25 of 34

77. Image formed when the object is placed at the focus(F) of the convex mirror.

Then image is formed

1) behind the mirror 2) In front of the mirror

3) At the focus 4) None of these

78. Image formed when the object is placed at the focus(F) of the convex mirror.

Then image size is

1) Diminished 2) Magnified

3) Highly Magnified 4) All of these

79. Find the focal length of a convex mirror whose radius of

curvature is 32 cm.

1) -16cm 2) +16cm

3) -64cm 4) +64cm

80. For a convex mirror according to sign convention which of the

following is/are positive.

A) focal length B) object distance C) image distance

D) radius of curvature

1) Only A and C 2) Only A and D

3) Only B, C,D are

positive

4) Only A, C,D are positive

Page 66: 8 & 9_MPC_SHP_E1 & E2& E4 & E5

CLASS-VIII_SUBGROUP_E1 TO E3 SIMPLE HOLIDAY PACKAGE

NARAYANA GROUP OF SCHOOLS Page 26 of 34

VIII_PHYSICS_SHP_E1 TO E3_KEY

Q.No. 1 2 3 4 5 6 7 8 9 10

Key 1 2 2 2 2 2 1 2 3 2

Q.No. 11 12 13 14 15 16 17 18 19 20

Key 3 1 4 4 3 2 3 3 2 1

Q.No. 21 22 23 24 25 26 27 28 29 30

Key 2 3 1 1 1 1 3 1

1 3

Q.No. 31 32 33 34 35 36 37 38 39 40

Key 2 1 3 2 3 3 1 2 2 1

Q.No. 41 42 43 44 45 46 47 48 49 50

Key 4 1 1 2 3 1 3 1 3 4

Q.No. 51 52 53 54 55 56 57 58 59 60

Key 1 3 1 3 2 4 1 1 1 2

Q.No. 61 62 63 64 65 66 67 68 69 70

Key 3 1 3 3 4 1 3 4 1 3

Q.No. 71 72 73 74 75 76 77 78 79 80

Key 1 2 3 4 2 1 1 1 2 4

Page 67: 8 & 9_MPC_SHP_E1 & E2& E4 & E5

CLASS-VIII_SUBGROUP_E1 TO E3 SIMPLE HOLIDAY PACKAGE

NARAYANA GROUP OF SCHOOLS Page 27 of 34

CHEMISTRY (Q.No.1 TO 80)

1. Co-ordination number of cation is maximum in

1)NaCl 2)CsCl 3)ZnO 4) KCl

2. Water acts as a powerful ionizing solvent due to its high

1)Polar nature 2)ionic nature

3)dielectric constant 4)Covalent nature

3. Arrange the following in the decreasing order of their donating ability:

Ca, Mg, K, Cs, Al, B?

1)Ca < Mg < K < Cs < Al < B 2)B < Al < Cs < K < Mg < Ca

3)B < Al < Mg < Ca < K < Cs 4)Cs > K > Ca > Mg > Al > B

4. Which of the following is a favorable factor for cation formation

1)Low ionisation potential 2)High electron affinity

3)High electro negativity 4)Small atomic size

5. Number of ion pairs in CsCl unit cell is

1)1 2)2 3)4 4) 8

6. The stability of an ionic compound is mostly due to

1)ionization energy 2)electron affinity

3)lattice energy 4)electro negativity

7. Among the following correct statement is /are

(A) NaCl is FCC with coordination number 6

(B) CsCl is BCC with coordination number 8

(C) CsCl is more ionic than NaCl

(D) NaCl& CsCl are water soluble

1)A,B only 2)C,D only 3) A,B,C,D 4)B only

8. Arrange the following is the decreasing ionic nature : MCl, MCl2 and MCl

1)MCl > MCl2 > MCl3 2)MCl2 < MCl < MCl3

3)MCl3< MCl2 < MCl 4)MCl < MCl3 < MCl2

9. Among LiCl, BeCl2, BCl3 and CCl4, the ionic bond character follows the

increasing order:

1) 2 3 4LiCl BeCl BCl CCl 2) 2 3 4LiCl BeCl BCl CCl

3) 2 4 3LiCl BeCl CCl BCl 4) 4 3 2CCl BCl BeCl LiCl

10. Maximum covalent nature can be seen in

1)CsF 2)RbF 3)NaF 4)LiF

11. The compound having less ionic nature is

1)BaCl2 2)MgCl2 3)SrCl2 4)BeCl2

Page 68: 8 & 9_MPC_SHP_E1 & E2& E4 & E5

CLASS-VIII_SUBGROUP_E1 TO E3 SIMPLE HOLIDAY PACKAGE

NARAYANA GROUP OF SCHOOLS Page 28 of 34

12. Which of the following is correct .

1) KF is more covalent than LiF

2) SnCl4 is is more covalent than SnCl2

3) The order of lattice energy NaF>Mgo>AlN

4) Polarising power charge on anion

13. Which of the following is more ionic?

1)Si3N4 2)AlN 3)BN 4)Ca3N2

14. If the radius ratio of cation to anion is 0.414 to 0.732 . Coordination

number is 4. The shape of the crystal lattice is

1)Linear 2) Planar triangle 3) Tetra hedral 4) Square planar

15. Which of the following is correct regarding covalent character of halide

ions ?

1) F Cl Br I 2) F Cl Br I

3)Cl F Br I 4)Cl F Br I

16. Molecule having an incomplete octet in the central atom is

1)PCl5 2)NH3 3)BCl3 4) H2O

17. Molecule having maximum number of lone pairs of electrons on central

atom is

1)PH3 2)H2S 3)CH4 4) BrF5

18. Bonds present in sodium hydroxide

1)Covalent and dative 2)Covalent and ionic

3)Covalent and hydrogen bond 4)Partial ionic

19. The formal charge on nitrogen atom in 4NH ion is ?

1)-1 2)0 3)+1 4)-2

20. Example of super octet molecule is :

1)F3Cl 2)PCl5 3)IF7 4) All of these.

21. The covalence of central atom is maximum in

1)HCN 2)NH4+ 3)PCl5 4)H2O

22. The number of electron pairs present in the valency shell of central atom

in SF6 molecule are

1)4 2)6 3)8 4) 7

23. Expanded octet can be observed in the valency shell of the central atom

in

1)NH3 2)CH4 3)PCl5 4) BeCl2

24. Molecule having maximum number of covalent bonds is

1)NH4OH 2)NH4Cl 3)CO(NH2)2 4) CH3OH

Page 69: 8 & 9_MPC_SHP_E1 & E2& E4 & E5

CLASS-VIII_SUBGROUP_E1 TO E3 SIMPLE HOLIDAY PACKAGE

NARAYANA GROUP OF SCHOOLS Page 29 of 34

25. Number of bonded electrons in ethane molecule are (H3C – CH3)

1)7 2)12 3)10 4) 14

26. The total number of protons in 20 g of calcium carbonate is (N0 =

6.02 × 1023)

1)1.5057 × 1024 2)2.0478 × 1024 3)3.0115 × 1024 4)6.023 × 1024

27. Co-ordination number of cation is maximum in

1)NaCl 2)CsCl 3)ZnO 4)KCl

28. The types of hybrid orbitals of nitrogen in 2 3 4NO ,NO and NH

respectively are expected to be

1) 3 2sp,sp andsp 2) 2 3sp,sp andsp 3) 2 3sp ,spandsp 4) 2 3sp ,sp andsp

29. The weight of 8gm. atoms of nitrogen is

1)56 gms. 2)112 gms. 3)42 gms. 4)28 gms.

30. According to Fajans rule covalent bond is favoured by

1)Small cation and large anion 2)Small cation and small anion

3)Large cation and large anion 4)Large cation and small anion

31. In 3

4PO ion the formal charge on the oxygen atom of P–O bond is

1)+ 1 2)– 1 3)– 0.75 4)+ 0.75

32. How many years it would take to spend Avogadro’s number of rupees at the rate of 10 lac rupees per second?

1)1.9090 × 1015 year 2)1.9099 × 1010 year

3)1.9800 × 105 year 4)1.978 × 1020 year

33. The number of atoms present in one mole of an element is equal to Avogadro number. Which of the following element contains the greatest number of atoms?

1)4g He 2)46g Na 3)0.40g Ca 4)12g He

34. Calculate the oxidation number of the element which is

underlined from the following :

i) 3HNO ii) HNC

1)(i ) -3 (i i ) +5 2)(i ) -3 (i i ) -5

3)(i ) 5 (i i ) 5 4)(i ) +5 (i i ) -3

35. For which of the following compounds oxidation state of oxygen is

1

2 ?

1)K2CrO4 2)OF2 3)H2O 4)KO2

Page 70: 8 & 9_MPC_SHP_E1 & E2& E4 & E5

CLASS-VIII_SUBGROUP_E1 TO E3 SIMPLE HOLIDAY PACKAGE

NARAYANA GROUP OF SCHOOLS Page 30 of 34

36. The oxidation number of IA group elements in all their compounds

is:

1)+2 2)+1 3)+3 4) +1/2

37. The oxidation number of IIA group elements in all their

compounds is:

1)+1 2)+2 3)+3 4) +4

38. The oxidation number of H in most of its compounds is:

1)+1 2)+2 3)0 4) +3

39. From which of the following the oxidation number of underlined

element is 4?

1) 2CO 2) 2 2CH Cl 3)CO 4) 4 10C H

40. A number which indicates the loss or gain or sharing of electrons

by atom during the formation of a chemical compound is called :

1)Oxidation number 2)Co-ordination number

3)Incommensurable number 4)Avogadro’s number

41. Oxidation number of hydrogen in HCl molecule :

1)+1 2)-1 3)+2 4) -2

42. The oxidation number of an atom in its elementary form is always

1)0 2)1 3)Practional 4)2

43. The common oxidation state of alkaline earth metals :

1)+1 2)+2 3)+3 4) +4

44. The oxidation number of oxygen in peroxides :

1)-1 2)-2 3)-3 4) -4

45. The oxidation number of oxygen in superoxides :

1)-1 2)-1/2 3)-3 4) -4

46. The oxidation number of CH4 molecule is :

1)0 2)1 3)2 4) 3

47. The oxidation number of chlorine in HCl molecule

1)-1 2)-2 3)+1 4) +2

Page 71: 8 & 9_MPC_SHP_E1 & E2& E4 & E5

CLASS-VIII_SUBGROUP_E1 TO E3 SIMPLE HOLIDAY PACKAGE

NARAYANA GROUP OF SCHOOLS Page 31 of 34

48. The oxidation number of an element in a compound is evaluated

on the basis of certain rules. Which of the following rules is not

correct in this respect?

1)The oxidation number of hydrogen is always +1.

2)The algebraic sum of all the oxidation numbers in a compound is

zero.

3)An element in the free or the uncombined state bears oxidation

number zero.

4)In all its compounds, the oxidation number of fluorine is – 1.

49. In which of the following compounds, an element exhibits two

different Oxidation states.

1)NH2OH 2)NH4NO3 3)N2H4 4) N3H

50. The oxidation number of Mn in 4KMnO molecule

1)+2 2)+7 3)+6 4) +3

51. Chlorine is passed into dilute cold KOH solution. What are the

oxidation numbers of chlorine in the products formed?

1)-1, +5 2)-1, +3 3)+1, +7 4) +1, -1

52. The oxidation number of Nitrogen in 3NH molecule

1)+3 2)-5 3)+5 4) -3

53. Which one of the following reaction is an example of de-electro

nation?

1)Fe - 2e- Fe2+ 2)Fe3+ + 1e- Fe2+

3)Sn4+ + 2e- Sn2+ 4)Cl2 + 2e- 2Cl=

54. In 3NO ion, the number of bond pairs and lone pairs of electrons on

nitrogen atom are

1)2, 2 2)3, 0 3)1, 3 4)4, 0

55. Number of sigma bonds and pi bonds present in Pentane are (

3 2 2 2 3H C-CH -CH -CH CH )

1)12 , 1 2)13 , 0 3)0 , 13 4)16 , 0

56. Which one of the following is correct statement

i) A pi bond is formed when a sigma already exists

ii) A pi bond may be formed by the overlapping of ‘p’ or ‘d’ orbital’s

iii) A pi bond is formed by the overlapping of s-orbital’s

A pi bond is formed by the lateral overlapping of atomic orbital’s

1)Only i and ii 2)Only i ,ii and iii

3) Only i ,iii and iv 4)Only i ,ii and iv

Page 72: 8 & 9_MPC_SHP_E1 & E2& E4 & E5

CLASS-VIII_SUBGROUP_E1 TO E3 SIMPLE HOLIDAY PACKAGE

NARAYANA GROUP OF SCHOOLS Page 32 of 34

57. BCl3 molecule is planar while NCl

3 molecule is pyramidal because

1) BCl3 does not have lone pair on B but NCl3 has on N.

2) B-Cl bond is more polar than N-Cl bond

3) N atom is smaller than B

4) N-Cl bond is more covalent than B-Cl bond.

58. Which of the following species has tetrahedral geometry?

1) 4BH 2) 2NH 3) 2

3CO 4) 3H O

59. The central atom in PCl5 has five pairs of valency electrons. The

shape of the molecule as per VSEPR theory is

1)Trigonal bipyramidal 2)hexagonal pyramid

3)pentagonal bipyramidal 4)Square bipyramidal

60. p-p overlapping is present in

1)Cl2 2)H

2 3)HCl 4)HF

61. The weight of 8gm. atoms of nitrogen is

1) 56 gms. 2) 112 gms.

3) 42 gms. 4) 28 gms.

62. Volume occupied by 44 g of 2CO is

1) 2240 lit at STP 2) 2.24 lit at STP

3) 22.4 lit at STP 4) 224 lit at STP

63. Which of the following is Avogadro number?

1) 6.023 × 1023

atoms 2) 3.0115 × 1023

atoms

3) 1.505 × 1023

atoms 4) 12.0 × 1023

atoms

64. Which of the following is correct?

1) Molecular weight of oxygen is 32.

2) Gram molecular mass of sulphur (S8) is 256 g.

3) The weight of one molecule of O3 is 48 amu.

4) All

65. The weight of 1 mole of atoms of an element = _______.

1) 1.66× 10–24

g 2) Gram molecular weight

3) Gram atomic weight 4) 6.023×1023

g

66. 1 amu is equal to the mass of:

1) 1

12th of C - 12 atom 2)

1

14th of O -16 atom

3) 1g of H2 4) 1.66 × 10

–23 kg

67. The weight of Helium atom in grams is:

1) 2 2) 4 3) 6.64 × 10–24

4) 1.66 × 10–24

68. Which of the following is the smallest particle of matter that exist independently?

1) Atom 2) Molecule 3) element 4) compound 69. A: H

2O, CH

4, NH

3; B: H

2, N

2, O

2, F

2

1) ‘A’ contains homogeneous molecules. 2) ‘B’ contains homogeneous molecules. 3) ‘A’ contains heterogeneous molecules. 4) Both 2 & 3

Page 73: 8 & 9_MPC_SHP_E1 & E2& E4 & E5

CLASS-VIII_SUBGROUP_E1 TO E3 SIMPLE HOLIDAY PACKAGE

NARAYANA GROUP OF SCHOOLS Page 33 of 34

70. The mass of one atom of an unknown element is 4×1.66×10–24

g. The

element is:

1) Hydrogen 2) Helium

3) Oxygen 4) Sulphur

71. One a. m. u is equal to:

1) 271.66 10 g 2)

241.66 10 g

3) 221.66 10 g 4)

201.66 10 g

72. th

Mass of1atomof thatelement

1/12 mass of C 12atom

1) RAM 2) GAW

3) Mol. Wt 4) Eq. Wt

73. The atomic weight of Chlorine is:

1) 35g 2) 37g

3) 35.5g 4) 36.5g

74. Number of moles =

1) Weight / Molecular weight 2) Weight / Atomic weight

3) Weight /Number of atoms 4) Both 1 & 2

75. One mole electrons means:

1) N electrons 2) 236.023 10 electrons

3) 0.55mg electrons 4) All of these

76. The shape of CH4; SO

4

-2; [PO4]

-3 is

1) Trigonal planar 2) Angular

3) Tetrahedral 4) Trigonal Bipyramidal

77. The hybridization of atomic orbitals of nitrogen in 2 2 4,NO NO and NH

are

1) 2 3 2sp ,sp andsp respectively 2)

2 3sp,sp andsp respectively

3) 2 3sp ,spandsp respectively 4)

2 3sp ,sp andsp respectively

78. Isostructural species are those which have the same shape and

hybridisation. Among the given species identify the isostructural pairs.

1) [NF3 and BF3] 2) [ 4BF and 4NH

]

3) [BCl3 and BrCl3] 4) [NH3 and 3NO ]

79. On catalytic hydrogenation, ethylene gives ethane during this reaction

2 2 3 3H C CH H C CH

1) Hybridization of carbon atoms changes from sp2 to sp

3

2) bond angle decreases from 1200 to 109.5

0

3) C-C bond length increases from1.34 A0 to 1.54A

0

4) all of these.

80. The ratio of pure orbitals to hybridized orbitals in ethylene is

1) 2:3 2) 3:1

3) 1:1 4) 1:3

Page 74: 8 & 9_MPC_SHP_E1 & E2& E4 & E5

CLASS-VIII_SUBGROUP_E1 TO E3 SIMPLE HOLIDAY PACKAGE

NARAYANA GROUP OF SCHOOLS Page 34 of 34

VIII_CHEMISTRY_SHP_E1 TO E3_KEY

Q.No. 1 2 3 4 5 6 7 8 9 10

Key 2 3 4 1 1 3 3 1 4 4

Q.No. 11 12 13 14 15 16 17 18 19 20

Key 4 2 4 4 2 3 2 2 3 4

Q.No. 21 22 23 24 25 26 27 28 29 30

Key 3 2 3 3 4 4 2 2 2 1

Q.No. 31 32 33 34 35 36 37 38 39 40

Key 2 2 4 4 4 2 2 1 1 1

Q.No. 41 42 43 44 45 46 47 48 49 50

Key 1 1 2 1 2 1 1 1 2 2

Q.No. 51 52 53 54 55 56 57 58 59 60

Key 4 4 1 2 4 4 1 1 1 1

Q.No. 61 62 63 64 65 66 67 68 69 70

Key 2 3 1 4 3 1 3 2 4 2

Q.No. 71 72 73 74 75 76 77 78 79 80

Key 2 1 3 4 4 3 2 2 4 3

Page 75: 8 & 9_MPC_SHP_E1 & E2& E4 & E5

CLASS-IX_SUBGROUP_E1 TO E3 SIMPLE HOLIDAY PACKAGE

NARAYANA GROUP OF SCHOOLS Page 1 of 40

(OLYMPIAD AND E-TECHNO)

SIMPLE HOLIDAY

PACKAGE PONGAL

Mathematics

Physics

Chemistry

CLASS – IX – E1 to E3

IX_MPC_E1 to E3_SHP

INDEX

SUBJECT PAGE NO.

1. MATHEMATHICS 2 - 14

2. PHYSICS 15 - 29

3. CHEMISTRY 30 - 41

Page 76: 8 & 9_MPC_SHP_E1 & E2& E4 & E5

CLASS-IX_SUBGROUP_E1 TO E3 SIMPLE HOLIDAY PACKAGE

NARAYANA GROUP OF SCHOOLS Page 2 of 40

MATHEMATICS (Q.No.1 to 80)

1. The diagonals of a parallelogram formed by the lines

1 1 1a x+b y+c = 0, 1 1 3a x+b y+c = 0, 2 2 2a x +b y +c =0,

2 2 4a x+b y+c =0 will be right angles if

1) 1 1

2 2

a b

a b

2) 2 2

1 2

2 2

1 2

a a

b b

3) 2 2 2 2

1 1 2 2 a b a b 4) None of these

2. The perpendicular distance from (1,2) to the line 3x + 4y + 14 = 0 is

1) 7 units 2) 3 units 3) 5 units 4) 2 units

3. The angle between the lines 2x + 3y – 5 = 0, 5x + ky – 6 =0 is 450

then the value of k is

1) 1 2) 2

3) - 1 4) -2

4. One vertex of a square ABCD is A(3,4) and the equation of one

diagonal BD is 3x + 4y + 5 = 0, then C =

1) 21 28,

5 5

2) -21 28

,5 5

3) 21 -28,

5 5

4) -21 -28

,5 5

5. If the algebraic sum of the perpendicular distances from the

points (2, 0), (0, 2) and (1, 1) to a variable straight line be zero,

then the line passes through the point

1) (–1, 1) 2) (1, 1)

3) (1, – 1) 4) (–1, –1)

6. Equation of the straight line joining the points 2

1 2am , 2am and

2

2 1am , 2am is

1)

1 2

2 2

1 2

2

2 0

1 22x+ m +m

y am m

a m m

2) 2x – (m1 + m2)y

+ 2am1m2 = 0

3)

1 2

2 2

1 2

2

2 0

1 22x+ m +m

y am m

a m m

4)

None of these

Page 77: 8 & 9_MPC_SHP_E1 & E2& E4 & E5

CLASS-IX_SUBGROUP_E1 TO E3 SIMPLE HOLIDAY PACKAGE

NARAYANA GROUP OF SCHOOLS Page 3 of 40

7.

The normal form of the line 4x + 3y + 12 = 0 is

1)

4 3 12

5 5 5x y

2) 4 3 12

5 5 5

x y

3) 4 3 12

5 5 5 x y

4) None of these

8. The equation of the straight line passing through the point (3,-4)

and making X and Y-intercepts which are in the ratio

2 : 3 is

1) 3x + 2y - 1 = 0 2) 2x + 3y + 12 = 0

3) 3x + 5y + 11 = 0 4) 3x + y + 1 = 0

9. If the line 2x - 3y + 1 = 0 is denoted by L, then the points (3, 4) and (1,2) are

1) lie on L 2) lie on same side of L 3) lie on opposite sides of

L 4) equidistant from L

10. The reflection of the point (4,-13) in the line 5x + y + 6 = 0 is

1) (-1,-14) 2) (3,4) 3) (1,2) 4) (-4,13)

11. The image of the point(–2, –5) w.r.t the origin is

1) (2,5) 2) (-2,5)

3) (2,–5) 4) None of these

12. If 3x ky 2 0, 2x y 2 0 are perpendicular, then the value

of k is

1) 5 2) -6

3) 6 4) -5

13. If is the angle between

x y x y1, 1

a b b a,then cos =

1) 2 2

ab

a b

2) 2 2

ab

a b

3) 2 2

2ab

a b

4)

2 2

2ab

a b

14. If the four straight lines 0, 0,ax by p ax by q 0cx dy r and

0cx dy s form a parallelogram, then the area of the

parallelogram so formed is

1)

p q r s

bd ac

2)

p q r s

bc ad

Page 78: 8 & 9_MPC_SHP_E1 & E2& E4 & E5

CLASS-IX_SUBGROUP_E1 TO E3 SIMPLE HOLIDAY PACKAGE

NARAYANA GROUP OF SCHOOLS Page 4 of 40

3)

p q r s

bd ac

4)

p q r s

bc ad

15. 1,1 , 5,3A B are opposite vertices of a square in the XY – plane.

Find the equation of the other diagonal (not passing through A,B) of the square.

1) 3x + y + 8 = 0 2) x + 3y + 8 = 0 3) 3x + y - 8 = 0 4) x + 3y - 8 = 0

16. A ray of light passing through the point (1,2) reflects on the X-axis at a point A and the reflected ray passes through the point (5,3). Find the coordinates of A.

1) 13,0

5

2) 11

,05

3) (13, 0) 4) (11, 0)

17. The mid points of the sides of a triangle are (5, 0), (5, 12) and (0, 12).The circumcenter of this triangle is

1) (0, 0) 2) (10, 0) 3) (0, 24) 4) (5, 12)

18. If 2 26x +5xy +y = 0 represents a pair of lines then

I: 1 2 5m m

II: 1 2

1m m Which of the above statements is/are correct?

1) only I 2) only II 3) both I and II 4) neither I nor II

19. The equation to the two lines represented by the equation

2 2x 2xysec y 0 is

1) x y cosec cot 0, x y

2) x y cosec 0, x y cot 0

3) x y 0, x sin 0 4)

x y s ec tan 0,

x y s ec tan 0

20. A variable straight line drawn through the point of intersection

of the straight lines 1x y

a b and 1

x y

b a meets the coordinate

axes at A and B, then the locus of the mid-point of AB is.

1) 2 a b xy ab x y 2) 2 a b xy ab x y

3) 2 a b xy ab x y 4) 2 a b xy ab x y

21. The incenter of the triangle formed by the straight lines

3 , 3y x y x and 3.y is

1) (0, -2) 2) (0, 2)

Page 79: 8 & 9_MPC_SHP_E1 & E2& E4 & E5

CLASS-IX_SUBGROUP_E1 TO E3 SIMPLE HOLIDAY PACKAGE

NARAYANA GROUP OF SCHOOLS Page 5 of 40

3) (-2, 0) 4) (2,3)

22. Equation of the line passing through the point of intersection of

the lines 2x + 3y – 1 = 0, 3x + 4y – 6 = 0 and Parallel to 5x – 2y – 7 = 0 is

1) 5x – 2y + 88 =0 2) 10x – 4y – 88 =0 3) 5x – 2y – 88 =0 4) 10x – 4y + 88 =0

23. If a b c and if 0 ax by c 0 bx cy a and 0 cx ay b are

concurrent. Then the value of 2 -1 -1 2 -1 -1 2 -1 -1a b c b c a c a b3 3 3 is

1) 1 2) 27 3) 8 4) 16

24. The nature of the triangle formed by the lines 2 23 0x y and

2x is

1) scalene 2) right angled 3) equilateral 4) isosceles

25. 1,3A and 2 / 5, 2 / 5C are the vertices of a triangle ABC and the

equation of the internal angle bisector of ABC is 2.x y

The equation of side BC is

1) 7 3 4 0x y 2) 7 3 4 0x y

3) 7 3 4 0x y 4) 7 3 4 0x y

26. 1,3A and 2 / 5, 2 / 5C are the vertices of a triangle ABC and the

equation of the internal angle bisector of ABC is 2.x y

The coordinates of vertex B are

1) 3 /10,17 /10 2) 17 /10,3 /10

3) 5 / 2,9 / 2 4) 1,1

27. 1,3A and 2 / 5, 2 / 5C are the vertices of a triangle ABC and the

equation of the internal angle bisector of ABC is 2.x y The

equation of side AB is

1) 3 7 24x y 2) 3 7 24 0x y

3) 13 7 8 0x y 4) 13 7 8 0x y

28. If ' ' is the angle between the lines represented by

2 2ax 2hxy by 0 , then

2 2

a bcos

a b 4h The angle between

the pair of lines 2 2x 2xysec +y =0 is

1) 2) y

3) 2 4) x

29. If ' ' is the angle between the lines represented by

2 2ax 2hxy by 0 , then

2 2

a bcos

a b 4h The angle between

the lines represented by 2 2 2 2 2y sec xy sec x tan 1 0 is

1) 4

2)

3

Page 80: 8 & 9_MPC_SHP_E1 & E2& E4 & E5

CLASS-IX_SUBGROUP_E1 TO E3 SIMPLE HOLIDAY PACKAGE

NARAYANA GROUP OF SCHOOLS Page 6 of 40

3) 2

4) None of these

30. If ' ' is the angle between the lines represented by

2 2ax 2hxy by 0 , then

2 2

a bcos

a b 4h If

2

is the angle

between the pair of lines 2 26x 5xy ky 0 , then the value of ‘k’

is

1) 6 2) 8 3) – 6 4) – 8

31. If three consecutive vertices of a parallelogram are

4,3,5 , 0,6,0 . 8,1,4A B C and D is the fourth vertex then the

angle between AC

and BD

is

1) 1 55

149 161

Cos 2)

1 65

149 161

Cos

3) 1 15

149 161

Cos 4)

1 3

149 161

Cos

32. The direction cosines of the line joining the points

A(-6, 5, -4), B(-5, 2, -4) is

1) 1 3, ,0

10 10

2) 2 3

, ,010 10

3) 1 2, ,0

10 10

4) None of these

33. The three lines with d.r’s , (1,1,2) ( 3 1, 3 1, 4) , ( 3 1, 3 1, 4)

forms

1) An equilateral triangle 2) A right angled triangle

3) An isosceles triangle 4) A right angled isosceles triangle

34. If is an angle given by

2 2 2

2 2 2

cos cos coscos

sin sin sin

where , , are the angles made by a

line with the axis , ,OX OY OZ

respectively then the value of is

Page 81: 8 & 9_MPC_SHP_E1 & E2& E4 & E5

CLASS-IX_SUBGROUP_E1 TO E3 SIMPLE HOLIDAY PACKAGE

NARAYANA GROUP OF SCHOOLS Page 7 of 40

1)

3

2)

6

3)

2

4)

4

35. If a line makes angles

5,

12 12

with oy, oz respectively where O =

(0, 0,0) then the angle made by that line with ox is

1) 45o 2) 90o

3) 60o 4) 30o

36. The vertices of a triangle are 2,3,5 , 1,3,2 , 3,5, 2 , then the

angles are

1) 300, 300, 1200 2) 1 0 11 5

,90 ,5 3

Cos Cos

3) 0 0 030 ,60 ,90 4) 1 0 11 2

,90 ,3 3

Cos Cos

37. An acute between the lines whose direction ratios are

(1, -2, 1) and (-1, 1, 0) is

1) 030 2) 0120

3) 060 4) None of these

38. The direction cosines of the line passing through P 2,3,1 and

the origin are

1) 2

14

3 1, ,14 14

2) 2 3

,14 14

1

14

3) 2 3 1, ,

14 14 14

4) 2 3 1, ,

14` 14 14

39. If a line OP where O is the origin makes angles , 45 and 60 with

,OX OY

andOZ

axes respectively then the direction cosines of OP

are

1) 1 1 1, ,2 2 2

2) 1 1 1

, ,2 22

3) 1 1 1, ,2 22

4) 1 1 1

, ,4 2 2

40. If P = (3, 4, 5), Q = (4, 6, 3), R = (-1, 2, 4) and S = (1, 0, 5) are four points then the projection of RS on PQ is

1) 8

3

2) 4

3

3) 4 4) None of these

41. The sum of the d.c's of the line which makes equal angles with

Page 82: 8 & 9_MPC_SHP_E1 & E2& E4 & E5

CLASS-IX_SUBGROUP_E1 TO E3 SIMPLE HOLIDAY PACKAGE

NARAYANA GROUP OF SCHOOLS Page 8 of 40

ox, oy, oz is

1) 1 2) 3

3) 1

3 3

4) 1

3

42. If 21OP and d.c.’s of OP

are

2 6 3, ,7 7 7

then P=

1) 6, 12,4 2) 6,18, 9

3) 3/ 2, 6,2 4) 5, 10,6

43. If 2,3,6P then the d.c.’s of OP

are

1) 3 4 12, ,

13 13 13

2) 6,6, 3

3) 2 3 6, ,7 7 7

4) 2,2, 1

44. If the line joining the points (2,3,4),(0,1,2) is perpendicular to

the line joining the points (x,0,4), (7,-4,3), then x =

1) 2

2) 10

3) 3

10

4) 10

3

45. If the direction ratios of two lines are given by

3lm - 4ln + mn = 0 and l + 2m + 3n = 0, then the angle between

the lines is

1)

2

2)

3

3)

4

4)

6

46. If the angles made by a straight line with the coordinate axis are

, ,2

then

1) 0 2)

6

3)

2

4)

47. If a line in the space makes angles , and with the

coordinate axes, then 2 2 2cos2 cos2 cos 2 sin sin sin

1) -1 2) 0

3) 1 4) 2

48. A line makes the same angle , with each of the x and z axis. If

Page 83: 8 & 9_MPC_SHP_E1 & E2& E4 & E5

CLASS-IX_SUBGROUP_E1 TO E3 SIMPLE HOLIDAY PACKAGE

NARAYANA GROUP OF SCHOOLS Page 9 of 40

the angle , which it makes with y-axis, is such that 2 2sin 3sin , then 2sin

1) 2/ 3 2) 2/ 5

3) 3/ 5

4) 1/ 5

49. A line AB in three-dimensional space makes angles 045 and 0120 with the positive x-axis and the positive y-axis respectively. If AB makes an acute angle with the positive z-axis, then

equals.

1) 030 2) 045

3) 060 4) 075

50. If the d.r.’s of a line are 3, 4,12 then d.c.’s of the line are

1) 4 2 4, ,3 3 3

2) 2 1 2

, ,3 3 3

3) 2 1 2, ,6 6 6

4) 3 4 12

, ,13 13 13

51. The points A(4, 9, 6), B(1, 6, 6) and C(0, 7, 10) form

1) a collinear points 2) a scalene triangle

3) a right angled isosceles

triangle

4) an equilateral triangle

52. If the points (1, 2, 3) is changed to the points (2,3,1) through

translation of axes, then the new origin is

1) (1, 1, –2) 2) (–1, –1, 2)

3) (3, 5, –2) 4) (3, 5, 4)

53. The locus of the point whose distance from y-axis is thrice its

distance from (1, 2, –1) is

1) 2 2 28x +9y +8z 18x 36y +18z+54=0

2) 2 2 28x +9y +8z 18x 36y 18z+54=0

3) 2 2 28x 9y +8z 18x 36y +18z+54=0

4) 2 2 28x +9y +8z 18x 36y+18z+54=0

54. If the straight line (x + y + 1) + K(2x – y – 1) = 0 is parallel to

2x + 3y – 8 = 0, then K =

1) 1

8

2) 1

8

3) 5 4) - 5

Page 84: 8 & 9_MPC_SHP_E1 & E2& E4 & E5

CLASS-IX_SUBGROUP_E1 TO E3 SIMPLE HOLIDAY PACKAGE

NARAYANA GROUP OF SCHOOLS Page 10 of 40

55. The ratio in which the line joining the points A(1, –1) and

B(2, 1) divides the line joining C(3, 4) and D(1, 2) is

1) 7 : 5 2) –7 : 5

3) –1 : 3

4) None of these

56. If the algebraic sum of the perpendicular distances from the

points (2, 0), (0, 2) and (1, 1) to a variable straight line be zero,

then the line passes through the point

1) (–1, 1) 2) (1, 1)

3) (1, – 1) 4) (–1, –1)

57. The centroid of the triangle formed by the lines

4x 7y 10 0, x y 5, 7x 4y 15 0 is

1) ( 1,2) 2) 68 361,

99 99

3) 35 295,

66 66

4) None of these

58. The value of ‘k’ if the equation 8x2–24lxy+18y2–6x +ky–5=0

represents a pair of parallel lines is

1) 9 2) 8

3) 7 4) 6

59. The centroid of the tetrahedron formed by the points 1 1 1, ,A x y z

, 2 2 2, ,B x y z , 3 3 3, ,C x y z and 4 4 4, ,D x y z is

1 2 3 4 1 2 3 4 1 2 3 4, ,4 4 4

x x x x y y y y z z z z

If A(1, 1, 3), B(4, 3, 2) and C(5, 2, 7) are three vertices of a

tetrahedron for which the centroid is5 17

4, ,2 4

, the fourth vertex

is

1) (6, 5, 8) 2) (6, 4, 8)

3) (5, 4, 8) 4) (6, 4, 5)

60. The centroid of the tetrahedron formed by the points 1 1 1, ,A x y z

, 2 2 2, ,B x y z , 3 3 3, ,C x y z and 4 4 4, ,D x y z is

1 2 3 4 1 2 3 4 1 2 3 4, ,4 4 4

x x x x y y y y z z z z

The centroid of the tetrahedron formed by the points

(8, 2, 0), (4, 6, 7), (3, 1, 2), (9, 2, 4) is

Page 85: 8 & 9_MPC_SHP_E1 & E2& E4 & E5

CLASS-IX_SUBGROUP_E1 TO E3 SIMPLE HOLIDAY PACKAGE

NARAYANA GROUP OF SCHOOLS Page 11 of 40

1) 1 15 3

, ,4 4 4

2) 11 13

6, ,4 4

3)

7 10, ,

4 4

4) 19 111, ,

44 4

61. The centroid of the tetrahedron formed by the points 1 1 1, ,A x y z

, 2 2 2, ,B x y z , 3 3 3, ,C x y z and 4 4 4, ,D x y z is

1 2 3 4 1 2 3 4 1 2 3 4, ,4 4 4

x x x x y y y y z z z z

If origin is the centroid of the tetrahedron with vertices

(2, -1, 3), (-1, 3, 1), (3, 4, -2), (a, b, c), then the ascending order of

a, b, c is

1) a, b, c 2) c, b, a

3) b, a, c 4) b, c, a

62. The equations of the lines bisector of the anlges between the

lines 1 1 1 2 2 2a x b y c 0, a x b y c 0 are

2 2 21 1 1

2 2 2 2

1 1 2 2

a x b y ca x b y c0

a b a b

The equations of the bisector of the angles between the lines 3x

– 4y + 7 = 0, 12x + 5y – 2 = 0 is

1) 5x + 5y +3 = 0 2) 10x – 3y + 7 = 0

3) 5x – 2y + 1 = 0 4) 11x – 3y + 9 = 0

63. The equations of the lines bisector of the anlges between the

lines 1 1 1 2 2 2a x b y c 0, a x b y c 0 are

2 2 21 1 1

2 2 2 2

1 1 2 2

a x b y ca x b y c0

a b a b

The equation to the pair of bisector of the angle between the

pair of lines 3x2 – 10xy + 4y2 =0 is

1) 2 25x xy 5y 0 2) 2 25x xy 5y 0

3) 2 25x xy 5y 0 4) 2 25x xy 5y 0

64. The equations of the lines bisector of the anlges between the

lines 1 1 1 2 2 2a x b y c 0, a x b y c 0 are

2 2 21 1 1

2 2 2 2

1 1 2 2

a x b y ca x b y c0

a b a b

The equation of the pair of bisectors of the angle between the

two straight lines 5x2 + 6xy – y2 = 0 is

Page 86: 8 & 9_MPC_SHP_E1 & E2& E4 & E5

CLASS-IX_SUBGROUP_E1 TO E3 SIMPLE HOLIDAY PACKAGE

NARAYANA GROUP OF SCHOOLS Page 12 of 40

1) 2 23x 2xy 3y 0 2) 2 2x 2xy y 0

3) 2 2x 2xy y 0

4) 3x2 – 4xy – 3y2 = 0

65. The angle between the lines whose direction cosines are

3 1 3, ,

4 4 2 and

3 1 3, ,

4 4 2is

1) 2

2)

6

3) 5

4)

3

66. The sum of the squares of the sines of the angles made by the

line AB with ox, oy, oz where ‘O’ is the origin is

1) 1 2) 2

3) -1 4) 3

67. If array make angles

3

and3

withOX and

OY respectively, then

the angle made by the ray withOZ is

1)

1 1cos

2

2)

1 3cos

2

3)

1 3cos

2

4) 0

68. If the dr’s of a line are 1 ,1 ,2 and it makes an angle

060 with the Y- axis then is

1) 1 3 2) 4 5

3) 2 2 5 4) 2 5

69. If the angle between line with d.c’s

2, ,

21 21 21

a b and other

line with d.c’s 3 3 6, ,

54 54 54

is 900 then a pair of possible

values of ‘a’ and ‘b’ respectively are

1) -1, 4 2) 4, 2

3) 4,1 4) -4,-2

70. If the angles made by a line with the positive directions of X and Y- axis are complementary angles then the angle made by the line with Z axis is

1) 0 2)

3

Page 87: 8 & 9_MPC_SHP_E1 & E2& E4 & E5

CLASS-IX_SUBGROUP_E1 TO E3 SIMPLE HOLIDAY PACKAGE

NARAYANA GROUP OF SCHOOLS Page 13 of 40

3)

4

4)

2

71. If the direction cosines of two nonparallel lines are related by 2l

+ m + 2n = 0 and 2 2 23 5 11 0 l m n then angle between them is

1) 0 2)

3) 2

4) None of these

72. If the direction ratios of two lines are (x, 3, 5) and (2, -1, 2) and

if the angle between those lines is4

,then ‘x’ is

1) 52 2) 42 3) 62 4) 60

73. Let a line makes an angle ‘ ’ with X and Z - axes and with

Y -axis. If sin 3 sin , then 2cos

1) 3

5

2) 5

3

3) 2

5

4) 1

5

74. If 1 1

, , 022

n n are the d.c's of a line then the angle made by

that line with oz where O= (0, 0, 0) is

1) 90o 2) 150o

3) 60o 4) None of these

75. If 21OP and d.c.’s of OP

are

2 6 3, ,7 7 7

then P=

1) 6, 12,4 2) 6,18, 9

3) 3/ 2, 6,2 4) 5, 10,6

76. If the line joining the points (2,3,4),(0,1,2) is perpendicular to the line joining the points (x,0,4), (7,-4,3), then x =

1) 2

2) 10

3) 3

10

4) 10

3

77. If the angles made by a straight line with the coordinate axis are

, ,2

then

1) 0 2)

6

3)

2

4)

78. A line makes the same angle , with each of the x and z axis. If

the angle , which it makes with y-axis, is such that 2 2sin 3sin , then 2sin

1) 2/ 3 2) 2/ 5

Page 88: 8 & 9_MPC_SHP_E1 & E2& E4 & E5

CLASS-IX_SUBGROUP_E1 TO E3 SIMPLE HOLIDAY PACKAGE

NARAYANA GROUP OF SCHOOLS Page 14 of 40

3) 3/ 5 4) 1/ 5

79. A line AB in three-dimensional space makes angles 045 and 0120 with the positive x-axis and the positive y-axis respectively. If AB makes an acute angle with the positive z-axis, then equals.

1) 030 2) 045

3) 060 4) 075

80. If the d.r.’s of a line are 3, 4,12 then d.c.’s of the line are

1) 4 2 4, ,3 3 3

2) 2 1 2

, ,3 3 3

3) 2 1 2, ,6 6 6

4) 3 4 12

, ,13 13 13

IX_MATHEMATICS_SHP_E1 TO E3_KEY

Q.No. 1 2 3 4 5 6 7 8 9 10

Key 4 3 1 4 2 1 2 1 2 1

Q.No. 11 12 13 14 15 16 17 18 19 20

Key 1 2 4 4 3 1 4 2 4 4

Q.No. 21 22 23 24 25 26 27 28 29 30

Key 2 3 2 3 2 1 1 1 3 3

Q.No. 31 32 33 34 35 36 37 38 39 40

Key 1 1 1 1 2 4 1 4 2 2

Q.No. 41 42 43 44 45 46 47 48 49 50

Key 2 2 3 1 1 3 3 2 3 4

Q.No. 51 52 53 54 55 56 57 58 59 60

Key 3 2 4 1 3 2 2 1 4 2

Q.No. 61 62 63 64 65 66 67 68 69 70

Key 3 4 2 3 4 2 1 4 3 4

Q.No. 71 72 73 74 75 76 77 78 79 80

Key 3 1 1 1 2 1 3 2 3 4

Page 89: 8 & 9_MPC_SHP_E1 & E2& E4 & E5

CLASS-IX_SUBGROUP_E1 TO E3 SIMPLE HOLIDAY PACKAGE

NARAYANA GROUP OF SCHOOLS Page 15 of 40

PHYSICS ( Q.NO. 1 TO 80)

1. The level of water in a tank is 5m high. A hole of area 2 cm2 is made at the bottom of the tank. The rate of leakage of

water from the hole in m3/s is (g= 10 ms-2)

1) 32 10 2) 42 10 3) 20 4) 22 10

2. A fresh water on a reservoir is 10 m deep. A horizontal pipe

4.0 cm in diameter pass through the reservoir 4.0 m below the water surface as shown in figure. If a plug secures the pipe opening , then the friction force between the plug and pipe wall is (Assume area of reservoir to be too large)

1) 49.2 N 2) 73.9 N 3) 10.84 N 4) 37.4 N

3. The readings of a meter which reads pressure fitted in a

closed pipe is 7.5×105 Nm–2. On opening the valve of the

pipe, the reading of that meter reduces to 5.5×105 Nm–2. Then speed of water flowing in pipe is

1) 10 1ms 2) 20 1ms 3) 0.1 1ms 4) 0.01 1ms

4. An aeroplane of mass 5000 kg is flying at an altitude of 3 km.

If the area of the wings is 10m2 and pressure at the lower surface of wings is 50 6 10. pa , the pressure on the upper surface

of wings is (in pascal) ( g= 10 ms-2)

1) 359 10 2) 42 10 3) 355 10 4) 59

5. A train goes past a person standing at the edge of a platform at high speed. Then the person will be

1 4

1) attracted towards the train 2) unaffected by the train 3) reflected by the train 4) affected only if its speed is

greater than critical velocity. 6. The dynamic lift of an aeroplane is based on

1) Torricelli theorem 2) Bernoulli's theorem 3) Conservation of angular

Momentum 4) Principle of continuity

7. A horizontal pipe of non-uniform cross section allows water

to flow through it with a velocity 11ms when pressure is 50

kPa at a point. If the velocity of flow has to be 12ms at some

other point, the pressure at that point should be

1) 51.5 kPa 2) 100 kPa 3) 48.5 kPa 4) 24.25 kPa

Page 90: 8 & 9_MPC_SHP_E1 & E2& E4 & E5

CLASS-IX_SUBGROUP_E1 TO E3 SIMPLE HOLIDAY PACKAGE

NARAYANA GROUP OF SCHOOLS Page 16 of 40

8. A pipe is running full of water. At a point A it tapers from

400 cm2 cross sectional area to 200 cm2 at other point B. If the pressure difference between A and B is 15 cm of water column, the rate of volume flow of water through the pipe is

in litres/sec (g = 10 ms–2)

1) 20 2) 40 3) 10 4) 30

9. Water flows through a horizontal tube of variable cross section. The area of cross section at A and B are 4 mm2 and 2 mm2 respectively. If 1 cc of water enters per second

through A, find the pressure difference A BP P

1) 25 N/m2 2) 50 N/m2 3) 94 N/m2 4) 60 N/m2

10. In a horizontal pipe line of uniform cross section as pressure falls between two points separated by certain distance, the change in the kinetic energy of the oil flowing between these

two points is 0.01 J kg–1 and density of the oil is 600 kg m–

3. Then the fall in pressure between those two points is

1) 2 2Nm 2) 4 2Nm 3) 6 2Nm 4) 8 2Nm

11. The velocity distribution curve of the stream line flow of a liquid advancing through a capillary tube is

1) Circular 2) Elliptical 3) Parabolic 4) a straight line

12. A tank is completely filled with water. One of the vertical walls of the tank has pin holes from top to bottom. Which of holes will allow the water to fall at maximum distance on the floor?

1) Hole at the top 2) Hole at the bottom 3) Hole in the middle 4) Hole above middle but below top

13. There is a small hole of diameter 1 mm in the, wall of a water at a depth of 40 m below free water surface. The velocity of efflux of water from the hole will be

1) 0.14 m/s 2) 28 m/s 3) 0.28 m/s 4) 14 m/s

14. A cylindrical vessel of 90 cm height is kept filled up to the brim. It has four holes 1,2,3 and 4 which are respectively at heights of 20 cm, 30 cm, 45 cm and 50 cm from the horizontal floor PQ. The water falling at the maximum horizontal distance from the vessel comes from

Page 91: 8 & 9_MPC_SHP_E1 & E2& E4 & E5

CLASS-IX_SUBGROUP_E1 TO E3 SIMPLE HOLIDAY PACKAGE

NARAYANA GROUP OF SCHOOLS Page 17 of 40

1) Hole number 4 2) Hole number 3 3) Hole number 2 4) Hole number 1

15. Figure shows two holes in a wide tank containing a liquid

common. The water streams coming out of these holes strike the ground at the same point. The height of liquid column in the tank is

1) 10 cm 2) 8 cm 3) 9.8 cm 4) 980 cm

16. The flow of blood in a large artery of an anesthetised dog is diverted through a venture meter. The wider part of the meter has a cross-section area equal to that of the artery

28A mm . The narrower part has an area 24a mm . The

pressure drop in the artery is 24 pa. The speed of blood in

the artery is 3 31.06 10 / blood kg m .

1) 0.125 m/s 2) 125 m/s 3) 1.25 m/s 4) 12.5 m/s

17. A point object is enclosed in a glass sphere of 8 cm radius. It is situated

4 cm from centre and is viewed from the side to which it is nearer.

Where will it appear to be if of glass = 1.5 ?

1) 6 cm from the centre 2) 4 cm from the nearer surface

3) 227

cm from the nearer surface 4) 2

23

cm from the centre

18. A mark is made on the surface of a glass sphere of diameter 20 cm and refractive index 1.5. It is viewed through the glass from a portion directly opposite. The distance of the image of the mark from the centre of the sphere will be

1) 10 cm 2) 5 cm

3) 20 cm 4) 40 cm

Page 92: 8 & 9_MPC_SHP_E1 & E2& E4 & E5

CLASS-IX_SUBGROUP_E1 TO E3 SIMPLE HOLIDAY PACKAGE

NARAYANA GROUP OF SCHOOLS Page 18 of 40

19. A glass dumbell of length 30 cm and = 1.5 has ends 3 cm radius of

curvature. An object is situated in air at a distance of 9 cm from the end of dumbbell along the axis. The position of image formed due to refraction at one end only is

1) 16 cm 2) 18 cm

3) 22cm 4) 27 cm

20. A paperweight in the form of a hemisphere of radius 3.0 cm is used to hold down a printed page. An observer looks at the page vertically through the paperweight. At what height above the page will the printed letters near the centre appear to the observer

1) 2 cm 2) 1 cm 3) 0.5 cm 4) Zero

21. If 1 and 2 refractive index of the rarer and denser medium respectively.

If i and r are the angles of incidence and refraction. When refraction

from rarer to denser medium takes place, then Snell’s law is

1)

1

2

sinr

sin i

2)

2

1

sin i

sinr

3)

2

1

sinr

sin i

4) Both 1 and 2

22. A light ray, going through a prism with the angle of prism 060 , is found

to deviate by 030 . What limit on the refractive index can be put from

these data?

1) 2 2) 2

3) 3

2

4) 1

2

23. A crown glass prism with refracting angle 40, is to be achromatized for red and blue light with a flint glass prism. Angle of the flint glass prism

should be (Given Crown glass 1.513, 1.523r b Flint glass

1.645, 1.665r b )

1) 2 2) 3 3) 2.4 4) 4.5

24. When a ray of light is refracted by a prism such that the angle of deviation is minimum, then

1) the angle of emergence is equal to the angle of incidence

2) The refracted light ray must parallel to the base of the prism

3) Both 1 and 2 4) the sum of the angle of incidence and the angle of

emergence is equal to 900

Paragraph for Questions 25 to 27

A light ray is incident normally on the face AB of a right-angled prism ABC 1.50

Page 93: 8 & 9_MPC_SHP_E1 & E2& E4 & E5

CLASS-IX_SUBGROUP_E1 TO E3 SIMPLE HOLIDAY PACKAGE

NARAYANA GROUP OF SCHOOLS Page 19 of 40

as shown in figure.

f

A

CB

P

25. The largest angle for which the light ray is totally reflected at the

surface AC is

1) 1 2

cos3

2)

1 2sin

3

3) 1 3

cos2

4)

1 3sin

2

26. The angle of refraction at the surface AB is 00

1) 00 2) 045

3) 030 4) 090

27. The angle of incidence at a point P on the surface AC is

1) 2) 090

3) 0180 4) Zero

Paragraph for Questions 28 to 30

Dispersive power of the material of a prism is the ratio of the angular dispersion of two extreme colours to their mean deviation.

28. In a prism, the refractive indices of different colours are

V R1.6, 1.52, 1.56 . The dispersive power of the prism is

1) 1/56 2) 1/8

3) 1/7 4) infinite

29. The angles of minimum deviations are 53° and 51° for blue and red colours respectively produced in an equilateral glass prism. The dispersive power is

1) 51/26 2) 1/26

3) 1/52 4) 1/51

30. Dispersive power of the material of a prism is 0.0221. If the deviation produced by it for yellow color is 38°, then the angular dispersion

Page 94: 8 & 9_MPC_SHP_E1 & E2& E4 & E5

CLASS-IX_SUBGROUP_E1 TO E3 SIMPLE HOLIDAY PACKAGE

NARAYANA GROUP OF SCHOOLS Page 20 of 40

between red and violet colors is

1) 0.65° 2) 0.84°

3) 0.48° 4) 1.26°

31. If air is blown with a straw under of the pans of a physical balance present in equilibrium position, then that pan.

1) rises up 2) remains in the same position 3) lowers down 4) rises or lowers depending upon

the velocity of air blown. 32. Water enters a horizontal pipe of non uniform cross section

with a velocity of 0.3 ms–1 and leaves the other end with a

velocity of 0.6 ms–1. Pressure of water at the first end is

1500 Nm–2, then pressure at the other end is

1) 1000 2Nm 2) 1635 2Nm 3) 1400 2Nm 4) 1365 2Nm

33. A convex lens forms a real image three times larger than the object on a screen. Object and screen are moved until the image becomes twice the size of the object. If the shift of the object is 6 cm. the shift of screen is

1) 36 cm 2) 72 cm 3) 18 cm 4) 9 cm

34. An illuminated wire gauge is kept at a distance of 10cm from a convex lens. An image is obtained on a screen kept at a distance of 10 cm from the lens. The focal length of the convex lens is

1) 5 cm 2) -5 cm 3) -20 cm 4) 20 cm

35. An object is placed first at infinity and then at 20 cm from the

object-side focal plane of a convex lens. The two images thus

formed are 5 cm apart. The focal length of the lens is

1) 5 cm 2) 10 cm

3) 15 cm 4) 20 cm

36. When the object is placed in between the Infinity and the Optical centre of the Position of the image is the concave lens.

1) Between focus and optical centre

2) Between 1 1&2F F

3) Beyend 12F 4) At infinity

37. The graph shows the variation of magnification m produced by

convex lens with image distance v. The power of the lens is :

Page 95: 8 & 9_MPC_SHP_E1 & E2& E4 & E5

CLASS-IX_SUBGROUP_E1 TO E3 SIMPLE HOLIDAY PACKAGE

NARAYANA GROUP OF SCHOOLS Page 21 of 40

1) b/c 2) b/ca

3) bc/a 4) c/b

38. An object is situated at a distance of 20 cm from a convex lens of focal length 30 cm. The position of the image formed by it will be

1) - 30 cm 2) 30 cm 3) -60 cm 4) + 60cm

39. The distance between two point sources of light is 24 cm and a

converging lens is kept in between two sources. The object

distances of two sources from a converging lens of focal length

of 9 cm, so that the image distances of two sources are equal

1) 6 cm 2) 18 cm

3) 18 cm or 6 cm 4) 24 cm

40. The plano-convex lenses of focal length 80 cm are placed with

their plane surfaces in contact to form an ordinary convex lens.

The distance of an object from this lens in order to obtain its

real, inverted image of unit magnification will be

1) 20 cm 2) 80 cm

3) 40 cm 4) 160 cm

41. A luminous object is placed at a distance of 30 cm from the convex lens of focal length 20 cm. On the other side of the lens, the distance from the lens a convex mirror of radius of curvature 10 cm be placed so that the real images formed on the source itself is

1) 12 cm 2) 30 cm

3) 50 cm 4) 60 cm

42. To get real magnified image in convex lens, the object position is

Page 96: 8 & 9_MPC_SHP_E1 & E2& E4 & E5

CLASS-IX_SUBGROUP_E1 TO E3 SIMPLE HOLIDAY PACKAGE

NARAYANA GROUP OF SCHOOLS Page 22 of 40

1) in between lens and F 2) in between F and 2F

3) at 2F 4) beyond 2F

43. An object is placed at a distance 20 cm from the convex lens of focal length 15 cm. the position of the image is

1) 60

7cm

2) -60 cm

3) 60 cm 4) 60

7cm

44. An object is situated at a distance of 60 cm from a convex lens of focal length of 30 cm The position of the image formed by the lens is

1) 20 cm 2) -20 cm 3) 60 cm 4) -60 cm

45. A pin of length 2.00 cm is placed perpendicular to the principal

axis of a converging lens. An inverted image of size 1.00 cm is

formed at a distance of 40.0 cm from the pin. the focal length of

the lens is

1) 8.89 cm 2) 8.39 cm

3) 26.7 cm 4) 8.55 cm

46. A convex lens produces a double size real image when an object

is placed at a distance of 18 cm from it. Where should the object

be placed to produce a tripe size real image?

1) 16 cm 2) 18 cm

3) 10 cm 4) 21 cm

47. An object is placed at a distance of 20 cm from the convex lens

of focal length 15 cm. the magnification produced by the lens is

1) 3 2) -3

3) 1

3

4) 1

3

48. A 2.0 cm tall object is placed perpendicular to the principal axis

of a convex lens of focal length 10 cm. The distance of the

object from the lens is 15 cm. the magnification is

1) 2 2) -2

3) 1

2

4) 1

2

49. A ray of light enters a diamond from air. If the refractive index of

diamond is 2.42, by what per cent does the speed of light reduce

on entering the diamond?

1) 41 % 2) 59 %

3) 2.4 % 4) 100 %

Page 97: 8 & 9_MPC_SHP_E1 & E2& E4 & E5

CLASS-IX_SUBGROUP_E1 TO E3 SIMPLE HOLIDAY PACKAGE

NARAYANA GROUP OF SCHOOLS Page 23 of 40

50. A ray of light travelling in air falls on the surface of a transparent slab. The ray makes an angle of 450 with the normal to the surface. Find the angle made by the refracted ray with the normal within the slab.

Refractive index of the material of the slab 2

1) 300 2) 600

3) 450 4) 900

51. A ray of light traveling in air falls obliquely on the surface of a calm pond. It will

1) go into the water without deviating from its path

2) Deviate away from the normal

3) Deviate towards the normal 4) Turn back on its original path

52. A ray of light goes from a medium of refractive index 1 to a medium of

refractive index 2 . The angle of incidence is i and the angle of

refraction is r. Then, sin i/ sin r is equal to

1) 1 2)

2

3) 1 2/ 4)

2 1/

53. A ray of light travelling in a transparent medium falls on a surface

separating the medium from air at an angle of incidence045 . The ray

undergoes total internal reflection. If is the refractive index of the

medium with respect to air, select the possible value(s) of from the

following

1) 1.3 2) 1.4 3) 1.5 4) 1.2

54. The reflected and refracted rays are observed to be perpendicular to

each other, when ray of light is incident at an angle of 060 on a

transparent block. The refractive index of that block is

1) 3

2

2) 1

2

3) 2

3

4) 3

55. The critical angle for light going from a medium in which wave length is

4000 A to medium in which its wavelength is 6000A . The refractive index of medium is

1) 2

3

2) 1

3) 3

2

4) 1 2sin3

56. Time taken by sun light to pass through a window of thickness 2 mm

with refractive index 1.5 is [speed of light in vacuum = 3 x 108 m/s]

1) 2 x 10-3s 2) 2 x 105s 3) 2 x 10-11s 4) 1 x 10-11s

Paragraph for Questions 57 to 59

Page 98: 8 & 9_MPC_SHP_E1 & E2& E4 & E5

CLASS-IX_SUBGROUP_E1 TO E3 SIMPLE HOLIDAY PACKAGE

NARAYANA GROUP OF SCHOOLS Page 24 of 40

Snell’s law 2 1

1 2

sin

sin

vi

r v

57. The angle a made by the light ray when it gets refracted from water to

air, as shown in the figure

1) 1 4

sin5

a

2) 1 5

sin4

a

3) 1 3

sin5

a

4) 1 5

sin3

a

58. Find the speed of light in medium ‘a’ if speed of light in medium ‘b’ is c/3, where c=speed of light in vacuum and light refracts from medium ‘a’ to medium ‘b’ making 450 and 600, respectively, with the normal.

1) 2

3 3

c

2) 2

3c

3) 1

3 3

c

4) 2

2 3

c

59. A ray of light is incident on a transparent glass slab of refractive index

3 . If the reflected and reflected rays are mutually perpendicular, what

is the angle of incidence?

1) 600 2) 300

3) 450 4) 900

Paragraph for Questions 60 to 62

Relation between Refractive index and Critical angle:

r

dSin 90 1

Sin C Sin C

60. The refractive index of glass is 1.5, The critical angle for glass is

Page 99: 8 & 9_MPC_SHP_E1 & E2& E4 & E5

CLASS-IX_SUBGROUP_E1 TO E3 SIMPLE HOLIDAY PACKAGE

NARAYANA GROUP OF SCHOOLS Page 25 of 40

1) 1sin 0.67c 2) 1sin 1.5c

3) 1cos 1.5c 4) 1cos 0.67c

61. The refractive index of glass with respect to air is a g and refractive

index of water with respect to air a w , then refractive index of water

with respect to glass is

1) a g

a w

2)

a w

a g

3) 1

w g

4) 11

w g

62. A ray of light passing through a certain medium meets the surface

separating the medium from air at 045 and is just not refracted. The

refractive index of the medium is

1) 1.414 2) 0.707

3) 2.428 4) 2.121

63. An object is placed at a distance of 20 cm from a concave lens of focal length 15 cm. The line as magnification produced by the lens is

1) 3

7

2) -3

3) 3 4) 3

7

64. An object 4 cm long is placed at a distance of 20 cm from a concave lens of focal length 15 cm, The position of image is

1) 60

7cm

2) -60 cm

3) 60 cm 4) 60

7cm

65. The angle of deviation when light is incident at an angle of 045 on one

of the refracting faces of an equilateral prism of refractive index 1.414 is

1) 40 2) 30

3) 45 4) 50

66. The number of surfaces bounding a prism is

1) 3 2) 4 3) 5 4) 6

67. A ray of light takes grazing incidence on the first face of a prism and the emergent ray is normal to the 2nd face of the prism. If ‘A’ is angle of prism then the angle of the deviation ‘D’ is equal to

1) 090 2A 2) 090 -A

Page 100: 8 & 9_MPC_SHP_E1 & E2& E4 & E5

CLASS-IX_SUBGROUP_E1 TO E3 SIMPLE HOLIDAY PACKAGE

NARAYANA GROUP OF SCHOOLS Page 26 of 40

3) 0902

A

4) 0180 2A

68. A ray of light is incident normally on one of the faces of a prism of apex

angle 30° and refractive index 2 .The angle of deviation of the ray is ........... degree when it emerges from the second surface of the prism

1) 30° 2) 60°

3) 15° 4) 90° 69.

A thin prism P1 of angle 03 and refractive index 1.54 is combined with

another thin prism P2 of refractive index 1.72 to produce dispersion without deviation. The angle of P2 is

1) 02.25 2) 80

3) 2.6 4) 4.45

70. If the angle of a prism is 80° and angle of minimum deviation is 40°,

then the angle of refraction in the prism will be

1) 30° 2) 40°

3) 50° 4) 20°

71. A crown glass prism with refracting angle 40, is to be achromatized for

red and blue light with a flint glass prism. Angle of the flint glass prism

should be (Given Crown glass 1.513, 1.523r b Flint glass

1.645, 1.665r b )

1) 2 2) 3

3) 4.2 4) 4.5

72.

A given ray of light suffers minimum deviation in an equilateral prism P. Additional prisms Q and R of identical shape and of the same material as P are now added as shown in the figure. The ray will suffer

1) greater deviation 2) no deviation

3) same deviation as before 4) total internal reflection

73.

A deviation in the path of a ray of light can be produced

1) By a glass prism but not by 2) By a rectangular glass slab

Page 101: 8 & 9_MPC_SHP_E1 & E2& E4 & E5

CLASS-IX_SUBGROUP_E1 TO E3 SIMPLE HOLIDAY PACKAGE

NARAYANA GROUP OF SCHOOLS Page 27 of 40

a rectangular glass slab but not by a glass prism

3) By a glass as well as rectangular glass slab

4) Neither by a glass prism nor by a rectangular glass slab

74.

The angle of minimum deviation of the prism is 060 , then the

refractive index of the substance of an equilateral prism is

1) 3 2) 1

2

3) 1

3

4) 2

75. The prism is surrounded by a medium of refractive index 1 . If the

refractive index index of the material of the prism is 2 , then

1)

2

1

sin2

sin2

mA

A

2)

1

2

sin2

sin2

mA

A

3)

1

sin2

sin2

mA

A

4)

2

sin2

sin2

mA

A

76. The ratio of the angular dispersion of two extreme colours to

their mean deviation is

1) Dispersive power 2) Dispersion

3) Angular dispersion 4) Angular deviation

77. A ray of light incident normally on the first face of a prism of

refracting angle ‘A’ and critical angle ‘C’. The ray emerges from

the second face of the prism when the following condition satisfies

1) C>A 2) C<A

3) C A 4) C A

78. If the critical angle for the medium of a prism is C and the angle of

the prism is A, then there will be no emergent ray when

Page 102: 8 & 9_MPC_SHP_E1 & E2& E4 & E5

CLASS-IX_SUBGROUP_E1 TO E3 SIMPLE HOLIDAY PACKAGE

NARAYANA GROUP OF SCHOOLS Page 28 of 40

1) A<2C 2) A= 2C

3) A>2C 4) A=C/2

79. An equilateral prism is placed on a horizontal surface. A ray PQ is

incident onto it. For minimum deviation

1) PQ is horizontal 2) QR is horizontal

3) RS is horizontal 4) Any one will be horizontal

80. A ray of light incident at an angle ' ' on a refractive face of a

prism emerges from the other face normally. If the angle of the

prism is 05 and the prism is made of a

material of refractive index 1.5 the angle of incidence is

1) 07.5 2) 05

3) 01.5 4) 02.5

IX_PHYSICS_SHP_E1 TO E3_KEY

Q.NO. 1 2 3 4 5 6 7 8 9 10

KEY 1 1 2 3 1 2 3 2 3 3

Q.NO. 11 12 13 14 15 16 17 18 19 20

KEY 3 2 2 2 1 1 3 4 4 4

Q.NO. 21 22 23 24 25 26 27 28 29 30

KEY 4 1 1 3 1 1 2 3 2 2

Q.NO. 31 32 33 34 35 36 37 38 39 40

KEY 3 4 1 1 2 1 1 3 3 2

Q.NO. 41 42 43 44 45 46 47 48 49 50

Page 103: 8 & 9_MPC_SHP_E1 & E2& E4 & E5

CLASS-IX_SUBGROUP_E1 TO E3 SIMPLE HOLIDAY PACKAGE

NARAYANA GROUP OF SCHOOLS Page 29 of 40

KEY 3 2 3 3 1 1 2 2 2 1

Q.NO. 51 52 53 54 55 56 57 58 59 60

KEY 3 4 3 4 3 4 1 1 1 1

Q.NO. 61 62 63 64 65 66 67 68 69 70

KEY 2 1 1 1 2 3 2 3 1 2

Q.NO. 71 72 73 74 75 76 77 78 79 80

KEY 1 3 1 1 1 1 3 3 2 1

CHEMISTRY( Q.NO. 1 TO 80)

1. The value of (Kp / Kc) for the reversible reaction SO2(g) + 1/2

O2(g) SO3(g) at constant temperature T is:

1)(RT)1/2 2)RT 3) 1/2( )RT 4)1/RT

2. The knowledge of equilibrium constant helps us in 1) Predicting the extent of a reaction 2) Predicting the direction in which the net reaction is taking place 3) Calculation of equilibrium concentration 4) All the above 3. If Qc is less than Kc the net reaction will occur in__________

1)Backward direction 2)Forward direction 3)Remain unchanged 4)The reaction stops 4. If the value of K is large, then 1) The reactants are more stable 2) The products are more stable 3) The reactants and products both are equally stable. 4) Both reactants and products are unstable. 5. The equilibrium constant expression for a gaseous reaction is .

4 5

3 2

C 4 6

2

NH OK =

NO H OWrite the balanced chemical equation

corresponding to this expression? 1) 4NO + 6H2O 4NH3 + 5O2 2)2NO + 6H2O 2NH3 + 3O2

3)4NO + 3H2O 4NH3 + 2O2 4)2NO + 3H2O 4NH3 + 3O2 6. The equilibrium constant, Kp for the reaction 2SO2(g) + O2

2SO3(g) is 4.0 atm–1 at 1000 K. What would be the partial

pressure of O2 if at equilibrium the amount of SO2 and SO3 is the

same?

Page 104: 8 & 9_MPC_SHP_E1 & E2& E4 & E5

CLASS-IX_SUBGROUP_E1 TO E3 SIMPLE HOLIDAY PACKAGE

NARAYANA GROUP OF SCHOOLS Page 30 of 40

1)0.50 atm 2)0.25 atm 3)1 atm 4)0.75 atm

7. For the reaction, at 800 K N2(g) + 3H2(g) 2NH3(g) the ratio of

Kp and Kc is: (R = 0.0821 litre.atm.mol–1.K–

1)2.3 × 10–4 2)3.2 × 10–6 3)2.3 × 104 4)3.2 × 106

8. A reversible reaction A C takes place in two steps A B; B

C, which are also equilibrium steps. If the equilibrium constants

of the two steps are K1& K2 respectively, the overall equilibrium

constant K is equal to

1)K1 / K2 2)K2 / K1 3)K1 x K2 4)K1 - K2

9. In an equilibrium reaction for which , 0G 0 the equilibrium

constant K should be equal to

1)1 2)<1 3)>1 4) undefined

10. The equilibrium constant kc of a reversible reaction is 5. The rate

constant for the reverse reaction is 2.8. What is the rate constant

for the forward reaction

1) 14 2) 28

3) 0.028 4) 280

11. Which of the reactions will be formed when the standard free

energy change of the reaction is greater than unity

0G 1

1)Forward reaction 2)Backward reaction

3)Both will occur with the same speed 4)Reaction will stop

12. When ºG 0 then

1)Reaction is at equilibrium 2)Not at equilibrium

3)Forward reaction is feasible 4)Reverse reaction is Feasible

13. The relation between standard free energy change of a reaction

and its equilibrium constant is given by

1) 0G c2.303RT K 2)

0

c

2.303G log

RT

3) 0

cG 2.303RT log K 4) 0 2

cG 2.303RT K

14. Kc for the reaction 2(g ) 2 g 3 gN 3H 2NH at 773 K is 26.0 10 . Kp

for the above reaction will be: (R = 0.0821)

Page 105: 8 & 9_MPC_SHP_E1 & E2& E4 & E5

CLASS-IX_SUBGROUP_E1 TO E3 SIMPLE HOLIDAY PACKAGE

NARAYANA GROUP OF SCHOOLS Page 31 of 40

1)1.5 × 10–5 2)4 × 10–6 3)6 × 10–6 4) 8 × 10–6 15. The equilibrium constant, Kp for the reaction

2 24 g 2 g 6 gC H H C H is 17 15.04 10 atm at 25ºC. Calculate Gº .

1)+101 kJ 2) 101kJ 3) 1.01kJ 4)1.01kJ 16. For A B C Dthen the reaction quotient Q =

1) A B

C D 2)

C D

A B 3)

A B

C D 4)

A D

C B

17. Acidity of BF3 can be explained on the basis of which of the

following concepts?

1)Arrhenius concept

2)Bronsted Lowry concept

3)Lewis concept

4)Bronsted Lowry as well as Lewis concept.

18. In which of the following reactions, the equilibrium remains

unaffected on addition of small amount of argon at constant

volume?

1) 2 2H g I g 2HI g

2) 5 3 2PCl g PCl g Cl g

3) 2 2 3N g 3H g 2NH g

4)The equilibrium will remain unaffected in all the three cases.

19. Conjugate acid of a weak base is always stronger.

What will be the decreasing order of basic strength of the

following conjugate bases? - - - -

3OH , RO , CH COO , Cl

1) - - - -

3OH > RO > CH COO > Cl 2) - - - -

3CH COO >RO > OH > Cl

3) - - - -

3RO > OH > CH COO > Cl 4) - - - -Cl > RO > OH > CH3COO

20. In the reaction SnCl2 + 2Cl-SnCl4 +2e- the Lewis acid is

1)SnCl 2)SnCl3 3)SnCl2 4) 4SnCl

21. Which among the following reactions will be favoured at low

pressure?

1) 2 2N g O g 2NO g 2) 2 2H g I g 2HI g

3) 5 3 2PCl g PCl g Cl g 4) 2 2 3N g 3H g 2NH g

22. In a reaction 5 3 2AX (g) AX (g) X (g) ;H > 0, the dissociation of

AX5(g) will be favored at:

Page 106: 8 & 9_MPC_SHP_E1 & E2& E4 & E5

CLASS-IX_SUBGROUP_E1 TO E3 SIMPLE HOLIDAY PACKAGE

NARAYANA GROUP OF SCHOOLS Page 32 of 40

1)Low temperature and high pressure

2)High temperature and low pressure

3)Low temperature and low pressure

4)High temperature and high pressure

23. On increasing the pressure, in which direction will the gas phase

reaction proceed to re-establish equilibrium, is predicted by

applying the Le Chatelier’s Principle. Consider the reaction.

N2 (g) + 3H2 (g) 2NH3 (g)

Which of the following is correct, if the total pressure at which

the equilibrium is established, is increased without changing the

temperature?

1)Kwill remain same

2)Kwill decrease

3)Kwill increase

4)Kwill increase initially and decrease when pressure is very high

24. Conjugate base of a acid is obtained by

1)Addition of proton to a base

2)Removal of proton from the acid

3)Addition of

OH ion to the base

4)Removal of proton from the base

Paragraph for Questions 25 to 27

In the reaction, 2 3HC H O H O C , HCl and H3O+ are Bronsted –

Lowry acids.

25. Review the equilibrium and choose the correct statement 4 2 3 4HC O H O H O C O

1) 4HC O is the conjugate acid of 2H O

2) 3H O is the conjugate base of 2H O

3) 2H O is the conjugate acid of 3H O

4) 4C O is the conjugate base of 4HC O

26. 2 3HC H O H O C .

In the above reaction which acts as Bronsted base?

Page 107: 8 & 9_MPC_SHP_E1 & E2& E4 & E5

CLASS-IX_SUBGROUP_E1 TO E3 SIMPLE HOLIDAY PACKAGE

NARAYANA GROUP OF SCHOOLS Page 33 of 40

1)HCl 2) 3H O 3) 2H O 4)None

27. The conjugate base of 2NH is :

1) 3NH 2) 2NH 3) 4NH 4)

3N

Paragraph for Questions 28 to 30

Physical and chemical equilibrium can respond to a change in their

pressure, temperature, and concentration of reactants and products.

To describe the change in the equilibrium we have a principal named

Le Chatelier's principle. According to this principle, even if we make

some changes in equilibrium, then also the system even re-establishes

the equilibrium by undoing the effect.

28. Consider the following equilibrium:

2 2 32 2 ; ,NO O NO H ve

If 2O is added and volume of the reaction vessel is reduced, the

equilibrium

1)Shifts in the product side 2)Shifts in the reactant side

3)Cannot be predicted 4)Remains unchanged

29. If We add 2

4SO ion to a saturated solution of 2 4Ag SO , it will result

in a/an

1)Increase in Ag concentration.

2)Decrease in Ag concentration

3)It will shift Ag ions from solid into solution.

4)It will decrease the 2

4SO ion concentration in the solution

30. In the reaction. If we increase the pressure of the system, the

equilibrium is

1)Shifts in the product side 2)Shift in reactant side

3)Remains unchanged 4)Cannot be predicted

31. For the reaction 2(g ) 2 g 3 gN 3H 2NH , the units of Kc are:

1) mole / litre 2)litre / mol 3) 2 2lit mol 4) lit2mol2

32. The equilibrium constant for H2+I2 2HI is 81 then the

equilibrium constant for 1

2 H2+

1

2I2 HI is_________

1)6 2)7 3)9 4)12

33. Which of the following are correct :

Page 108: 8 & 9_MPC_SHP_E1 & E2& E4 & E5

CLASS-IX_SUBGROUP_E1 TO E3 SIMPLE HOLIDAY PACKAGE

NARAYANA GROUP OF SCHOOLS Page 34 of 40

1)pH of a weak acid is = a

1 1PK log C

2 2

2)pH of a weak base is = w b

1 1PK PK logC

2 2

3)pH of a weak base is = b

114 PK logC

2

4)All the above

34. Find the pH of a 0.002 N acetic acid solution, it is 3.3% ionised at a given dilution. [log 6.6 = 081]

1)4.34 2)5.33 3)4.18 4)3.33

35. A 0.01 M ammonia solution is 4 % ionized, the concentration of

OH– ions is

1)0.005 M 2)0.0004 M 3)0.0005 M 4)0.05 M

36. A weak mono acidic base is 7% ionized in 0.01 M. solution. The Hydroxide ion concentration in the solution is

1)7 x 10-4 2)5 x 10-4 3)5 x 10-10 4)2 x 10-11

37. The pH of neutral water at 25°C is 7.0. As the temperature increases, ionization of water increases, however, the concentration of H+ ions and OH– ions are equal. What will be the pH of pure water at 60°C?

1)Equal to 7.0 2)Greater than 7.03)Less than 7.0 4)Equal to zero

38. 0.4g of NaOH is present in one litre of the solution shows that H+ concentration of the solution is

1)10-2 2)10-4 3)10-10 4)10-12

39. One litre of water contains 710 moles of H ions .Degree of ionisation of water ( in percentage) is

1)7

1.8 10 2)

91.8 10

3)7

3.6 10 4)

93.6 10

40. At 700C the concentration of H+ ion in aqueous solution of NaCl

is 10-6 mole/lit. The OH- ion concentration is

1)10-8 moles/lit. 2)10-6moles/lit. 3)10-7 moles/lit. 4)10-9moles/lit.

41. A monobasic acid solution has pH value of 5. Its molarity is 0.0005M. The degree of ionisation of the acid is

1)5 x 10-3 2)2x10-3 3)5 x10-2 4)2x10-2

Page 109: 8 & 9_MPC_SHP_E1 & E2& E4 & E5

CLASS-IX_SUBGROUP_E1 TO E3 SIMPLE HOLIDAY PACKAGE

NARAYANA GROUP OF SCHOOLS Page 35 of 40

42. Determine the degree of ionisation of 0.05M NH3 at 250C in a

solution of pH=11 (Kb=1.76 x 10-5)

1)0.0173 2)0.173 3)1.73 4)17.3

43. The ionisation constant of a mono basic acid is 6 x 10-2. The pH of 0.01 M - acid solution is [log 2.4 = 0.38]

1)1.61 2)3.30 3)5.0 4)1.65

44. The pH of a 0.01M aqueous solution of sodium hydroxide will be.

1)12 2)7.5 3)9.0 4)11.0

45. Which of the following aqueous solutions of compounds are acidic in nature :

1) 2 4K SO 2) 3FeC 3) 4CuSO 4)Both 2 and 3

46. The dissociation constants of two acids HA1 and HA2 are 4.5 x

10-4 and 1.8 x 10-5 respectively. If both are having equal concentrations the relative strength of acids is

1)5:2 2)2:5 3)5:1 4)1:5

47. Degree of dissociation of CH3COOH and 4NH OHare the same. If

0.01 M solution of CH3COOH has pH=4.0; then pH of 0.01 M

NH4OH will be

1)4 2)7 3)10 4)14

48. pH of a sample of KOH and another of NaOH are 9 and 12

respectively. Their normalities are related as NaOH KOHN xN . What

is the value of x?

1)5/6 2)6/5 3) 210 4) 310

49. 50ml of 0.1M solution of sodium acetate and 50 ml of 0.01M

acetic acid mixed. The pKa of acetic acid is 5.76. The HP of the

buffer solution is

1)6.76 2)4.76 3)5.76 4)9.24

50. Which of the following will produce a buffer solution when mixed

in equal volumes?

1)0.1 moldm–3 NH4OH and 0.1 moldm–3HCl

2)0.05 moldm–3 NH4OH and 0.1 moldm–3HCl

3)0.1 moldm–3 NH4OH and 0.05 moldm–3 HCl

4)0.1 moldm–3 CH4COONa and 0.1 moldm–3NaOH

51. Conjugate acid of a weak base is always stronger.

What will be the decreasing order of basic strength of the

followingconjugate bases?

- - - -

3OH , RO , CH COO , Cl

Page 110: 8 & 9_MPC_SHP_E1 & E2& E4 & E5

CLASS-IX_SUBGROUP_E1 TO E3 SIMPLE HOLIDAY PACKAGE

NARAYANA GROUP OF SCHOOLS Page 36 of 40

1) - - - -

3OH > RO > CH COO > Cl 2) - - - -

3CH COO >RO > OH > Cl

3) - - - -

3RO > OH > CH COO > Cl 4) - - - -Cl > RO > OH > CH3COO

52. For the reaction, at 800 K N2(g) + 3H2(g) 2NH3(g) the ratio

of Kp and Kc is: (R = 0.0821 litre.atm.mol–1.K–1)

1)2.3 × 10–4 2)3.2 × 10–6 3)2.3 × 104 4)3.2 × 106

53. The equilibrium constant for the reaction

2 2A g +B g 2AB g is 48 at 600 K What is the equilibrium

constant for 2 22AB g A g +B g

1)1.56×10–2 2)2.08×10–2 3)1.56×102 4)2.56×102

54. If the reversible reaction

SO2(g) + NO2(g) SO3(g) + NO(g) - (1)

takes place in two reversible steps (2 & 3), with equilibrium constant values 4.0 and 0.45 respectively

SO2(g) + 2 3

1( ) ( )

2O g SO g K = 4.0 - (2)

2 2

1( ) ( ) ( )

2NO g NO g O g K = 0.45 - (3)

the equilibrium constant Kc of the reaction (1) is

1)0.9 2)400 / 9 3)9 / 400 4)1.8

55. Find the pH of a 0.002 N acetic acid solution, it is 3.3% ionised at

a given dilution. [log 6.6 = 081]

1)4.34 2)5.33 3)4.18 4)3.33

56. A monobasic acid solution has pH value of 5. Its molarity is

0.0005M. The degree of ionisation of the acid is

1)5 x 10-3 2)2x10-3 3)5 x10-2 4)2x10-2

Paragraph for Questions 57 to 59

Page 111: 8 & 9_MPC_SHP_E1 & E2& E4 & E5

CLASS-IX_SUBGROUP_E1 TO E3 SIMPLE HOLIDAY PACKAGE

NARAYANA GROUP OF SCHOOLS Page 37 of 40

Many reactions, particularly the biochemical reactions are to be carried

out at a constant pH. But it is observed that solutions and even pure

water (pH = 7) cannot retain the constant pH for long.

57. Which of the following statements are correct for about buffer solution :

1)It has a definite pH

2)Its pH does not change on dilution.

3)It’s pH does not change on standing for long time. 4)All the above 58. Solutions with reserve acidity and alkalinity are called :

1)Buffer solution 2)True solution

3)Isohydric solution 4)Ideal solution

59. A buffer solution is used in : 1)Preparation of potash alum 2)The removal of phosphate ions

3)Increasing the pH value of a solution

4)Precipitation of 3

Cr OH from 3CrC .l

Paragraph for Questions 60 to 62

In the reaction, 2 3HC H O H O C , HCl and H3O+ are Bronsted -

Lowry acids.

60. Review the equilibrium and choose the correct statement 4 2 3 4HC O H O H O C O

1) 4HC O is the conjugate acid of 2H O

2) 3H O is the conjugate base of 2H O

3) 2H O is the conjugate acid of 3H O

4) 4C O is the conjugate base of 4HC O

61. 2 3HC H O H O C .

In the above reaction which acts as Bronsted base?

1)HCl 2) 3H O 3) 2H O 4)None

62. The conjugate base of 2NH is :

1) 3NH 2) 2NH 3) 4NH 4)

3N

63. 1 2 3a a aK ,K and K are the respective ionisation constants for the

following Reactions.

Page 112: 8 & 9_MPC_SHP_E1 & E2& E4 & E5

CLASS-IX_SUBGROUP_E1 TO E3 SIMPLE HOLIDAY PACKAGE

NARAYANA GROUP OF SCHOOLS Page 38 of 40

2H S H HS

2HS H S

2

2H S 2H S

The correct relationship between 1 2 3a a aK ,K and K is

1)3 1 2a a aK K K 2)

3 1 2a a aK K K

3)3 1 2a a aK K K

4)

3 1 2a a aK K /K

64. The ionisation constant of an acid, Ka, is the measure of strength of an acid. The Ka values of acetic acid, hypochlorous acid and formic acid are 1.74 × 10–5, 3.0 × 10–8 and 1.8 × 10–4 respectively. Which of the following Orders of pH of 0.1 moldm–3 solutions of these acids are correct?

1) Acetic acid >hypochlorous acid > formic acid

2) Hypochlorous acid > acetic acid > formic acid

3) Formic acid >hypochlorous acid > acetic acid

4) Formic acid > acetic acid >hypochlorous acid

65. hK of salt obtained from strong acid and weak base is

55 10 . The bK of weak base is

1) 192 10 2) 105 10

3) 102 10 4) 95 10

66. pH of 0.01M HS- will be

1) log7

2

apK C

pH 2) log

72

apK C

pH

3) 1 272

pK pK

pH 4)

72

a bpK pK

pH

67. The degree of hydrolysis of a salt of weak acid and weak base in its 0.1M solution is found to be 25%. If the molarity of the solution is 0.2M, the percentage hydrolysis of the salt sholud be

1) 100% 2) 50% 3) 25% 4) 80%

68. From separate solutions of four sodium salts NaW, NaX, NaY and NaZ had pH 7.0, 9.0, 10.0 and 11.0 respectively when each solution was 0.1M, the weakest acid is

1) HW 2) HX 3) HY 4) HZ

69. Nature of 0.1M solution of potassium sulphate is

1) Acidic 2) Alkaline 3) Neutral 4) Amphoteric

70. The no.of hydroxyl ions produced by one molecule of

2 3Na CO on hydrolysis is

1) 4 2) 2

Page 113: 8 & 9_MPC_SHP_E1 & E2& E4 & E5

CLASS-IX_SUBGROUP_E1 TO E3 SIMPLE HOLIDAY PACKAGE

NARAYANA GROUP OF SCHOOLS Page 39 of 40

3) 3 4) 0

71. MX is the salt of weak base, MOH and weak acid, HX .

Aqueous solution of MX is

1) Acidic, if a bK K 2) Basic, if a bK K

3) Neutral, if a bK K

4) All the above

72. The HP of 0.1M solution of the following compounds increase in the order

1) 3 2 2 4 KNO K S K SO 2)

2 2 4 3 K S K SO KNO

3) 2 4 3 2 K SO KNO K S 4)

3 2 4 2 KNO K SO K S

73. Identify the salt of a strong acid and weak base. Among the following?

1) 3FeCl 2) 3CH COONa

3) 3 4CH COONH 4) NaBr

74. Which of the following salt solution have HP greater than 7?

1) NH4Cl 2) NaCl 3) CH3COONa 4) Na2SO4

75. The aqueous solution of sodium cyanide is basic in nature. This is due to the hydrolysis of

1) Sodium ion 2) Cyanide ion 3) Cyanide ion and sodium

ion 4) Iso cyanide ion

76. pH of aqueous NaHSO4 solution is

1) >7 2) 7 3) <7 4) 14

77. A salt of weak acid and weak base undergoes

1) Only cationic hydrolysis 2) Only anionic hydrolysis 3) Both cationic and

anionic hydrolysis 4) neither cationic nor

anionic hydrolysis

78. Which of the following is basic solution?

1) NaCl 2) NaNO2 3) NH4NO3 4) KBr

79. The pH of the salt of strong acid and weak base is given by

1) 1log

2 w apK pK C

2) 1log

2 w bpK pK C

3) 1

2 w a bpK pK pK

4) 1[ ]2

wpK

80. When 20 mL of M/20 NaOH is added to 10 mL of M/10 HCl, the resulting solution will

Page 114: 8 & 9_MPC_SHP_E1 & E2& E4 & E5

CLASS-IX_SUBGROUP_E1 TO E3 SIMPLE HOLIDAY PACKAGE

NARAYANA GROUP OF SCHOOLS Page 40 of 40

1) Turn blue litmus red 2) turn phenolpthalein solution pink

3) Turns methyl orange red 4) Will have no effect on either red or blue litmus

IX_CHEMISTRY_SHP_E1 TO E3_KEY

Q.NO. 1 2 3 4 5 6 7 8 9 10

KEY 3 4 2 2 1 2 1 3 1 1

Q.NO. 11 12 13 14 15 16 17 18 19 20

KEY 2 1 3 1 2 2 3 4 3 3

Q.NO. 21 22 23 24 25 26 27 28 29 30

KEY 3 2 1 2 4 3 2 1 2 4

Q.NO. 31 32 33 34 35 36 37 38 39 40

KEY 3 3 4 3 2 1 3 4 1 2

Q.NO. 41 42 43 44 45 46 47 48 49 50

KEY 4 1 1 1 4 3 3 4 1 3

Q.NO. 51 52 53 54 55 56 57 58 59 60

KEY 3 1 2 4 3 4 4 1 2 4

Q.NO. 61 62 63 64 65 66 67 68 69 70

KEY 3 2 1 4 3 1 3 4 3 2

Q.NO. 71 72 73 74 75 76 77 78 79 80

KEY 4 3 1 3 2 3 3 2 2 4

Page 115: 8 & 9_MPC_SHP_E1 & E2& E4 & E5

CLASS-IX_SUBGROUP_E4 & E5 SIMPLE HOLIDAY PACKAGE

NARAYANA GROUP OF SCHOOLS Page 1 of 45

(OLYMPIAD AND E-TECHNO)

SIMPLE HOLIDAY

PACKAGE PONGAL

Mathematics

Physics

Chemistry

CLASS – IX – E4 & E5

IX_MPC_E4 to E5_SHP

INDEX

SUBJECT PAGE NO.

1. MATHEMATHICS 2 - 14

2. PHYSICS 15 - 31

3. CHEMISTRY 32 - 46

Page 116: 8 & 9_MPC_SHP_E1 & E2& E4 & E5

CLASS-IX_SUBGROUP_E4 & E5 SIMPLE HOLIDAY PACKAGE

NARAYANA GROUP OF SCHOOLS Page 2 of 45

MATHEMATICS (Q.No.1 to 80)

1. The equation of the straight line passing through the point (3,-4)

and making X and Y-intercepts which are in the ratio

2 : 3 is

1) 3x + 2y - 1 = 0 2) 2x + 3y + 12 = 0

3) 3x + 5y + 11 = 0 4) 3x + y + 1 = 0

2. A straight line L with negative slope passes through the point

(8,2) and cuts positive coordinate axes at the points P and Q.

Find the minimum value of OP+OQ as L varies, where O is the

origin.

1) 16 2) 18

3) 14 4) 20

3. 1,1 , 5,3A B are opposite vertices of a square in the XY – plane.

Find the equation of the other diagonal (not passing through

A,B) of the square.

1) 3x + y + 8 = 0 2) x + 3y + 8 = 0

3) 3x + y - 8 = 0 4) x + 3y - 8 = 0

4. The angle between the lines formed by joining the points

2, 3 , 5,1 and 7, 1 , 0, 3

1) /2 2) /4

3) 0 4) /6

5. If the points (1,2) and (3,4) were to be on the same side of the line 3 5 0x y a , then

1) 7 < a < 11 2) a = 7

3) a = 11 4) a < 7 or a > 11

6. The line L given by 15

x y

b passes through the point 13,32 . The

line K is parallel to L and has the equation 1.3

x y

c Then the

distance between L and K is

1) 23

15

2) 16

17

3) 17

15

4) 23

17

Page 117: 8 & 9_MPC_SHP_E1 & E2& E4 & E5

CLASS-IX_SUBGROUP_E4 & E5 SIMPLE HOLIDAY PACKAGE

NARAYANA GROUP OF SCHOOLS Page 3 of 45

7. If the point (1,2) is reflected through the origin and then through the line x = y, then the new coordinates of the point are

1) 1,2 2) 2, 1

3) 2, 1 4) 2,1

8. If p,q are lengths of perpendiculars from the origin to

cosα sinα sin2α;x y sinα cosα cos2α,x y then 2 2p q

1) 2 2) 3

3) 4 4) 1

9. A square of side a lies above the x-axis and has one vertex at the origin. The side passing through the origin makes an angle

πα 0 α4

with the positive direction of x-axis. The equation of

its diagonal not passing through the origin is

1) cosα sin α sin α cosαy x

2) cosα sinα sinα cosαy x a

3) cosα sin α sin α sin αy x

4) cosα sinα cosα sinαy x a

10. The product of the perpendiculars drawn from the two points

2 2 ,0a b to the line cosα sin α 1x y

a b is

1) a 2) b

3) 2a 4) 2b

11. The arc of a triangle is 5 square unit. Two of its vertices are

2,1 , 3, 2 and the third vertex lies on the line 3.y x the third

vertex can be

1) 7 / 2,13/ 2 2) 3/ 2,3/ 2

3) 7 / 2, 13 / 2 4) 3/ 2, 3/ 2

12. The equation of the straight line whose slope 2 / 3 and which

divides the line segment joining 3, 4 , 7, 2 in the ratio 3 : 2

externally is

1) 2 3 12 0x y 2) 3 2 27 0x y

3) 2 3 29 0x y 4) 2 3 72 0x y

Page 118: 8 & 9_MPC_SHP_E1 & E2& E4 & E5

CLASS-IX_SUBGROUP_E4 & E5 SIMPLE HOLIDAY PACKAGE

NARAYANA GROUP OF SCHOOLS Page 4 of 45

13. If , , are the roots of 3 2 2 0 x x x then the equation whose

roots are 2 2 2 2 2 2, , is

1) 3 25 4 16 0 y y y 2) 3 25 4 16 0 y y y

3) 3 25 4 16 0 y y y 4) 3 25 4 16 0 y y y

14. The roots of 4 3 2x 10x 26x 10x 1 0 are

1) 3 2 2,2 3 2) 2 3 2,2 3

3) 3 2 2,3 2 4) 2 3 2,3 2

15. If 2,-2,4 are the roots of 3 2ax bx cx d 0 , then the roots of 3 28ax 4bx 2cx d 0 are

1) 2,-2,4 2) 1/2,-1/2,1/4

3) 1,-1,2 4) 4,-4,8

16. The transformed equation of 5 4 3 2x 5x 3x x x 1 0

by eliminating second term is

1) 5 3 2y 7y 12y 7y 0 2) 3y 2y 1 0

3) 4 2y 4y 1 0 4) 4 2y 24y 65y 55 0

17. A variable straight line drawn through the point of intersection

of the straight lines 1x y

a b and 1

x y

b a meets the coordinate

axes at A and B, then the locus of the mid-point of AB is.

1) 2 a b xy ab x y 2) 2 a b xy ab x y

3) 2 a b xy ab x y 4) 2 a b xy ab x y

18. The incenter of the triangle formed by the straight lines

3 , 3y x y x and 3.y is

1) (0, -2) 2) (0, 2) 3) (-2, 0) 4) (2,3)

19. The nature of the triangle formed by the lines 2 23 0x y and

2x is

1) scalene 2) right angled 3) equilateral 4) isosceles

20. The line 3x-3y+17=0 bisects the angle between a pair of lines of which one line is 2x+y+4=0, then the equation to the other line is

1) 3x+6y-5=0 2) 3x+6y-7=0

3) 3x+6y+8=0 4) None

Page 119: 8 & 9_MPC_SHP_E1 & E2& E4 & E5

CLASS-IX_SUBGROUP_E4 & E5 SIMPLE HOLIDAY PACKAGE

NARAYANA GROUP OF SCHOOLS Page 5 of 45

21. If the lines 2 2 0px qxy y make angles and with the x-axis,

then the value of tan is

1) q

1+p

2) q

1-p

3) -q

1+p

4) -q

1-p

22. 7x y and 2 22 0, 0 ,ax hxy ay a are three real distinct lines

forming a triangle. Then the triangle is

1) Isosceles 2) Scalene 3) Equilateral 4) Right-angled

23. Let PS be the median of the triangle with vertices 2,2 , 6, 1 ,P Q

and 7,3 .R The equation of the line passing through 1, 1 and

parallel to PS is

1) 2 9 7 0x y 2) 2 9 11 0x y

3) 2 9 11 0x y 4) 2 9 7 0x y

24. Two sides of a triangle are 1 0 x y and 2 4 0 x y . If its

circumcentre is (2,1) the third side is

1) 4 3 6 0 x y 2) 3 4 6 0 x y

3) 4 3 6 0 x y 4) 3 4 6 0 x y

25. 1,3A and 2 / 5, 2 / 5C are the vertices of a triangle ABC and

the equation of the internal angle bisector of ABC is 2.x y The equation of side BC is

1) 7 3 4 0x y 2) 7 3 4 0x y

3) 7 3 4 0x y 4) 7 3 4 0x y

26. 1,3A and 2 / 5, 2 / 5C are the vertices of a triangle ABC and

the equation of the internal angle bisector of ABC is 2.x y The coordinates of vertex B are

1) 3/10,17 /10 2) 17 /10,3/10

3) 5/ 2,9 / 2 4) 1,1

27. 1,3A and 2 / 5, 2 / 5C are the vertices of a triangle ABC and

the equation of the internal angle bisector of ABC is 2.x y The equation of side AB is

1) 3 7 24x y 2) 3 7 24 0x y

3) 13 7 8 0x y 4) 13 7 8 0x y

Page 120: 8 & 9_MPC_SHP_E1 & E2& E4 & E5

CLASS-IX_SUBGROUP_E4 & E5 SIMPLE HOLIDAY PACKAGE

NARAYANA GROUP OF SCHOOLS Page 6 of 45

28. 1 1 2 2 3 3, , , ,A x y B x y C x y are the vertices of a triangle ABC.

0lx my n is an equation of the line L.If L intersects the sides

BC,CA and AB of the triangle ABC at P,Q,R respectively then

BP CQ AR

PC QA RB is equal to

1) -1 2) -1/2 3) 1/2 4) 1

29. 1 1 2 2 3 3, , , ,A x y B x y C x y are the vertices of a triangle ABC.

0lx my n is an equation of the line L.If the centroid of the

triangle ABC is at the origin and algebraic sum of the lengths of the perpendiculars from the vertices of the triangle ABC on the line L is equal to 1 then sum of the squares of the intercepts made by L on the coordinate axes is equal to

1) 0 2) 4 3) 9 4) 16

30. 1 1 2 2 3 3, , , ,A x y B x y C x y are the vertices of a triangle ABC.

0lx my n is an equation of the line L.If P dives BC in the ratio

2 : 1 and Q divides CA in the ratio 1 : 3 then R divides AB in the ratio (P,Q,R are the points as in Ex. 36)

1) 2 : 3 internally 2) 2 : 3 externally 3) 3 : 2 internally 4) 3 : 2 externally

31. The pair of lines 2 2 0h x y pxy bisects the angles between the

pair 2 22 0ax hxy by . The value of p is

1) a b 2) b a 3) a b 4) a b

32. If 2 24 2 6 10 0x lxy y x ky represent a pair of parallel lines,

then

1) 3, 2k l 2) 22, 12k l

3) 3, 2k l 4) 16, 9k l

33. If 2 23 2 5 10 0 lx xy y x y k represents a pair of perpendicular

lines, then

1) 3, 7k l 2) 12, 2k l

3) 3, 15k l 4) 15, 9k l

34. The equations of the diagonals of the rectangle formed by the lines 0 hxy gx fy c and the coordinate axes are

1) 0, 0gx fy gx fy c 2) 0, 0fx gy gx fy c

3) 0, 0gx fy c gx fy 4) 0, 0fx gy c gx fy

Page 121: 8 & 9_MPC_SHP_E1 & E2& E4 & E5

CLASS-IX_SUBGROUP_E4 & E5 SIMPLE HOLIDAY PACKAGE

NARAYANA GROUP OF SCHOOLS Page 7 of 45

35. Perpendiculars AL, AM are drawn from any point A on the x –

axis to the pair of lines 2 22 3 0x xy y . The angle made by LM

with x – axis is

1) 2)

2

3)

3

4)

4

36. All the chords of the curve 2 23x y 2x 4y 0 which subtend a

right angle at the origin pass through a fixed point, then the point is

1) (1, 1) 2) (-1, -1) 3) (1, -1) 4) (-1, 1)

37. If the pair of straight lines given by 2 2 2Ax + 2Hxy + By = 0 H > AB

forms an equilateral triangle with line ax + by + c = 0, then

A +3B 3A +B is equal to

1) 2H 2) – 2H 3) 2 2H 4) 4 2H

38. The area of the equilateral triangle formed by the pair of lines

passing through the origin and the line 12x 5y +13 = 0 in

sq.units is

1) 3 3 2) 2 3

3) 3 4) 1

3

39. The two lines represented by 2 2 23ax + 5xy + a 2 y = 0 are

perpendicular to each other for

1) two values of ‘a’ 2) three values of ‘a’

3) for one value of ‘a’ 4) for no value of ‘a’

40. If 2 2a x 1 + 2h x 1 y 2 + b y 2 = 0 has one angular

bisector 2x + 3y 8 = 0 , then other bisector is

1) 2 3 1 0x y 2) 2 3 1 0x y

3) 3 2 1 0x y 4) None of these

41. The angle between the straight lines represented by 2 22 5 2 5 7 3 0 x xy y x y is

1) 1 1

5

Cos 2)

1 3

5

Cos

3) 1 4

5

Cos 4)

1 2

5

Cos

Page 122: 8 & 9_MPC_SHP_E1 & E2& E4 & E5

CLASS-IX_SUBGROUP_E4 & E5 SIMPLE HOLIDAY PACKAGE

NARAYANA GROUP OF SCHOOLS Page 8 of 45

42. If the pairs of lines represented by 2 22 0ax hxy by and 2 22 2 2 0ax hxy by gx fy c form a rhombus, then

1) 2 2 0. a b fg h f g 2) 2 2 0. a b fg h f g

3) 2 2 0. a b fg h f g 4) 2 2 0.a b fg h f g

43. The equation 2 28 24 18 6 9 5 0x xy y x y represents a pair of

parallel straight lines , then the distance between them is

1) 7

53

2) 5

52

3) 7

52

4) 5

53

44. The equation 2 22 13 7 23 6 0x xy y x y represents a pair of

straight lines, then the angle between the lines is

1) 1tan 2 2) 1tan 3

3)

2

4)

4

45. The equation of the bisector of the acute angle between the lines 3 4 7 0x y and 12 5 2 0 x y is

1) 11x – 3y + 9 =0 2) 11x + 3y + 9 =0

3) 11x – 3y – 9 =0 4) 11x + 3y – 9 =0

46. A diagonal of the rectangle formed by the lines 2x 7x 6 0

and 2y 14y 40 0 is

1) 5x + 6y = 0 2) 5x – 6y = 0

3) 6x – 5y + 14 = 0 4) 6x – 5y - 14 = 0

47. The angle between the pair of straight lines formed by joining

the points of intersection of 2 2x y 4 and y =3x+c to the origin

is a right angle, then 2c

1) 20 2) 13

3) 1/5 4) 5

48. The equation 2 28x 8xy 2y 26x 13y 15 0 represents a pair

of parallel straight lines, then the perpendicular distance between them is

1) 7

5

2) 3

2 5

3) 7

2 5

4) 2

3 5

Page 123: 8 & 9_MPC_SHP_E1 & E2& E4 & E5

CLASS-IX_SUBGROUP_E4 & E5 SIMPLE HOLIDAY PACKAGE

NARAYANA GROUP OF SCHOOLS Page 9 of 45

49. If 5,4,6 , 1, 1,3 , 4,3, 2A B C are vertices of a triangle ABC, then the

coordinates of the point in which the bisector of BAC meets the side

BC is

1) 23 3 19, ,

8 2 8

2) 23 3 19

, ,8 2 8

3) 23 3 19, ,

8 2 8

4) 23 3 19

, ,8 2 8

50. If a triangle formed by the points 1,2,3 , 2,3,1 and 3,1, 2 , then

orthocenter is

1) 3,1, 2 2) 2,2,2

3) 2,2,1 4) 1,1, 2

51. If A,B,C are three points on , , Ox Oy Oz respectively, at distances a, b, c

0, 0, 0a b c from the origin ‘O’, then the coordinates of the point

which is equidistant from A,B,C and O is

1) , ,

2 2 2

a b c

2)

, ,3 3 3

a b c

3) , ,a b c 4) , ,2 2 2

a b c

52. The in-centre of the triangle formed by the points 0,0,0 , 3,0,0 and

0,4,0 is

1) (1, 1, 0) 2) (0,1,1)

3) (1, 0, 1) 4) (1,1,1)

53. The equations to a pair of opposite sides of a parallelogram are 2 5 6 0x x and

2 6 5 0.y y The equations to its diagonals are

1) 4 13, 4 7x y y x

2) 4 13,4 7x y y x

3) 4 13, 4 7x y y x 4) 4 13, 4 7y x y x

54. The lines joining the origin to the points of intersection of 2 2x y 2gx c 0 and

2 2x y 2fy c 0 are at right angles is

1) 2 2g f c 2) 2 2g f 0

3) 2 2g f 2c 4) 2 2 2g f c

Page 124: 8 & 9_MPC_SHP_E1 & E2& E4 & E5

CLASS-IX_SUBGROUP_E4 & E5 SIMPLE HOLIDAY PACKAGE

NARAYANA GROUP OF SCHOOLS Page 10 of 45

55. The distance from the point of intersection of the lines 2 2 2 2 0 x y x y to the point of intersection of the lines

2 22 5 2 1 0 x xy y x y is

1) 1 2) 0

3) 2 4) 4

56. The circumcentre of the triangle formed by the lines x2 – y2 = 0 and y–

5=0 is

1) (5, 0) 2) (0, 5)

3) (0, 0) 4) (5, 5)

57. Let ABCD be a parallelogram whose equations for the diagonals AC and

BD are and , respectively If length of diagonal AC is

4 units and the area of parallelogram ABCD is 8 sq. units, then the

length of other diagonal BD is

1) 10/3 2) 2

3) 20/3 4) none of these

58. Let ABCD be a parallelogram whose equations for the diagonals AC and

BD are and , respectively The length of side AB is

equal to

1) 2)

3) 4)

59. Let ABCD be a parallelogram whose equations for the diagonals AC and

BD are and , respectively The length of BC is equal

to

1) 2)

3) 4) none of these

60. The centroid of the tetrahedron formed by the points 1 1 1, ,A x y z ,

2 2 2, ,B x y z , 3 3 3, ,C x y z and 4 4 4, ,D x y z is

1 2 3 4 1 2 3 4 1 2 3 4, ,4 4 4

x x x x y y y y z z z z

The circumcentre of the triangle formed by the points (1,2,3), (3,-1,5), (4,0,-3) is

1) (1,1,1) 2) 7 1, ,12 2

3) (3,3,3) 4) 7 1, ,12 2

x 2y 3 2x y 3

x 2y 3 2x y 3

2 58 / 3 4 58 / 9

3 58 / 9 4 58 / 9

x 2y 3 2x y 3

2 10 / 3 4 10 / 3

8 10 / 3

Page 125: 8 & 9_MPC_SHP_E1 & E2& E4 & E5

CLASS-IX_SUBGROUP_E4 & E5 SIMPLE HOLIDAY PACKAGE

NARAYANA GROUP OF SCHOOLS Page 11 of 45

61. The centroid of the tetrahedron formed by the points 1 1 1, ,A x y z ,

2 2 2, ,B x y z , 3 3 3, ,C x y z and 4 4 4, ,D x y z is

1 2 3 4 1 2 3 4 1 2 3 4, ,4 4 4

x x x x y y y y z z z z

Let A (4,7,8), B (2,3,4) and C (2,5,7) be the vertices of ABC . The

length of the median AD is

1) 2 2) 1

2

3) 77

2

4) 89

2

62. The centroid of the tetrahedron formed by the points 1 1 1, ,A x y z ,

2 2 2, ,B x y z , 3 3 3, ,C x y z and 4 4 4, ,D x y z is

1 2 3 4 1 2 3 4 1 2 3 4, ,4 4 4

x x x x y y y y z z z z, , are the root of

3 22 2 0 x x x .

Centroid of triangle with verties , , , ( , , ), ( , , ) is

1) 2 2 2, ,3 3 3

2) 2 1 2

, ,3 3 3

3) 2 2 2, ,3 3 3

4) 4 2 2

, ,3 3 3

63. If three consecutive vertices of a parallelogram are

4,3,5 , 0,6,0 . 8,1, 4A B C and D is the fourth vertex then

the angle between AC

and BD

is

1) 1 55

149 161

Cos 2)

1 65

149 161

Cos

3) 1 15

149 161

Cos 4)

1 3

149 161

Cos

64. The three lines with d.r’s , (1,1,2) ( 3 1, 3 1, 4) , ( 3 1, 3 1, 4) forms

1) An equilateral triangle 2) A right angled triangle 3) An isosceles triangle 4) A right angled isosceles triangle

65. If the dr’s of a line are 1 ,1 , 2 and it makes an angle 060

with the Y- axis then is

1) 1 3 2) 4 5

3) 2 2 5 4) 2 5

Page 126: 8 & 9_MPC_SHP_E1 & E2& E4 & E5

CLASS-IX_SUBGROUP_E4 & E5 SIMPLE HOLIDAY PACKAGE

NARAYANA GROUP OF SCHOOLS Page 12 of 45

66. If the angle between line with d.c’s

2, ,

21 21 21

a b and other line

with d.c’s 3 3 6, ,

54 54 54

is 900 then a pair of possible values of

‘a’ and ‘b’ respectively are

1) -1, 4 2) 4, 2 3) 4,1 4) -4,-2

67. The vertices of a triangle are 2,3,5 , 1,3,2 , 3,5, 2 , then the

angles are

1) 300, 300, 1200 2) 1 0 11 5

,90 ,5 3

Cos Cos

3) 0 0 030 ,60 ,90 4) 1 0 11 2

,90 ,3 3

Cos Cos

68. If the vertices of a triangle are 1,1,1 , 4,1,1, , 4,5,1 , then the area

of triangle is

1) 5 sq. unit 2) 6 sq. unit 3) 3 sq. unit 4) 2 sq. unit

69. Let a line makes an angle ‘ ’ with X and Z - axes and with

Y -axis. If sin 3 sin , then 2cos

1) 3

5

2) 5

3

3) 2

5

4) 1

5

70. If ( ,3,5)x and (2, 1,2) are d.r’s of two lines and angle between the

lines is 045 then the values of x are

1) 4, 52 2) 3,42

3) 4,52 4) 3,32

71. A line OP where O = (0, 0, 0) makes equal angles with OX, OY, OZ. The point on OP, which is at a distance of 6 units from ‘O’ is

1) 12 12 12, ,3 3 3

2) 2 3, 2 3, 2 3

3) 2 3, 2 3, 2 3 4) 6 3, 6 3, 6 3

72. The d.r’s of the lines AB and AC are (1, 2, -2) and (2, -3, 6).

The d.r’s of a bisector of the BAC are

1) 3, -1, -8 2) -1, 5, -8 3) 1, 23, -32 4) None of these

Page 127: 8 & 9_MPC_SHP_E1 & E2& E4 & E5

CLASS-IX_SUBGROUP_E4 & E5 SIMPLE HOLIDAY PACKAGE

NARAYANA GROUP OF SCHOOLS Page 13 of 45

73. The line passing through the points A 5,1,a and B 3, ,1b

crosses the yz plane at the point 17 13

0, ,2 2

, then the

values of a, b are

1) 6,4 2) 3,5 3) 2,4 4) 1,6

74. The point to which the axes should be translated to eliminate

first degree terms in the equation 2 2 2 2 4 2 3 0 x y z x y z is

1) (1,2,-1) 2) (2,4,-2) 3) (3,2,1) 4) (2,6,3)

75. The d.c's of two lines are

3 1 3, ,

4 4 2and

3 1, ,

4 4k If the angle

between the lines is 60o then k =

1) 3 2) 2

3

3) 1

2 3

4) 3

2

76. A = (1, 2, 3), B = (4, 5, 7), C = (-4, 3, -6), D = (2, k, 2) are four

points. If the lines AB and CD are parallel then k =

1) 0 2) -9

3) 9 4) None of these

77. If a line makes angles 60 , 60 , 45o o o and with the four diagonals of

a cube then 2sin

1) 2

3

2) 11

12

3) 11

12

4) 31

12

78. If the direction cosines of any two lines are

2 / 3,1/ 3, 2 / 3 , 3/ 5, 4 / 5,0 respectively, then the acute angle

between those two lines is

1) 1 1

7Cos

2)

1 2

15Cos

3) 1 5

7Cos

4)

1 10

7Cos

Page 128: 8 & 9_MPC_SHP_E1 & E2& E4 & E5

CLASS-IX_SUBGROUP_E4 & E5 SIMPLE HOLIDAY PACKAGE

NARAYANA GROUP OF SCHOOLS Page 14 of 45

79. If a line in the space makes angles , and with the

coordinate axes, the 2 2 2cos2 cos2 cos2 sin sin sin

1) -1 2) 0 3) 1 4) 2

80. If the angles made by a straight line with the coordinate axis are

, ,2

then

1) 0 2)

6

3)

2

4)

IX_MATHEMATICS_SHP_E4 &E5_KEY

Q.No. 1 2 3 4 5 6 7 8 9 10

Key 1 2 3 3 4 4 3 4 4 4

Q.No. 11 12 13 14 15 16 17 18 19 20

Key 1 4 2 1 3 1 4 3 4 4

Q.No. 21 22 23 24 25 26 27 28 29 30

Key 1 4 4 1 2 2 2 2 2 2

Q.No. 31 32 33 34 35 36 37 38 39 40

Key 2 1 2 1 4 3 4 4 1 3

Q.No. 41 42 43 44 45 46 47 48 49 50

Key 3 4 3 2 1 3 1 3 3 2

Q.No. 51 52 53 54 55 56 57 58 59 60

Key 4 1 3 3 2 2 3 1 1 4

Q.No. 61 62 63 64 65 66 67 68 69 70

Key 3 3 1 1 4 3 4 2 1 3

Q.No. 71 72 73 74 75 76 77 78 79 80

Key 3 3 1 1 3 3 1 2 3 3

Page 129: 8 & 9_MPC_SHP_E1 & E2& E4 & E5

CLASS-IX_SUBGROUP_E4 & E5 SIMPLE HOLIDAY PACKAGE

NARAYANA GROUP OF SCHOOLS Page 15 of 45

PHYSICS ( Q.NO. 1 TO 80)

1. Water flows through horizontal tube As shown in the figure. If the difference of heights of water column in the vertical tubes ish= 0.03 m, and the areas of cross section act A and B

are 4 10-4m2 and 1 10-4m2metal scrap respectively. Then the rate of flow of water across any section is (in cm3/s)

1) 55.56 2) 56.56

3) 57.56 4) 58.56

2. The diagram shows a pipe of uniform cross section in which

water is flowing.The directions of flow and the volume flow

rates(in cm3/s) are shown for various portions of the pipe.

The direction of flow and the volume flow rate in the portion

marked A are

1) 315 /secand cm 2) 35 /secand cm

3) 315 /secand cm 4) 35 /secand cm

3. Which of the following is a characteristic of turbulent flow?

1) Velocity more than critical

velocity

2) Irregular flow

3) Molecules crossing from one

layer to the other

4) All the above

Page 130: 8 & 9_MPC_SHP_E1 & E2& E4 & E5

CLASS-IX_SUBGROUP_E4 & E5 SIMPLE HOLIDAY PACKAGE

NARAYANA GROUP OF SCHOOLS Page 16 of 45

4. The below figure shows a liquid of density 1200 kg/m3

flowing steadily in a tube of varying cross section the cross-

section at ‘A’ point is 1.0 cm2 and that get ‘B’ is 20 mm the

points A and B are in the same horizontal plane.The speed of

the liquid ‘A’ is 10cm/sec then the difference in difference in

pressure at A and B is

1) 144a 2) 154a

3) 164a 4) 174a

5. The areas of cross section of the wide and narrow portions of the tube are 10 cm2 and 4 cm2 respectively are shown in figure. The rate of flow of the water through the tube is

1000cm3/sec.The difference of Mercury levels in the U-Tube

is

1) 0.97cm 2) 1.97cm

3) 0.67cm 4) 6.67cm

6. A cylindrical vessel of 90 cm height is kept filled up to the brim. It has four holes 1, 2, 3 and 4 which are respectively at heights of 20 cm, 30 cm, 45 cm and 50 cm from the horizontal floor PQ. The water falling at the maximum horizontal distance from the vessel comes from

1) Hole number 4 2) Hole number 3

3) Hole number 2 4) Hole number 1

Page 131: 8 & 9_MPC_SHP_E1 & E2& E4 & E5

CLASS-IX_SUBGROUP_E4 & E5 SIMPLE HOLIDAY PACKAGE

NARAYANA GROUP OF SCHOOLS Page 17 of 45

7. Water flows through a non-uniform tube of area P,Q and R, as shown in. The portion having the highest velocity is

1) P 2) Q

3) R 4) Same in all

8. Figure shows two holes in a wide tank containing a liquid column. The water streams coming one of these holes strike the ground at the same point. The height of liquid column in the tank is

1) 10cm 2) 8cm

3) 9.8cm 4) 980cm

9. One end of a cylindrical pipe has a radius of 2.5 cm water

(density=1.0103 kg/m3) streams steadily out at 9.0 m/s the rate at which mass is leaving the pipe is

1) 16.77 kg/sec 2) 16.77 g/sec

3) 17.66 kg/sec 4) 17.66 g/sec

10. A liquid of density 3800kgm is filled in a tank open at the top

the pressure, of the liquid. At the bottom of the tank is 6.4 atmospheres. The velocity of efflux through a hole at the

bottom is 5 2(1 10 )atmosphere Nm

1) 110ms 2) 120ms

3) 130ms 4) 140ms

11. A large open tank has two small holes in its vertical wall as shown in figure. One is a square hole of side ' 'L at a depth '4 'y from the top and the other is a circular hole of radius ' 'R

at a depth ‘y’ from the top. When the tank is completely

filled with water, the quantities of water flowing out per second from both holes are the same Then ' 'R is euql to :

Page 132: 8 & 9_MPC_SHP_E1 & E2& E4 & E5

CLASS-IX_SUBGROUP_E4 & E5 SIMPLE HOLIDAY PACKAGE

NARAYANA GROUP OF SCHOOLS Page 18 of 45

1)

2

L

2) 2 L

3) 2L

4)

2

L

12. Which of the following diagrams does not represent a streamline flow

1)

2)

3)

4)

13. The height of water level in a tank is H the range of water

stream coming out of a hole at a depth H

4 from upper water

level will be

1) 3H

2

2) 2H

3

3) H

3

4) 3H

14. The cylindrical tube of a spray pump has a cross section of

8.0 cm2 on one end which has 40fine holes each of diameter

1.0 mm. if the liquid flow inside the tube is 1.5 m min-1. what

is the speed of a ejection of the liquid through the holes

1) 0.376 m/s 2) 0.367 m/s

3) 0.637 m/s 4) 0.736 m/s

Page 133: 8 & 9_MPC_SHP_E1 & E2& E4 & E5

CLASS-IX_SUBGROUP_E4 & E5 SIMPLE HOLIDAY PACKAGE

NARAYANA GROUP OF SCHOOLS Page 19 of 45

15. The liquid flow is most streamlined when

1) liquid of high viscosity and High density flowing through a tube of small radius

2) liquid of high viscosity and the low density flowing through a tube of small radius

3) liquid of low viscosity and the low density flowing through a tube large radius

4) liquid of low viscosity and high density flowing through a tube of arger radius

16. A non-viscous incompressible liquid is flowing through a

horizontal pipe of constant cross-section. Bernoulli's

equation and the equation of continuity predict that the

change in pressure along the pipe

1) is zero 2) depends on the length of the

pipe

3) depends on the fluid velocity 4) depends on the cross sectional

area of the pipe

17. The time of making empty a hemispherical tank of radius `R’ through an orifice of cross sectional area`a’ at its bottom

1) T=∏ 2) T=14∏ 3) T=14∏ 4) T=∏

18. The quantity `h’ appearing in Bernoulli’s equation ( P+ v2+ gh

=constant) must be measured:

1) Upward from the centre of earth

2) Upward from the surface of earth

3) Upward from the lowest point in the flow

4) Upward from any convenient level.

19. If the refractive index of the material of a prism is cot (A/2)and

vertex angle of the prism is A. the angle of minimum deviation is

1) ∏-A 2) ∏ − 2 3)

∏ − 4) ∏-2A

20. Light travels a distance `x’ in medium and `xo’ in vacuum during

the same time. Then critical angle of the medium is

1) sin-1(xxo) 2) sin ( ) 3) sin ( ) 4) sin (√xxo)

21. A convex refracting spherical surface of radius `R’ seperates two media of refractive indices µ1and µ2 .Where should an object be placed in the medium `1’ so that a real image is formed in medium

Page 134: 8 & 9_MPC_SHP_E1 & E2& E4 & E5

CLASS-IX_SUBGROUP_E4 & E5 SIMPLE HOLIDAY PACKAGE

NARAYANA GROUP OF SCHOOLS Page 20 of 45

2 at the same distance. (µ1> µ2,convex surface is towards medium1)

1) =µ µµ µ

2) = µ µµ µ

3) = µ µµ µ

4) = µ µµ µ

22. A parallel beam of light travelling in water ( µw=4/3) is refractive by a spherical air bubble of radius 2mm situated in water. find the position of the image due to refraction at the first surface.

1) -6mm 2) -5mm 3) -3mm 4) 3mm

23. One end of a horizontal cylinder glass rod (µ =1.5)of radius 5.0 cm is rounded in the shape of hemisphere. An object 0.5 mm high is

placed perpendicular to the axis of the rod at a distance of 20.0

cm from the rounded edge. Find the image height.

1) 0.5 mm 2) -0.5 mm 3) 1 mm 4) -1 mm

24. Find the size of the image formed in the situation shown in figure.

1) 0.3cm 2) 0.6cm 3) 0.9cm 4) 0.2cm

25. If the light moving in a straight line bends by a small but fixed angle,it may be a case of

1) Interference 2) Refraction 3) Diffraction 4) Dispersion

Paragraph for Questions 26 to 28

The minimum angle of deviation m is given by µ=( )

/

Find the minimum angles of deviation of an equilateral prism for the

following cases.

26. m for glass prism of wave length 400nm having refractive index 1.66 is____________

1) 480 2) 240 3) 360 4) 520

27. m for glass prism of wave length 700mm having refractive index 1.61 is ______

1) 480 2) 240 3) 360 4) 520

28. m for glass prism having refractive index √3,is__________ 1) 300 2) 450 3) 600 4) 750

O

C

1 1m 1.53m

20 cm

40 cm

Page 135: 8 & 9_MPC_SHP_E1 & E2& E4 & E5

CLASS-IX_SUBGROUP_E4 & E5 SIMPLE HOLIDAY PACKAGE

NARAYANA GROUP OF SCHOOLS Page 21 of 45

Paragraph for Questions 29 to 31

The formula governing refraction from rarer to denser medium is

2 2 11

v u R

m m mm

29. A spherical convex surface of radius of curvature R separates air

(µa=1) from diamond (aµd=2.5). The center of curvature is in the

diamond .A point object O placed in air is found to have a real

image Q in the diamond .The line PQ cuts the surface at a point O

and PO=OQ The distance PO is equal to

1) 7R 2) 7R

3 3) 7 R

5 4) 7

R2

30. A glass dumbbell of length 30 cm and µ=1.5 has ends 3cm radius of curvature .An object is situated in air at a distance of 9 cm

from the end of dumbbell along the axis. The position of image

formed due to

1) 3cm 2) 9cm 3) 27 cm 4) 18cm

31. An object O is stuck on the surface of a transparent solid sphere of radius 5 cm .The refractive index of the sphere such that rays from the object after refraction from the opposite side emerge as a parallel beam is

1) 1.5 2) 1.414 3)1 4)2

32. Water from a tap emerges vertically downwards with initial

velocity 4 m/s. The cross-sectional area of the tap is A. The

flow is Steady and pressure is constant throughout the

stream of water. The distance h vertically below the tap

where the cross sectional area of the stream becomes2

3A

is

(g=10m/s2)

1) 0.5 m 2) 1 m

3) 1.5 m 4) 2.2 m

33. An object is kept at a distance of 16 cm from a thin lens and the image formed is real. If the object is kept at a distance of 6 cm from the same lens, the image formed is virtual. If the sizes, of the images formed are equal, the focal length of the lens will be

1) 15 cm 2) 17 cm 3) 21 cm 4) 11 cm

34. A convex lens of focal length 10 cm is painted black at the middle portion as shown in (figure). An object is placed at a distance of 20 cm from the lens then

Page 136: 8 & 9_MPC_SHP_E1 & E2& E4 & E5

CLASS-IX_SUBGROUP_E4 & E5 SIMPLE HOLIDAY PACKAGE

NARAYANA GROUP OF SCHOOLS Page 22 of 45

1) Only one image will be formed by the lens

2) The distance between the two images formed by such a lens is 6 mm

3) The distance between the images is 4 mm

4) The distance between the images 2 mm

35. A screen is placed a distance 40 cm away from an illuminated

object. A converging lens is placed between the source and the

screen and it is attempted to form the image of the source on

the screen. If no position could be found, the focal length of the

lens

1) must be less than 10 cm 2) must be greater than 20 cm

3) must not be greater than

20 cm

4) must not be less than 10 cm.

36. The graph shows the variation of magnification m produced by

convex lens with image distance v. The power of the lens is :

1) b/c 2) b/ca

3) bc/a 4) c/b

37. An object is situated at a distance of 20 cm from a convex lens of focal length 30 cm. The position of the image formed by it will be

1) - 30 cm 2) 30 cm

3) -60 cm 4) + 60cm

38. When an object is placed 15 cm from a lens, a virtual image is formed. Mark the correct statements.

1) The lens may be convex or concave

2) If the lens is diverging, the image distance has to be less than 15 cm

3) If the lens is converging, then its focal length has to

be greater than 15 cm

4) All of the above

Page 137: 8 & 9_MPC_SHP_E1 & E2& E4 & E5

CLASS-IX_SUBGROUP_E4 & E5 SIMPLE HOLIDAY PACKAGE

NARAYANA GROUP OF SCHOOLS Page 23 of 45

39. A lens forms a diminished and erect image of an object. The

magnification is 1

4. Find ratio of distances between object and

focus and focus and image:

1) 4 : 1 2) 1 : 4 3) 8 : 1 4) 2 : 1

40. When an object is placed on the principal axis of a convex lens at two different positions, it produces the images with magnifications +2 and -4 respectively. How many times more away from the lens the image will be formed in the second position as compared to the first position?

1) 2 2) 4 3) 5 4) 10

41. The distance between two point sources of light is 24 cm and a converging lens is kept in between two sources. The object distances of two sources from a converging lens of focal length of 9 cm, so that the image distances of two sources are equal

1) 10 cm 2) 12 cm

3) 18 cm or 6 cm 4) 24 cm

42. The image of a small electric bulb fixed on the wall of a room is to be obtained on the opposite wall 3 m away by means of a large convex lens. What is the maximum possible focal length of the lens required for the purpose?

1) 0.75 m 2) 0.8 m 3) 1m 4) 0.75 cm

43. The decrease in the aperture of the lens changes

1) The position of the image 2) The size of the image 3) The intensity of the image 4) Both the position and size of the

image 44. The graph drawn with object distance along abscissa and image

distance along ordinate measuring the distance from the convex lens is

1) Straight line 2) Parabola 3) Circle 4) A hyperbola

45. An object of height 6cm is kept at a distance of 30cm from the

optic centre of a convex lens of focal length 8cm. The height of

image is

1) 1cm

3

2) 1- cm3

3) -3 cm 4) None of these

Page 138: 8 & 9_MPC_SHP_E1 & E2& E4 & E5

CLASS-IX_SUBGROUP_E4 & E5 SIMPLE HOLIDAY PACKAGE

NARAYANA GROUP OF SCHOOLS Page 24 of 45

46. A point source of light is placed at a distance of 2 f from a

converging lens of focal length f. The intensity on the other side

of the lens is maximum at a distance

1) f 2) between f and 2f

3) 2f 4) more than 2 f.

47. A parallel beam of light is incident on a converging lens parallel

to its principal axis. As one moves away from the lens on the

other side on its principal axis, the intensity of light

1) remains constant 2) continuously increases

3) continuously decreases 4) first increases then decreases.

48. A convex lens forms a real image of a point object placed on its

principal axis. If the upper half of the lens is painted black,

1) the image will be shifted

downward

2) the image will be shifted upward

3) the image will not be

shifted

4) the intensity of the image will

increase.

49. An ideal fluid flows through a pipe of circular cross section made of two sections with diameters 2.5cm and 3.75cm.The ratio of the velocity in the two pipes is

1) 9:4 2) 3:2 3) 3 : 2 4) 2 : 3

50. During blood transfusion the needle is inserted in a vein where

the gauge pressure is 2000Pa. At what height must the blood

container be placed so that blood may just enter the vein

nearly(density of blood 3 31.06 10 /Kg m )

1) 0.8m 2) 0.5m

3) 0.2m 4) 0.4m

51. A plane is in level flight at constant speed and each of its two

wings has an area of 25m2. If the speed of the air is 180 km/h

over the lower wing and 234 km/h over the upper wing surface ,

determine the plane mass (take air density 1 kg/m3)

1) 2200kg 2) 3400kg

3) 4393kg 4) 6543kg

Page 139: 8 & 9_MPC_SHP_E1 & E2& E4 & E5

CLASS-IX_SUBGROUP_E4 & E5 SIMPLE HOLIDAY PACKAGE

NARAYANA GROUP OF SCHOOLS Page 25 of 45

52. In the figure shown

sin

sin

i

r is equal to

1) 2

2

3 1

2) 3

1

3) 3 1

2

2

4) 1

3

53. A ray of light travelling in glass 32g is incident on a glass

air surface at the critical angle. If a thin layer of water 43w

is now poured on the glass air surface , at what angle will the ray of light emerge into air at the water air surface?

1) 600 2) 300 3) 450 4) 900

54. A light ray is incident on a glass sphere of refractive index 3 at

an angle of incidence 600 as shown .Then the angles r, r1 , e and total deviation after two refractions are

1) 300,300,600 & 600 2) 600,600,300 & 300 3) 600,300,600 & 300 4) 300,600,300 & 600

55. A square box of water has a small hole located in one of the bottom corners. When the box is full and sitting on a level surface, complete opening of the hole results in a flow of water with a speed v0, as shown. When the box is still half empty, it is tilted by 450 so that the hole is at the lowest point. Now the water will flow out with a speed of

Page 140: 8 & 9_MPC_SHP_E1 & E2& E4 & E5

CLASS-IX_SUBGROUP_E4 & E5 SIMPLE HOLIDAY PACKAGE

NARAYANA GROUP OF SCHOOLS Page 26 of 45

1) 0v 2)

0

2

v

3) 0

2

v

4) 0

4 2

v

56. A ray of light from a denser medium strikes a rarer medium at

an angle of incidence i. The reflected and refracted rays make an angle of 900 with each other. The angles of reflection and refraction are r and r1 . The critical angle is

1) Sin-1(tanr) 2) Sin-1(coti) 3) Sin-1(tanr1) 4) tan-1(sini)

Paragraph for Questions 57 to 59

In total internal reflection , the incident light is completely reflected into same medium

57. Light takes time t1 to travel a distance x1 in vacuum and same light takes t2 to travel a distance of x2 in a medium. The critical angle for that medium is

1) 1 2 1

1 1

sinx t

x t

2)

1 1 2

2 1

sinx t

x t

3) 1 1 1

2 2

sinx t

x t

4)

1 2 1

1 2

sinx t

x t

58. Total internal reflection is possible when a ray of light passes from

1) Air into water 2) Air into glass 3) Glass into water 4) Water into glass

59. The critical angle for a ray of light buffering total internal reflection will be smallest for light travelling from

1) Water into air 2) Glass into air 3) Glass into water 4) Water into glass

Paragraph for Questions 60 to 62

Snell’s law 2 1

1 2

sin

sin

vi

r v

Page 141: 8 & 9_MPC_SHP_E1 & E2& E4 & E5

CLASS-IX_SUBGROUP_E4 & E5 SIMPLE HOLIDAY PACKAGE

NARAYANA GROUP OF SCHOOLS Page 27 of 45

60. The angle a made by the light ray when it gets refracted from

water to air, as shown in the figure

1) 1 4sin

5a

2) 1 5

sin4

a

3) 1 3sin

5a

4) 1 5

sin3

a

61. Find the speed of light in medium ‘a’ if speed of light in medium ‘b’ is c/3, where c=speed of light in vacuum and light refracts from medium ‘a’ to medium ‘b’ making 450 and 600, respectively, with the normal.

1) 2

3 3

c

2) 2

3c

3) 1

3 3

c

4) 2

2 3

c

62. A ray of light is incident on a transparent glass slab of refractive

index 3 . If the reflected and reflected rays are mutually

perpendicular, what is the angle of incidence?

1) 600 2) 300

3) 450 4) 900

63. At what distance must an object be placed from a convex

lens of power 4D to obtain a real image three times the size of

the object

1) 3 cm 2) 33.3 cm

3) 67 cm 4) 16.7 cm

Page 142: 8 & 9_MPC_SHP_E1 & E2& E4 & E5

CLASS-IX_SUBGROUP_E4 & E5 SIMPLE HOLIDAY PACKAGE

NARAYANA GROUP OF SCHOOLS Page 28 of 45

64. A convex lens of focal length 25 cm is placed in between the

source of light and screen. The distance between the source

and screen is 1.25 m. The position of the lens so that real

image of the source is formed on the screen

1) 25.44 cm 2) 69 cm

3) 34.55 cm 4) 89.45 cm

65. A ray of light is incident at an angle of 60o on one face of a prism

which has an angle of30o . The ray emerging out of the prism

makes an angle 30o with the incident ray. The refractive index of

the material of the prism is

1) 2 2) 3

2

3) 1

2

4) 3

66. The refractive indices of the crown glass for blue and red lights

are 1.51 and 1.49 respectively and those of the flint glass are

1.77 and 1.73 respectively. An isosceles prism of angle 6o is

made of crown glass. A beam of white light is incident at a small

angle on this prism. The other flint glass isosceles prism is

combined with the crown glass prism such that there is no

deviation of the incident light. The net dispersion of the

combined system is

1) 4o 2) 4

o

3) 0.04o 4) 0.004o

67. The R.I of the prism for violet colour is 1.7 and that for red is

1.65. if the refractive index for mean colour is1.66. Dispersive

power of that material is

1) 0.0157 2) 0.07575

3) 0.0525 4) 0.7575

68. An Optical fibre ( 1.7) is surrounded by a glass coating ( 1.5)

The Critical angle for total internal reflection at the fibre glass

interface is

1) 1 85sin75

2)

1 75sin85

3) 1 2

sin15

4) 1 2

sin17

Page 143: 8 & 9_MPC_SHP_E1 & E2& E4 & E5

CLASS-IX_SUBGROUP_E4 & E5 SIMPLE HOLIDAY PACKAGE

NARAYANA GROUP OF SCHOOLS Page 29 of 45

69. The difference between angle of minimum deviation of violet and red rays in the spectrum of white light from a prism is 20. If the angle of minimum deviation of mean ray is 480. Dispersive power of the material of the prism is

1) 0.0416 2) 0.0832 3) 0.0208 4) 0.0616

70. The ratio of the angle of minimum deviation of a prism in air

and when dipped in water will be ( 3 / 2 4 / 3)a g a w

and

1) 1 2) 3

3) 4 4) 5

71. A prism of refractive index and angle A is placed in minimum

deviation position. If the angle of minimum deviation is A, then

the value of A in terms of is

1) 1sin2

2)

1cos2

3) 12cos2

4)

12sin2

72. A black spot is present inside one of the face of an equilaterial

prism. A man places his eye directly at the opposite corner. He

sees two images of the spot at an angular separation of 60o .

Then the minimum value of refractive index of the prism is :

1) 1

3

2) 1

2

3) 1

2

4) 2

3

73. In the position of minimum deviation when a ray of yellow light

passes through the prism, then its angle of incidence is

1) Less than the

emergent angle

2) Greater than the emergent angle

3) Sum of the angle of

incident and emergent

angle is 90 degrees

4) Equal to emergent angle

74. A thin prism of angle 60 made of a glass of R.I 1.5 is combined

with another Prism made of glass of R.I 1.75 to produce

dispersion without deviation. Then the angle of second Prism is

1) 70 2) 40

3) 90 4) 50

75. In the formation of rainbow light from the sun on water droplets

undergoes

1) Only dispersion 2) Dispersion and total internal

reflection

3) Only total internal

reflection

4) None of these

Page 144: 8 & 9_MPC_SHP_E1 & E2& E4 & E5

CLASS-IX_SUBGROUP_E4 & E5 SIMPLE HOLIDAY PACKAGE

NARAYANA GROUP OF SCHOOLS Page 30 of 45

76. A thin Prism of angle 60 move up of glass of R.I 1.5 is combined

with another Prism move up of glass of R.I 1.75 to produce

dispersion without deviation. The angle of 2nd Prism is

1) 70 2) 90

3) 40 4) 50

77. If the refractive index of the material of a prism is cot( / 2)A and

vertex angle of the prism is A. The angle of minimum deviation

is

1) A 2) 2

2A

3)

2A

4) 2A

78. If the refractive index of diamond is 2.4, the velocity of light in

diamond is (c=3×108 m/s)

1) 0.125 × 108 m/s 2) 0.0125 × 108 m/s

3) 1.25 × 108 m/s 4) 2.5 × 108 m/s

79. By properly combining two prisms made of different material it

is possible to have

1) Have dispersion

without average

deviation

2) Have deviation without dispersion

3) Have both dispersion

and average deviation

4) All of these

80. A 60 degrees glass prism have a refractive index 1.5 The angle of

incidence for maximum deviation is

1) 90o 2) 38o

3) 58o 4) 42o

IX_PHYSICS_SHP_E4 & E5_KEY

Q.NO. 1 2 3 4 5 6 7 8 9 10

KEY 2 1 4 1 2 2 2 1 3 4

Q.NO. 11 12 13 14 15 16 17 18 19 20

KEY 3 4 1 3 2 1 2 4 4 2

Q.NO. 21 22 23 24 25 26 27 28 29 30

KEY 2 1 2 2 2 4 1 3 2 3

Q.NO. 31 32 33 34 35 36 37 38 39 40

Page 145: 8 & 9_MPC_SHP_E1 & E2& E4 & E5

CLASS-IX_SUBGROUP_E4 & E5 SIMPLE HOLIDAY PACKAGE

NARAYANA GROUP OF SCHOOLS Page 31 of 45

KEY 4 2 4 1 2 4 3 4 3 3

Q.NO. 41 42 43 44 45 46 47 48 49 50

KEY 3 1 3 4 4 3 4 3 1 3

Q.NO. 51 52 53 54 55 56 57 58 59 60

KEY 1 2 4 1 4 1 4 3 2 1

Q.NO. 61 62 63 64 65 66 67 68 69 70

KEY 1 1 2 3 4 3 2 2 1 3

Q.NO. 71 72 73 74 75 76 77 78 79 80

KEY 3 4 4 2 2 3 4 3 4 1

CHEMISTRY (Q.No.1 to 80)

1. The equilibrium constant for the reaction H

2(g) + I

2(g) 2HI (g) is 32 at a given temperature. The

equilibrium concentrations of I2 and HI are 0.25 × 10

–3 and

8 × 10–3

M respectively. The equilibrium concentration of H2 is:

1) 4 × 10–3

M 2) 1.5 × 10–3

M 3) 8 × 10

–3 M 4) none

2. For the reaction, 2 2 3N g +3H g 2NH g ,the value of Kp= 82

at 400 K. Find out the value of Kp for each of the following

reactions at the same temperature:

i) 3 2 22NH g N g +3H g

ii) 2 2 3

1 3N g + H g NH g

2 2

iii) 2 2 32N g +6H g 4NH g

1) i) 1681, ii) 2.44×10–2

, iii) 604

2) i)0.0121, ii) 9.05, iii) 6724

3) i) 2.44×10–2

, ii) 6 .4, iii) 1681

4) i) 2.44×10–2, ii) 1681, iii) 6.4

3. Hydrolysis of sucrose gives

2Sucross H O Glucos e Fructose

Equilibrium constant cK for the reaction is 132 10 at 300K.

Calculate G at 300K.

1) 4 17.64 10 Jmol 2) 4 17.64 10 Jmol

3) 4 176.4 10 Jmol 4) 4 176.4 10 Jmol

4. the ester, ethyl acetate, is formed by the reaction of ethanol and

acetic acid and the equilibrium is represented as:

Page 146: 8 & 9_MPC_SHP_E1 & E2& E4 & E5

CLASS-IX_SUBGROUP_E4 & E5 SIMPLE HOLIDAY PACKAGE

NARAYANA GROUP OF SCHOOLS Page 32 of 45

3 2 5 3 2 25CH COOH C H OH CH COOC H H O

Starting with 0.500 mol of ethanol and 1.000 mol of acetic acid

and maintaining it at 293K, 0.214 mol of ethyl acetate is found

after some time. Find the Qc.

1) 2.0 2) 0.20

3) 20 4) none

5. In an equilibrium reaction for which G 0 , the equilibrium

constant K should be equal to:

1) Zero 2) 10

3) 1 4) 2

6. If a mixture where Q=K is combined, what happens?

1) nothing appears to happen, but forward and reverse reactions are continuing at the same rate

2) the reaction shifts towards products

3) the reaction shifts toward reactants

4) nothing happen

7. For the reaction 2 5 2 22 4

g g gN O NO O

the units of Kc is:

1) 2 2mol L 2) mol/lit

3) no units 4) 3 3mol L

8. The equilibrium constant, p

K for the reacti

2 24 2 6

g g gC H H C H is 17 1

5.04 10 atm at 25ºC Calculate ºG

1) 101kJ 2) -101kJ

3) 51.010 10 kJ 4) 51.010 10 kJ

9. For the reaction ( )3 2 SS g

CaCO CaO CO the units of Kc is:

1) 2 2mol L 2) -1molL

3) no units 4) 3 3mol L

10. A mixture of 1.5 mol of 2N , 2.0 mol of

2H and 8.0 mol of 3NH is

introduced into a 20L vessel a 500K. The reaction sets in is

2 2 33 2 ;

g g gN H NH 221.7 10 /

cK mol L

Predict the direction in which the reaction proceeds.

Page 147: 8 & 9_MPC_SHP_E1 & E2& E4 & E5

CLASS-IX_SUBGROUP_E4 & E5 SIMPLE HOLIDAY PACKAGE

NARAYANA GROUP OF SCHOOLS Page 33 of 45

1) Reaction proceeds in the forward direction

2) Reaction proceeds in the backward direction

3) Reaction present at equilibrium

4) None

11. for an equilibrium reaction, which one of the following is not

true?

1) If the reaction quotient of the reaction is

greater than eqK , the

reaction has a tendency of move in the backward direction

2) If the reaction quotient of the reaction

is greater than eqK , the reaction has a

tendency to move in the forward direction

3) If the reaction quotient of the reaction is equal

to eqK , the reaction is

at equilibrium

4) If the reaction quotient of the reaction

is smaller than eqK , the reaction has a

tendency to move in the forward direction.

12. For the reaction 2 2 g 3 g

1SO O SO

2 if xp cK K RT where the

symbols have usual meaning then the value of x is (assuming

ideality)

1) 1 2) -1

3) 1/2 4) 1/2

13. For the reaction 2 2H g I g 2HI g , the standard free energy

is G > 0.The equilibrium constant (K) would be __________.

1) K = 0 2) K>1

3) K=1 4) K<1

14. 13.5 ml of HI are produced by the interaction of 8.1 ml of

hydrogen and 9.3ml of iodine vapour at 444ºC . Calculate the

equilibrium constant at this temperature of the reaction.

2 g 2 g gH I 2HI

1) 13.5 2) 6.75

3) 5.294 4) 52.94

Page 148: 8 & 9_MPC_SHP_E1 & E2& E4 & E5

CLASS-IX_SUBGROUP_E4 & E5 SIMPLE HOLIDAY PACKAGE

NARAYANA GROUP OF SCHOOLS Page 34 of 45

15. Some nitrogen and hydrogen gases are pumped into an empty

five-litre glass bulb at 500ºC . When equilibrium is established

3.00 moles of 2N . 2.10 moles of 2H and 0.298mole of 3NH are

found to be present. Find the value of cK for the reaction

2 g 2 g 3 gN 3H 2NH

1) 0.080 2) 8.00

3) 0.033 4) 3.33

16. The ester, ethyl acetate, is formed by the reaction of ethanol

and acetic acid and the equilibrium is represented as:

3 2 5 3 2 25CH COOH C H OH CH COOC H H O

What is the concentration, Qc for this reaction.(Note that water

is not in excess and is not a solvent in this reaction)

1) 3 2 5 2

3 2 5

[CH COOC H (l)[H O(l)]

[CH COOH(l)][C H OH(l)]

2) 3 2 5

3 2 5

[CH COOC H (l)]

[CH COOH(l)][C H OH(l)]

3) 3 2 5

3 2 5

[CH COOH(l)][C H OH(l)]

[CH COOC H (l)] 4)

3 2 5

3 2 5 2

[CH COOH(l)][C H OH(l)]

[CH COOC H (l)[H O(l)]

17. In which of the following systems doubling the volume of the container causes a equilibrium shift to right

1) 2 2

2 2g g g

CO O CO 2) 5 3 2g g g

PCl PCl Cl

3) 2 2 3

3 2g g g

N H NH 4) 2 2

2g g g

H I HI

18. The conjugate acid of hydride Ion is

1) 3

H O 2) H2

3) OH 4) H2O

19. Which is not a conjugate pair of acid-base?

1) 2, HS S 2) 3 , H O OH

3) 2, HONO NO 4)

6 5 6 5, C H COOH C H COO

20. For the chemical reaction

33g g g

X Y X Y ; the amount of 3X Y at equilibrium is affected

by

1) Temperature and pressure

2) Temperature only

Page 149: 8 & 9_MPC_SHP_E1 & E2& E4 & E5

CLASS-IX_SUBGROUP_E4 & E5 SIMPLE HOLIDAY PACKAGE

NARAYANA GROUP OF SCHOOLS Page 35 of 45

3) Pressure only 4) Temperature, pressure and catalyst

21. G for the reaction X Y C is 4.606 kcal at 1000 K. The

equilibrium constant for the reverse mode of the reaction will be:

1) 100 2) 10

3) 0.01 4) 0.1

22. In the reaction the 3 2 2

2 4 4 4 2 4HC O PO HPO C O Bronsted Bases

are

1) 3 2

4 2 4,PO C O 2) 3 2

4 4,HPPO O

3) 2

2 4 4,HC O HPO 4) 2

2 4 2 4,CHC O O

23. What is G for the following reaction ?

4

2 2 3 p

1 3N g H g NH g ;K 4.42 10

2 2 at 250C.

1) 126.5kJmol 2) 111.5kJmol

3) 12.2kJmol 4) 10.97kJmol

24. In the following reaction

2

aq aq2 3 2 2 3 aqaq4 3A C DB

Cu H O HCO Cu H O OH H CO

Species behaving as Bronsted – Lowry acids are

1) A, D 2) B, C

3) A, B 4) B, D

Paragraph for Questions 25 to 27

According to Lewis concept, a base is defined as a substance which can furnish a pair of electrons to form a co-ordinate bond where as an acid is a substance which can accept a pair of electrons. The acid is also known as electron acceptor or electrophile while the base is

electron donor (or) nucleophile.

Which of the following acts a Lewis acid?

25.

1) H+ 2) H2C=CH2

3) BF3 4) both 1 & 3

26. Among the following which one acts as Lewis base?

i) C2H2 ii) AlCl3 iii) 2

RNH iv) S

1) i, ii, iii & iv 2) i & iii

Page 150: 8 & 9_MPC_SHP_E1 & E2& E4 & E5

CLASS-IX_SUBGROUP_E4 & E5 SIMPLE HOLIDAY PACKAGE

NARAYANA GROUP OF SCHOOLS Page 36 of 45

3) only iv 4) only iii

27. Pick out odd one from the following

1) C2H4 2) C2H2

3) 3

CH 4) CN

Paragraph for Questions 28 to 30

Phosphorous pentachloride when heated in a sealed tube at 700k it decomposition as

5 3 2g g gPCl PCl Cl

; Kp = 38atm

Vapour density of the mixture is 74.25

If pressure is increased then equilibrium will;

28.

1) Be unaffected 2) Shift in backward direction

3) Shift in forward direction

4) Cannot be predicted

29. When inert gas is added to the given reversible process then the equilibrium will:

1) Be unaffected 2) Shift in backward direction

3) Shift in forward direction

4) Cannot be predicted

30. Which among the following reactions will be favoured at low pressure?

1) 2 g 2 g gN O 2NO 2) 2 g 2 g g

H I 2HI

3) 5 g 3 g 2 gPCl PCl Cl 4) 2 g 2 g 3 g

N 3H 2NH

31. The ionisation constant of an acid, Ka, is the measure of strength of an acid. The Ka values of acetic acid, hypochlorous

acid and formic acid are -51.74 × 10 , -83.0 × 10 and -41.8 × 10respectively. Which of the following Orders of pH of 0.1 mol dm–

3 solutions of these acids are correct?

1) Acetic acid > hypochlorous acid > formic acid

2) Hypochlorous acid > acetic acid > formic acid

3) Formic acid > hypochlorous acid > acetic acid

4) Formic acid > acetic acid > hypochlorous acid

32. What will be the value of pH of 0.01 mol dm–3 CH3COOH (Ka =

Page 151: 8 & 9_MPC_SHP_E1 & E2& E4 & E5

CLASS-IX_SUBGROUP_E4 & E5 SIMPLE HOLIDAY PACKAGE

NARAYANA GROUP OF SCHOOLS Page 37 of 45

1.74 × 10–5 )?

1) 3.4 2) 3.6

3) 3.9 4) 3.0

33. Arrange the following in increasing order of pH. KNO3 (aq), CH3COONa (aq), NH4Cl (aq), C6H5COONH4 (aq)

1) NH4Cl < C6H5COONH4 < KNO3 < CH3COONa

2) C6H5COONH4 < KNO3 < CH3COONa< NH4Cl

3) NH4Cl < CH3COONa <C6H5COONH4 < KNO3

4) NH4Cl < KNO3 < CH3COONa< C6H5COONH4

34. The dissociation constants of mono basic acids A,B,C and D are 4 5 6 106 10 ,5 10 ,3.6 10 and7 10 respectively. The

HP values of

their 0.1 molar aqueous solutions are in the order:

1) A B C D 2) A B C D

3) A B C D 4) A B C D

35. The degree of dissociation of 0.01M 3CH COOH is 4.24 %. Then

degree of dissociation of 0.1M 3CH COOH will be

1) 1.34% 2) 4.24%

3) 5.24% 4) 0.33%

36. HP of 0.005M HCOOH 4Ka 2 10 is equal to

1) 3 2) 2

3) 4 4) 5

37. Degree of dissociation of two weak acids 1 2and are in ratio of

1:2 ; 41Ka 2 10 then what will be 2Ka ?

1) 48 10 2) 42 10

3) 44 10 4) 41 10

38. Ostwald’s dilution law is applicable in the case of the solution of

1) NaCl 2) NaOH

3) 2 4

H SO 4) 3

CH COOH

39. The dissociation constants of two acids 1

HA and 2

HA are

43 10 and 51.8 10 respectively. The relative strength of the

acids will be:

1) 1 : 4 2) 4 : 1

3) 1 : 16 4) 16 : 1

Page 152: 8 & 9_MPC_SHP_E1 & E2& E4 & E5

CLASS-IX_SUBGROUP_E4 & E5 SIMPLE HOLIDAY PACKAGE

NARAYANA GROUP OF SCHOOLS Page 38 of 45

40. Degree of Dissociation of 0.001 M 3CH COOH is 5ka 1.0 10

1) 610 2) 710

3) 110 4) 210

41. Dissociation constant of NH3 is 51.8 10 calculate the

dissociation constant of its conjugate acid 4NH 14w[K 10 ]

1) 1055 10 2) 105.5 10

3) 105.5 10 4) 1055 10

42. Which of the following is the correct quadratic form of the Ostwald’s Dilution Law equation?

1) 2 0C K K 2) 2 0C K K

3) 2 0C K K 4) 2 0C K K

43. A monoprotic acid in 1.00 M solution is 0.001% ionized. The dissociation constant of acid is:

1) 31.0 10 2) 61.0 10

3) 81.0 10 4) 101.0 10

44. The 'apK for acid A is greater that apK for acid B. The strong

acid is:

1) Acid A 2) Acid B

3) are equally strong 4) None of these

45. pH of water is 7.0 at 25°C. If water is heated to 70°c, the:

1) pH will decrease and solution becomes acidic

2) pH will increase and solution becomes basic

3) pH will remain constant as 7

4) pH will decrease but solution will be natural

46. The role of catalyst in reversible reaction is :

1) To increase the rate of forward reaction

2) Decrease the rate after equilibrium

3) Allow equilibrium to be achieved quickly

4) None of these

47. Which statement/relationship is correct:

Page 153: 8 & 9_MPC_SHP_E1 & E2& E4 & E5

CLASS-IX_SUBGROUP_E4 & E5 SIMPLE HOLIDAY PACKAGE

NARAYANA GROUP OF SCHOOLS Page 39 of 45

1) Upon hydrolysis salt of a strong base and weak acid gives a solution with pH<7

2) 1pH log

H

3) Only at 25°C the pH of water is 7

4) The value of pKw at 25°C is 7

48. which equilibrium can be described as Lewis acid-base reaction but not Bronsted acid-base reaction:

1) 2 3 3 3H O CH COOH H O CH COO 2) 23 2 4 4 42NH H SO 2NH SO

3) 3 3 4 3NH CH COOH NH CH COO

4) 2 22 3 3 244

[Cu H O ] 4NH [Cu NH ] 4H O

49. The two Bronsted bases in the reaction are 3 2 2

2 4 4 24 4HC O PO HPO C O

1) 22 4 4HC O and PO 2) 3 3 2

4 42HPO and C O

3) 3 24 2 4PO and C O 4) 2

2 4 4HC O and HPO

50. Which does not act as Bronsted acid :

1) 4NH 2)

3CH COO

3) 3HCO 4)

3HSO

51. What is the HP of the buffer solution containing 0.15 moles of

4NH OH and 0.25 moles of

4NH Cl . bK for

4NH OH

51.8 10

[log(5) = 0.6989, log(3) = 0.4771, log(1.8) = 0.2552]

1) 4.96 2) 9.034

3) 8.23 4) 4.744

52. At 25 C the value of bkP for 3NH in aqueous solution is 4.7. What

is the HP of 0.1M aqueous solution of 4NH C with 0.01M 3NH ?

1) 5.7 2) 8.3

3) 4.7 4) 9

53. In the acid-base relation

3 3 2HCl CH COOH Cl CH COOH

Page 154: 8 & 9_MPC_SHP_E1 & E2& E4 & E5

CLASS-IX_SUBGROUP_E4 & E5 SIMPLE HOLIDAY PACKAGE

NARAYANA GROUP OF SCHOOLS Page 40 of 45

The conjugate acid of acetic acid is

1) Cl 2) HCl

3) 3 2

CH COOH 4)

3H O

54. For the following reaction

3 3 2

2aq aq

Ag NH Ag NH , 7

4 10K at 025 c Find 0

G ? ( 2 0.301)log

1) -53.375KJ/mole 2) +43.375KJ/mole

3) 53.375KJ/mole 4) -43.375KJ/mole

55. A solution of HCl has 5HP . If 1ml of it is diluted to 1 liter,

what will be the HP of the resulting solution?

1) 6.95 2) 7.56

3) 8.64 4) 6.23

56. for the reversible reaction 2 2 3N (g) 3H (g) 2NH (g) heat , The

equilibrium shifts in forward direction

1) by increasing the concentration of

3NH (g)

2) by decreasing the pressure

3) by decreasing the

concentration of 2N (g)

and 2H (g)

4) by increasing pressure and decreasing

temperature

57. The standard Gibbs energy change at 300 K for the reaction,

2A B C is 2494.2 J. At a given time, the composition of

the reaction mixture is 1 1

(A) , (B) 2 and (C) .2 2

the reaction

proceeds in the: (R) 8.314 J/K/mol, e 2.718)

1) forward direction

because Q > Kc

2) reverse direction because Q > Kc

3) forward direction

because Q < Kc

4) reverse direction because Q < Kc

58. Paragraph for Questions 58 to 60

Many reactions, particularly the bio chemical reactions are to be

carried out at constant HP . But it is observed that solutions and

Page 155: 8 & 9_MPC_SHP_E1 & E2& E4 & E5

CLASS-IX_SUBGROUP_E4 & E5 SIMPLE HOLIDAY PACKAGE

NARAYANA GROUP OF SCHOOLS Page 41 of 45

even pure water cannot retain the constant HP for long time. But few

solutions resist change of HP upon addition of small amount of acid

or alkali. These solutions are called Buffer solutions.

A solution is 1M in 3CH COO Na and 10M in 3CH COOH .If kaP of

3CH COOH is 4.8.What is the HP of the solution?

1) 3.8 2) 5.8

3) 4.9 4) 6.8

59. Which of the following statements is/are correct for a buffer

solution.

1) It has reserve acidity

or Alkalinity

2) Its

HP is slightly changed by the

addition of small quantity of an acid

or a base.

3) Its

HP changes on

dilution

4) Both 1 and 2

60. A basic Buffer solution contains a weak base B and its conjugate

acid BH . On adding some HC , which of the following

reactions takes place to maintain constant HP ?

1) BH B H

2)

2B H O BH OH

3)

2H OH H O

4)

2BH OH B H O

61.

Paragraph for Questions 61 to 63

The degree of dissociation of weak electrolyte is inversely proportional to the square root of concentration. It is called Ostwald’s law. If we know the strength of one Acid, the other Acid strength can be calculated.

The degree of dissociation of 0.01M 3CH COOH is 4.24 %. Then

degree of dissociation of 0.1M 3CH COOH will be

1) 1.34% 2) 4.24%

3) 5.24% 4) 0.33%

62. HP of 0.005M HCOOH 4Ka 2 10 is equal to

1) 3 2) 2

Page 156: 8 & 9_MPC_SHP_E1 & E2& E4 & E5

CLASS-IX_SUBGROUP_E4 & E5 SIMPLE HOLIDAY PACKAGE

NARAYANA GROUP OF SCHOOLS Page 42 of 45

3) 4 4) 5

63. Degree of dissociation of two weak acids 1 2and are in ratio of

1:2 ; 41Ka 2 10 then what will be 2Ka ?

1) 48 10 2) 42 10

3) 44 10 4) 41 10

64. ‘HX’ is a weak acid 510aK , It forms a salt ‘NaX’ (0.1M) on

reacting with caustic soda. The degree of hydrolysis of ‘NaX’ is

1) 0.01 2) 0.1 3) 0.0001 4) 0.5

65. 2.5mL of 2

5M weak monoacidic base ( 12

bK 1 10 at 025 C ) is

titrated with 2

15 M HCl in water at 025 C .The concentration of

H at equivalence point is: ( 14

wK 1 10 at 025 C )

1) 133.7 10 M 2) 73.2 10 M 3) 23.2 10 M 4) 22.7 10 M

66. Select the correct statements among the following

1) In the complete ionization of indicator

its InpH pK

2) Methyl orange (working range : 3.1 to 4.4) is a suitable indicator for when acid and strong base.

3) Bromothymol blue (working range 6.0 to 7.6) is a good indicator for titration of HCl and NaOH.

4) Thymol blue (working range 1.2 to 2.8) is good indicator for titration of

100 ml of 4.74bpK and 0.1 M HCl.

67. The rapid change of pH near the stoicho-metric point of an acid-

Base titration is the basis of indicator detection. pH of the

solution is related to ratio of the concentrations of the

conjugate acid(HIn) and base In forms of the indicator by the

expression

1) In

HInlog pK pH

In

2) In

HInlog pH pK

In

3)

In

Inlog pH pK

HIn

4)

In

Inlog pK pH

HIn

68. The HP of 0.1M solution of the following compounds increase in

the order

1) 3 2 2 4 KNO K S K SO 2)

2 2 4 3 K S K SO KNO

Page 157: 8 & 9_MPC_SHP_E1 & E2& E4 & E5

CLASS-IX_SUBGROUP_E4 & E5 SIMPLE HOLIDAY PACKAGE

NARAYANA GROUP OF SCHOOLS Page 43 of 45

3) 2 4 3 2 K SO KNO K S 4)

3 2 4 2 KNO K SO K S

69. Which of the following indicator is most suitable for titration of HBr with strong acid?

1) Phenolphthalein (8.3 – 10)

2) Bromothymol blue (6 - 7.6)

3) Methyl Red (4.2 – 6.3) 4) Malachite green (11.4 – 13)

70. The degree of hydrolysis of which of the following salt is independent of the concentration of salt solution?

1) CH3COONa 2) NH4Cl 3) CH3COONH4 4) NaCl

71. Which of the following expressions is not applicable on the hydrolysis equilibrium?

2CN H O HCN OH

1)

w

h

a

KK

K HCN

2) hK

hC

3) 1

2apH pK HCN

4)

w aK KH

C

72. The pKa of weak acid HA is 4.80 and the pKb of a weak base BOH

is 4.78. The pH of an aqueous solution of corresponding salt BA

will be

1) 9.22 2) 9.58

3) 4.79 4) 7.01

73. Dissociation of an indicator can be considered as,

HIn H In Colours of HIn and In– are different. Which

statement is correct?

1) Solution assumes

colours of HIn when

pH = pK – 1

2) Solution assumes colours of In– when

pH = pK – 1

3) Solution assumes

colours of HIn– when

pH = pK + 1

4) None

Page 158: 8 & 9_MPC_SHP_E1 & E2& E4 & E5

CLASS-IX_SUBGROUP_E4 & E5 SIMPLE HOLIDAY PACKAGE

NARAYANA GROUP OF SCHOOLS Page 44 of 45

74.

A certain monoprotic acid (weak) serves as indicator. Assuming

that colour change is seen when 1/3rd of the indicator has been

converted into ions and that at end point the pH of solution is

‘6’, the value of InpK

1) 7.3 2) 6.3

3) 5.3 4) 4.3

75. If pKb of CN– is 3.70, the pH of 0.5 M aqueous NaCN solution is

1) 10.8 2) 11.5

3) 11.99 4) 8.4

76. hK of salt obtained from strong acid and weak base is 55 10 . The

bK of weak base is

1) 192 10 2) 105 10

3) 102 10 4) 95 10

77. Calculate the pH at the equivalence point when a solution of

0.10 M acetic acid is titrated with a solution of 0.10M sodium

hydroxide. Ka for acetic acid 51.9 10 . (log 0.05= 1.3, log

1.9=0.27)

1) 8.71 2) 9.36

3) 6.485 4) 7.71

78. The pH of buffer solution prepared by mixing ‘50’ ml of 0.2 M,

CH3COOH and ‘25’ ml of CH3COONa is 4.8. What is the

concentration of CH3COONa? kaP of CH3COOH =4.8

1) 0.2 M 2) 0.4 M

3) 0.5 M 4) 0.8 M

Page 159: 8 & 9_MPC_SHP_E1 & E2& E4 & E5

CLASS-IX_SUBGROUP_E4 & E5 SIMPLE HOLIDAY PACKAGE

NARAYANA GROUP OF SCHOOLS Page 45 of 45

79. Useful buffer range of weak acid ‘HA’ 5Ka 10 is

1) 5 to 7 2) 4 to 6

3) 3 to 5 4) 7 to 9

80. One litre of an aqueous solution contain 0.15 mole of

Ka

3CH COOH P 4.8 and 0.15mole of 3CH COONa

. After the

addition of 0.05 mole of solid NaOH to this solution, the pH

will be (log 2 = 0.3010)

1) 4.5 2) 4.8

3) 5.1

4) 5.4

IX_CHEMISTRY_SHP_E4 & E5_KEY

Q.No. 1 2 3 4 5 6 7 8 9 10

Key 3 3 2 2 3 1 4 2 2 2

Q.No. 11 12 13 14 15 16 17 18 19 20

Key 2 3 4 4 1 1 3 4 2 2

Q.No. 21 22 23 24 25 26 27 28 29 30

Key 3 2 2 1 2 1 1 1 1 4

Q.No. 31 32 33 34 35 36 37 38 39 40

Key 4 1 1 1 1 1 1 4 2 3

Q.No. 41 42 43 44 45 46 47 48 49 50

Key 2 1 4 2 4 3 3 4 3 2

Q.No. 51 52 53 54 55 56 57 58 59 60

Key 4 1 2 1 2 2 1 2 2 1

Q.No. 61 62 63 64 65 66 67 68 69 70

Key 1 3 3 3 3 3 3 3 1 3

Q.No. 71 72 73 74 75 76 77 78 79 80

Key 3 4 1 2 3 3 1 2 2 3


Recommended